Sei sulla pagina 1di 269

Item: 1 of 64 ~.

I • M k <:] t> al ~· ~
QIO: 3935 .l. ar Previous Next lab 'Vfl1 ues Notes Calculator

•1 •
Cystic fibrosis (CF) shows autosomal recessive inheritance (rr) with an incidence of 1 in 2500.
•2
•3 Assuming that this population is in Hardy-Weinberg equilibrium, what va lue below is closest to the frequency of the het erozygote genotype ( Rr) in this
population?
•4
•5
A. 1/ 25
•6
•7 B. 1/ 2500

•8 c. 1/ 50
•9
D. 24/ 25
• 10
E. 49/ 50
· 11
• 12
• 13
• 14
• 15
• 16
• 17

• 18
• 19
• 20
• 21

Q
lock
s
Suspend
0
End Block
Item: 1 of 64 ~ 1 • M k -<:J 1>- Jil ~· !:';-~
QIO: 3935 ..L ar Prev ious Next Lab~lues Notes Calculat o r

& &
1 Th e co rrect a nsw er i s A. 520/o chose this.
.2 The Hardy-Weinberg equation can be used to calculate the genetic variation of a population at equilibrium. It is expressed as p2 + 2pq + q2 = 1, where p
Is the frequency of the dominant R allele and q is the frequency of the recessive r allele in the population. p2 represents the frequency of the homozygous
•3 genotype RR. 2pq represents the frequency of the heterozygous genotype Rr. q2 represents the frequency of the homozygous genotype rr. The sum of
allele frequencies must add to 1, therefore p + q = 1.
·4
In this example, we are informed that the frequency of the homozygous recessive genotype rr is 1/ 2500. Since q2 = 1/ 2500, then q must = 1/ 50.
•5
Remember that p + q = 1, so 1 - 1/ 50 = p = 49/ 50. 49/ 50 can be approximated to 1. Using this type of approximation is a valuable tool for efficiency for
•6 USMLE-style questions involving the Hardy-Weinberg equation.
.7 The frequency of the heterozygous genotype Rr, or 2pq. Since P"' 1, 2pq..,2q= 2*(1/ 50)= 1/25.
·8 Aooele Hete ozygous Genotype Homozygous Genetic variation Zygosity Recess;ve Aolele frequency Dominance (genetics)

.9 B i s n o t correct . 80/o chose this.

• 10 Using the Hardy-Weinberg equation, 1/ 2500 is the q 2 value. This means that 1/ 2500 is the frequency of the homozygous rr genotype. It is therefore not
the correct answer.
• 11 Homozygous Zygosity Genotype

• 12 C is no t co rrect. 26 0/o chose t his.


• 13 Using the Hardy-Weinberg equation, 1/ 50 is the q value . This means that 1/50 Is the frequency of the r allele in the population. It is therefore not the
correct answer.
• 14 Allele Hardy-Weinberg

• 15 D is no t co rrect . 60fo ch ose this.


• 16 Using the Hardy-Weinberg equation, 24/ 25 is the p 2 value . This means that 24/25 Is close to the frequency of the homozygous RR genotype. It Is
therefore not the correct answer.
• 17 Homozygous Zygosity Genotype

. 18 E i s not correct. 8 0/o ch ose this.


• 19 Using the Hardy-Weinberg equation, 49/50 is the p value, close to 1. This means that 49/50 is the frequency of the R allele in the population. It Is
therefore not the correct answer.
• 20 Allele P-value Hardy-Weinberg

• 21

a
Lock
s
Suspend
8
End Block
Item: 1 of 64 ~. I • M k <:] t> al ~· ~
QIO: 3935 .l. ar Previous Next lab 'Vfl1 ues Notes Calculator


. . - . -
1 Using the Hardy-Weinberg equation, 49/ 50 is the p value, close to 1. This means that 49/ 50 is the frequency of the R allele in the population . It is
•2 therefore not the correct answer.
Allele P-value Hardy-Weinberg
•3
•4
Bottom Line:
•5
The Hardy-Weinberg equation describes the frequencies of alleles and genotypes in stable populations .
•6 Hardy-Weinberg Genotype Allele Allele frequency
•7
•8
•9 I iii I;fi 1!1 I•J for year:[ 2017 "
FIRST AID FAC T S
• 10

· 11 FA17 p 53.1
• 12
Hardy-Weinberg Tf a population is in Hardy-Weinberg Hardy-Weinberg law assumptions include:
• 13 population genetics equilibrium and if p and q are the frequencies • o mutation occurri ng at the locus
• 14
pA qa of separa te alleles, then: p2 + 2pq + q2 = I and • Natural selection is not occurring
• 15 p + q = l, which implies that: • Completely random mating
• 16 pA
AA Aa p2 =frequency of homozygosity for allele A • f o net migration
p x p - p' pxq q 2 = frequency of homozygosity for allele a
• 17
Aa aa 2pq = frequency of heterozygosity (carrier
• 18 qa
px q q. q =q' frequency, if an autosomal recessive disease).
• 19 T he frequency of an X-Iinked recessive disease
• 20 in males= q and in females= q2.
• 21

6
lock
s
Suspend
0
End Block
Item: 2 of 64 ~ 1 • M k -<:J 1>- Jil ~· !:';-~
QIO: 5228 ..L ar Pre v ious Next Lab~lues Notes Calcula t o r

1
•2
&

A 47-year-old man presents to his primary care physician with complaints of new-onset difficulty in walking up stairs and rising from a seated position.
He has a history of cataracts and mild glucose intolerance. According to the patient, his mother had similar symptoms late in her life. Physical
IA•A] &

examination reveals the findings seen in the video clip .


•3
OPEN MEDIA
·4
•5
•6 Which of the following genetic mechanisms most likely accounts for this man's condition?

•7 :
·8 A. Framesh1ft mutation

.9 B. Gene deletion
• 10
c. Imprinting
• 11
o. Nucleotide repeat expansion
• 12
• 13 E. Robertsonian translocation

• 14
• 15
• 16
• 17
• 18
• 19
• 20
• 21

a
Lock
s
Suspend
8
End Bl ock
Item: 2 of 64 ~. I • M k <:] t> al ~· ~
QIO: 5228 .l. ar Previous Next lab 'Vfl1 ues Notes Calculator

1 •
The co rrect a nswer is D. 57 % cho se this.
2 Myotonic dystrophy is a myopathic disorder; and though type 1 often manifests in childhood or young adulthood, type 2 commonly develops among
•3 patients lat er in life. Proximal muscle weakness, often manifested as difficulty walking up stairs or rising from a chair; is an early sign. As seen in the
video, these patients demonstrat e delayed relaxation of muscles, characteristically observed in the hand. As opposed to patients with the type 1 disorder;
•4 these patients also frequently have cat aracts, mild glucose intolerance, and normal mental function . Type 2 myotonic dystrophy is associat ed with a
nucleotide repea t expansion (CCTG ) within the CNBP gene. Pa tients with type 1 myotonic dystrophy also frequently have a repea t expansion, but of a
•5 different gene.
•6 Myotonic dystrophy Gene Myopathy Nucleotide Muscle weakness Glucose Cataract Muscle Anatomical terms of location

•7 A is no t co rrect. 16 % cho se this.


Loss or gain of nucleotides (not in a multiple of three ), result in an "off-set" rea ding fra me (frameshift mutation) for the ribosomal translational
•8 machinery. Depending on the location of the mutation within the m RNA t ra nscript, the resultant protein can have variable levels of function . Duchenne
•9 muscular dystrophy results from a frameshift mutation within the dystrophin gene, producing a poorly active protein product .
Frameshift mutation Dystrophin Duchenne muscular dystrophy Gene Muscular dystrophy Mutation Nucleotide Translational frameshift Reading frame
• 10
Messenger RNA Protein Ribosome
· 11 B is no t co rrect. 13 % cho se this •
• 12 Gene deletion can produce multiple genetic disorders, depending on the size of the deletion and the genes involved. Known syndromes include cri-du-chat
syndrome and Wolf- Hirschhorn syndrome .
• 13 Wolf-Hirschhorn syndrome Cri du chat Gene Genetic disorder Deletion (genetics)
• 14
c is no t co rrect. 9 % cho se this •
• 15 Imprinting refers to inherited inactivation of a specific gene (eg, inherited regional methylation) ; deletion of the remaining active allele results in disea se.
Prader -Willi syndrome and Angelman syndrome are examples of disea ses resulting from imprinting .
• 16
Angelman syndrome Prader-Willi syndrome Allele Gene Methylation Genomic imprinting Imprinting (psychology)
• 17
E is no t co rrect. 5 % cho se this •
• 18 A robertsonian translocation is a nonreciprocal exchange of genetic information between two acrocentric chromosomes in which the fusion often results in
the loss of genetic information (usually at the short end of the chromosomes). Autosomal trisomies can result when individuals with silent robertsonian
• 19 translocations (themselves asymptomatic) procrea t e.
• 20 Centromere Acrocentric Autosome Chromosomal translocation Trisomy Chromosome

• 21

6
lock
s
Suspend
0
End Block
Item: 2 of 64 ~. I • M k <:] t> al ~· ~
QIO: 5228 .l. ar Previous Next lab 'Vfl1 ues Notes Calculator

1 • A robertsonian translocation is a nonreciprocal exchange of genetic information between two acrocentric chromosomes in which the fusion often results in
the loss of genetic information (usually at the short end of the chromosomes) . Autosomal trisomies can result when individuals with silent robertsonian
2 translocations (themselves asymptomatic) procreate.
Centromere Acrocentric Autosome Chromosomal translocation Trisomy Chromosome
•3
•4
•5 Bottom Line:

•6 Myotonic dystrophy is characterized by proximal muscle weakness and delayed muscle relaxation and m ay m anifest in childhood or adulthood depending
on the type . It is caused by a nucleotide expa nsion of CCTG.
•7 Myotonic dystrophy Nucleotide Muscle weakness Anatomical terms of location Muscle

•8
•9
• 10 Iiii I;fi IJ I•J for year:[ 2017 "
FI RST AID FAC T S
· 11
• 12 FA11 p 58.2
• 13 Trinucleotide repeat Try (trinucleotide) hunting for my fragile cage-
Hunti ngton disease, myotonic dys trophy,
• 14 expansion diseases fragile X syndrome, and Friedreieh ataxia. free eggs (X).
• 15 lay show genetic anticipation (disease severity
• 16 f and age of onset ! in successive generations).
Huntington disease = (CAG), C auda te has ! ACh and GABA
• 17
Myoton ic dystrophy= (CT G )11 C ataracts, Toupee (early balding in men),
• 18
G onadal atrophy
• 19 Fragile X syndrome = (CGG)11 C hin (protruding), G ian t G onads
• 20 Friedreich ataxia = (GAA)11 Ataxic GAAit
• 21

6
lock
s
Suspend
0
End Block
Item: 3 of 64 ~ 1 • M k -<:J 1>- Jil ~· !:';-~
QIO: 4863 ..L ar Pre v ious Next Lab~lues Notes Calcula t o r
& &
1
A 35-year-old woman presents with dull, persist ent flank and abdominal pain, polyuria, nocturia, and frequent urinary tract infections. Physical
2 examination is notable for blood pressure of 150/ 90 mm Hg and multiple bilateral abdominal m asses. Urinalysis is nota ble for microscopic hematuria
and 1+ protein. Gross pathology from a patient with a similar condition Is shown in the image. The patient notes t hat she was adopted and knows
•3 nothing about her family medical history.
·4
•5
•6
.7
·8
.9
• 10
• 11

• 12
• 13
• 14
• 15
• 16 What Is the most likely etiology of t his patient 's illness?
• 17
:
. 18 A . A mutation on chromosome 3
• 19
B. A mutation on chromosome 6
• 20
C. A mutation on chromosome 9
• 21

a
Lock
s
Suspend
8
End Bl ock
Item: 3 of 64
QIO: 4863 ~
~ , .1 • Mar k <1
Previous
t>
Next Lab
d\ ues
•.

Notes
~
calculator

1 A 35 -year-old woman presents with dull, persistent flank and abdominal pain, polyuria, nocturia, and frequent urinary tract infections , Physical examination is
notable for blood pressure of 150/ 90 mm Hg and multiple bilateral abdominal masses , Urinalysis is notable for m icroscopic hematuria and 1 + protein , Gross
2 pathology from a patient with a sim ilar condition is shown in the image , The patient notes that she was adopted and knows nothing about her family medical history,
•3
•4
•5
•6
•7
•8
•9
• 10
• 11

• 12
• 13
• 14
• 15
What is the most likely etiology of this patient's illness?
• 16
:
• 17
A. A mutation on chromosome 3
• 18
B. A mutation on chromosome 6
• 19
• 20 c. A mutation on chromosome 9

• 21 D. A mutation on chromosome 16
• 22
E. A mutation on chromosome X
• 23
• 24

a
Lock
s
Suspend
o
End Block
Item: 3 of 64 ~. I • M k <:] t> al ~· ~
QIO: 4863 .l. ar Previous Next lab 'Vfl1 ues Notes Calculator

2
The co rrect a nswer is D. 58 % cho se this.
3 The patient's symptoms are consistent with adult polycystic kidney disea se (APKD), a disea se most commonly caused by mutation of the polycystin 1 gene
.4 locat ed on chromosome 16. APKD can also be caused by a mutation in polycystin 2 locat ed on chromosome 4, although this is less common . APKD is
characterized by the growth of renal cysts, which are believed to lea d to renal failure by compressing adjacent normal parenchyma. The disea se often
•5 presents in the third or fourth decade of life, and symptoms and signs include abdominal discomfort, frequent urinary tract infections, hematuria, polyuria,
and nocturia. Cystic kidneys are often seen on imaging. Mild proteinuria is common . Hepatic cysts are also common . The disea se is also associat ed with
•6 berry aneurysms and mitral va lve prolapse.
Polyuria Hematuria Nocturia Proteinuria Polycystic kidney disease Gene Parenchyma Mitral valve prolapse Mitral valve Kidney Kidney disease
.7
Urinary tract infection Urinary system Cyst liver Chromosome 16 (human) Mutation Chromosome 4 (human) Prolapse Chromosome Abdominal pain
•8
A is no t co rrect. 13 % cho se this •
•9
A mutation on chromosome 3 is associat ed with von Hippei- Lindau disea se, which cannot only cause renal cysts but also retinal angiomas and central
• 10 nervous system hemangioblastomas.
Von Hippel-lindau disease Central nervous system Chromosome 3 (human) Mutation Chromosome Nervous system Cyst
· 11
B is no t co rrect. 14 % cho se this •
• 12
A mutation on chromosome 6 is associat ed with recessive polycystic kidney disea se, which is associat ed with a much younger age of onset .
• 13 Polycystic kidney disease Chromosome 6 (human) Recessive Kidney Dominance (genetics) Chromosome Mutation Kidney disease

• 14 c is no t co rrect. 13 % cho se this.


• 15 A mutation on chromosome 9 is associat ed with tuberous sclerosis, which cannot only cause renal cysts, but also presents with adenoma sebaceum and
central nervous system hamartomas.
• 16 Tuberous sclerosis Central nervous system Chromosome Mutation Chromosome 9 (human) Nervous system Adenoma Hamartoma Kidney Adenoma sebaceum

• 17 Cyst

• 18 E is no t co rrect. 2% cho se this •


Alport syndrome is an X-linked mutation in a basement membrane protein (COL4AS) causing glomerulonephropathy, sensioneural hearing loss, and ocular
• 19 defects.
• 20 Alport syndrome Basement membrane Protein Collagen, type IV, alpha 5 Mutation Sex linkage Hearing loss

• 21

6
lock
s
Suspend
0
End Block
Item: 3 of 64 ~. I • M k <:] t> al ~· ~
QIO: 4863 .l. ar Previous Next lab 'Vfl1 ues Notes Calculator

1
Bottom Line:
2 This patient most likely has adult polycystic kidney disease, a life-threatening condition that is mostly commonly (85%) caused by a mutation on
3 chromosome 16 . The growth of renal cysts in this disease can cause renal failure .
Polycystic kidney disease Chromosome 16 (human) Kidney Chromosome Mutation Kidney disease Cyst
.4
•5
•6
lijj ;fi IJ l•l for year:l 2017 ..
FI RST AID FAC T S
.7
•8
FA11 p 56.1
•9
• 10
Autosomal dominant Achondroplasia, autosomal dominant polycystic kidney disease, fam ilial adenomatous polyposis,
diseases familial hypercholesterolemia, hereditary hemorrhagic telangiectasia, hereditary spherocytosis,
· 11
Huntington disease, Li-Fraumeni syndrome, Marfan syndrome, multiple endocrine neoplasias,
• 12 neurofibromatosis type l (von Recklinghausen disease), neurofibromatosis type 2, tuberous
• 13 sclerosis, von H ippei-Lindau disease.
• 14
• 15 FA11 p 573.1

• 16 Renal cyst disorders


• 17 Autosomal dominant a
lumerous cysts in cortex and medulla causing bilateral enlarged kidneys ultimately destroy
• 18
polycystic kidney kidney parenchyma. Presents with Aank pain, hematuria, hypertension, urinary infection,
disease progressi,·e renal fai lure in - 50% of individuals.
• 19
Mutation in PKDl (85% of cases, chromosome 16) or PKD2 (15% of cases, chromosome 4). Death
• 20 from complications of chronic kidney disease or hypertension (caused by t renin production).
• 21 Associated with berry aneurysms, mitral ,·alve prolapse, benign hepatic cysts, diverticulosis.

6
lock
s
Suspend
0
End Block
Item: 4 of 64 ~ 1 • M k -<:J 1>- Jil ~· !:';-~
QIO: 3934 ..L ar Pre v ious Next Lab~lues Notes Calcula t o r
& &
1
A 22-year-old man enters the clinic with a wide-stepping gait, muscle wasting, myotonia, and bilateral cataracts. His father, who is 55, currently
2 suffers from similar symptoms, which began about 5 years ago. His father's sister also exhibits similar symptoms; hers also began in her 50s. The
patient's mother exhibits none of these symptoms.
3
•4 What Is the inheritance pattern of this patient's most likely disease?
•5
:
•6 A. Autosomal dominant
•7
B. Autosomal recessive
·8
c. Mitochondrial
.9
• 10 o. X-llnked recessive

• 11 E. Y - linked
• 12
• 13
• 14
• 15
• 16
• 17
• 18
• 19
• 20
• 21

a
Lock
s
Suspend
8
End Bl ock
Item: 4 of 64 ~ 1 • M k -<:J 1>- Jil ~· !:';-~
QIO: 3934 ..L ar Prev ious Next Lab~lues Notes Calculat o r

& &
1
2 Th e correct an sw er i s A. 7 0 0/o chose this.
3 Myotonic dystrophy is an autosomal dominant disease that is transferred from generation to generation via a CTG trinucleotide repeat. Myotonic
dystrophy Is characterized by muscle wasting (muscular dystrophy), cataracts, heart conduction defects, and myotonia, or difficulty relaxing muscles.
4 Myotonic dystrophy is a genetic disease that illustrates the principle of anticipation, in which children of those with the disease allele can acquire more
copies of the mutated trinucleotide, and thus exhibit disease symptoms at earlier ages. Understanding the role of anticipation in myotonic dystrophy Is
•5
especially important when affected individuals contemplate reproduction. A father woth a 50-150 CTG repeat expansion in the causative gene can have a
•6 child with a 1000-1500 CTG repeat expansion as a result of anticipation. This change causes the progression to the disease phenotype.
Myotonic dyst op y A lele Gene Dominance (genetics) Myotonoa Muscular dyst ophy Ge oetic disorder Trinucleotide repeat disorder Autosome Muscle atrophy
•7
Phenotype Cataract Muscle
·8
B i s not correct. 12% chose this.
.9 Myotonic dystrophy is autosomal dominant.
Myotonic dystrophy Dominance (genetics) Autosome
• 10
• 11 C i s n ot co rrect. 6 % chose this •
Mitochondrial diseases with myopathic features, such as the MELAS syndrome (meaning Mitochondrial myopathy, Encephalopathy, Lactic Acidosis, and
• 12 Stroke-like episodes), can only be passed down on the maternal side. Patients with MELAS syndrome usually present in childhood with symptoms such as
vomiting, recurrent headaches, muscle weakness and strokes, but not bilateral cataracts. While the penetrance of mitochondrial diseases can vary, they
• 13
do not exhibit anticipation.
• 14 MELAS syndrome Mitochondrial disease Mitochondrial DNA Myopathy Mitochondrion Penetrance Vomiting Cataract Muscle weakness Muscle

• 15 D is not co rrect . 110/o ch ose this .


Myotonic dystrophy is not X linked. While X-linked recessive disease can affect males who receive the disease allele from their mothers, women such as
• 16
this patient's aunt should not normally show symptoms themselves unless they have two copies of the recessive X allele.
• 17 Myotonic dystrophy Allele Dominance ( genetics) X- linked recessive inheritance Recessive Sex linkage

• 18 E is n ot correct . 1 o;o ch ose this .

• 19 While Y-llnked traits can be passed on from father to sons, myotonic dystrophy is an autosomal-dominant disease. This patient's aunt could not acquire a
Y-llnked trait.
• 20 Myotonic dystrophy Domonance (genetics) Genetic disorder Y linkage

• 21

a
Lock
s
Suspend
8
End Block
Item: 4 of 64 ~. I • M k <:] t> al ~· ~
QIO: 3934 .l. ar Previous Next lab 'Vfl1 ues Notes Calculator

1 Botto m Line:
2 Myotonic dystrophy is an autosomal-dominant disorder characterized by muscle wasting, a wide-stepping gait, and bilateral cataracts. It is caused by a
CTG trinucleotide repeat and exhibits anticipation, whereby the number of CTG repeats can increase across generations and cause symptoms at younger
3 ages.
Myotonic dystrophy Genetic disorder Trinucleotide repeat disorder Dominance (genetics) Muscle atrophy Gait Cataract Muscle
4

•5
•6
.7
lijj ;fi IJ l•l fo r yea r:l 20 17 ..
FI RST AI D FA CTS

•8
•9 FA17 p 58.2

• 10 Trinucleotide repeat Huntington disease, myoton ic dystrophy, Try (trinucleotide) hunting for my fragile cage-
· 11 expansion diseases fragile X syndrome, and Fricdreieh ataxia. free eggs (X).
May show genetic anticipation (disease severity
• 12
f and age of onset l in successive generations).
• 13
Huntington disease = (CAC )11 C auda te has l ACh and C ABA
• 14 lyotonic dystrophy = (C TC)n C ataracts, Toupee (early balding in men),
• 15 G onadal atrophy
• 16 Fragile X syndrome = (CCC)11 C hin (protruding), G iant G onads
Friedreich ataxia = (C AA), Ataxic G AAit
• 17

• 18
FA17 p 52.2
• 19
Genetic terms
• 20
TERM DEFINITION EXAMPLE
• 21 Codominance Both alleles contribute to the phenotype of the Blood groups A. B. AB; a ,-antitrypsin

6
lock
s
Suspend
0
End Block
Item: 4 of 64 ~. I • M k <:] t> al ~· ~
QIO: 3934 .l. ar Previous Next lab 'Vfl1 ues Notes Calculator

1 • •
FA17 p 52.2
2 Genetic terms
3 TERM DEFINITION EXAMPLE
4 Codominance Both alleles contribute to the phenotype of the Blood groups A, B, AB; a 1-antitrypsin
•5 heterozygote. deficiency.
•6 Variable expressivity Patients with the same genotype have varying 2 patients with neurofibromatosis type 1 ( 1Fl)
•7 phenotypes. may have varying disease severity.
•8 Incomplete ot all individuals with a mutant genotype BRCA.l gene mutations do not always result in
•9 penetrance show the mutant phenotype. breast or ovarian cancer.
• 10 Pleiotropy One gene contributes to multiple phenotypic Untreated phenylketonuria (PKU) manifests with
· 11 effects. light skin, intellectual disability, and musty body
odor.
• 12
• 13
Anticipation Increased severity or earlier onset of disease in Trinucleotide repeat diseases (cg, Huntington
succeeding generations. disease).
• 14
• 15 Loss of heterozygosity If a patient inherits or develops a mutation in Retinoblastoma and the "two-hit hypothesis,"
a tumor suppressor gene, the complementary Lynch syndrome (Hl PCC), Li-Fraumcni
• 16
allele must be deleted/mutated before cancer syndrome.
• 17
develops. This is not true of oncogenes.
• 18
Dominant negative Exerts a dominant effect. A heterozygote Mutation of a transcription factor in its allosteric
• 19
mutation produces a nonfunctional altered protein that site. onfunctioning mutant can still bind
• 20 also prevents the normal gene product from DNA, preventing wild-type transcription factor
• 21 functioning. from binding. •

6
lock
s
Suspend
0
End Block
Item: 5 of 64 ~ 1 • M k -<:J 1>- Jil ~· !:';-~
QIO: 3355 ..L ar Pre v ious Next Lab~lues Not es Calcula t o r
& &
1
During a routine self-examination, a 33-yea r-old woman finds a lump In her right breast. Her doctor performs a biopsy on the mass, which Indicates
2 that It Is a malignant tumor. Because the patient has an extensive family history of breast and ovarian cancers, the physician sends a blood sample
for genotyping. Results indicate that the woman is heterozygous for a germllne mutation in the BRCA1 gene.
3
4 Which of the following diseases features an etiologic process that is most similar to the disease process in this patient?
•5
:
•6 A. Burkitt's lymphoma
•7
B. Chronic myelogenous leukemia
·8
c. Familial adenomatous polyposis
.9
• 10 D. Follicular lymphoma

• 11 E. Multiple endocrine neoplasia, type II


• 12
• 13
• 14
• 15
• 16
• 17
• 18
• 19
• 20
• 21

a
Lock
s
Suspend
8
End Bl ock
Item: 5 of 64 ~. I • M k <:] t> al ~· ~
QIO: 3355 .l. ar Previous Next lab 'Vfl1 ues Notes Calculator

1
The co rrect a nswer is c. 63% cho se this.
2 Since BRCA l is a tumor suppressor gene and this woman carries one functional copy of the gene, there is a high probability that a spontaneous mutation
will disrupt that functioning copy. This predisposes women with the mutation toward ovarian or brea st carcinomas. This process is called loss of
3 het erozygosity, and is necessary for het erozygous tumor suppressor mutations to cause cancer. Familial adenomatous polyposis (FAP) is a similar disea se,
4 in which the APC tumor suppressor gene on chromosome 5 is delet ed. Over time, there is a 100% chance that a second hit will occur; lea ding to a loss of
het erozygosity that strongly predisposes colorectal cells to dysplasia, and eventually to malignant transformation .
5 Familial adenomatous polyposis BRCAl Heterozygous Tumor suppressor gene Gene Neoplasm Mutation loss of heterozygosity Chromosome s (human)
•6 Malignant transformation Adenomatous polyposis coli Chromosome Dysplasia Heterozygosity Malignancy Zygosity Cancer Carcinoma

.7 A is no t co rrect. 6 % cho se this.


Burkitt's lymphoma is caused by a dominant, gain of function mutation in the c-myc oncogene resulting from chromosomal translocation of c-myc onto the
•8 immunoglobulin hea vy-chain locus.
•9 Chromosomal translocation Burkitt' s lymphoma Myc Oncogene Mutation locus (genetics) Antibody lymphoma Genetic engineering Protein targeting Chromosome

• 10 B is no t co rrect. 7 % cho se this •


Chronic myelogenous leukemia is caused by a dominant, gain of function mutation resulting from formation of the bcr-abl fusion protein by chromosomal
· 11 translocation .
• 12 Philadelphia chromosome Chromosomal translocation Chronic myelogenous leukemia leukemia Fusion protein Protein Mutation Genetic engineering
Protein targeting Chromosome
• 13
• 14 D is no t co rrect. 7 % cho se this •
Follicular and undifferentiat ed lymphomas can be caused by dominant, gain of function mutations in the bcl-2 oncogene resulting from chromosomal
• 15 translocation of bcl-2 into the immunoglobulin hea vy chain locus.
Chromosomal translocation Bcl-2 Oncogene Antibody locus (genetics) lymphoma Mutation Follicular lymphoma Heavy chain Genetic engineering
• 16
Immunoglobulin heavy chain Protein targeting
• 17
E is no t co rrect. 1 7 % cho se this •
• 18
Multiple endocrine neoplasia types II and III are due to a dominant, gain of function mutation in the ret oncogene.
• 19 Oncogene Multiple endocrine neoplasia Endocrine system Mutation Neoplasm Genetic engineering

• 20
• 21 Botto m Li ne:

6
lock
s
Suspend
0
End Block
Item: 5 of 64 ~. I • M k <:] t> al ~· ~
QIO: 3355 .l. ar Previous Next lab 'Vfl1 ues Notes Calculator

6
lock
s
Suspend
0
End Block
Item: 5 of 64 ~ 1 • M k -<:J 1>- Jil ~· !:';-~
QIO: 3355 ..L ar Pre v ious Next Lab~lues Not es Calcula t o r
& &
1
FA17 p 371 .2
2
Colorectal cancer
3
EPIDEMIOLOGY 1\ lost patients are> 50 )ears old. - 25% ha,e a
4 family history.
5 RISK FACTORS Adenomatous and serrated pol) ps, familial
•6 cancer syndromes, IBD, tobacco use, diet of
.7 processed meat with low fiber.
·8 PRESENTATION Rectosigmoid >ascending> descending. Right side bleeds; left side obstructs.
.9 Ascending-exophytic mass, iron deficiency
• 10
anemia, \\eight loss.
Descending-infiltrating mass, partial
• 11
obstruction, colickr pain, hematochczia .
• 12 Rarely, presents with S bovis (gct llolyticus)
• 13 bacteremia.
• 14 DIAGNOSIS Iron defi ciency anemia in ma les (especially> 50
• 15 years old) and postmenopausa l females raises
• 16 suspicion .
Screen low-risk patients starting at age 50 wit h
• 17
colonoscopy r.J; alternatives include flexible
. 18
sigmoidoscopy, fecal occult blood lesting
• 19 (F'OBT ), fecal immunochemical testing
• 20 (FIT }, and CT colonography. Patients with
• 21 a first-degree relati,·e who has colon cancer
• _L ___ I.J L _ _______ .) _. :_ - - • - ---~--- _ .._ - - ·

a
Lock
s
Suspend
8
End Bl ock
Item: 5 of 64 ~ 1 • M k -<:J 1>- Jil ~· !:';-~
QIO: 3355 ..L ar Pre v ious Next Lab~lues Not es Calcula t o r
& &
1
FA17 p 218.2
2
Tumor suppressor Loss of function ... t cancer risk; both (h, o) alleles of a tu mor suppressor gene must be lost for
3
genes expression of disease.
4
GENE GENE PRODUCT ASSOCIATED CONDITION
5
APC 'egative regulator of ~-eaten in/\ V1 T patlm ay Colorectal cancer (associated with r AP)
•6
BRCA 1/ BRCA2 D~A repair protein Breast, o,·arian, and pancreatic cancer
•7
CDKN2A pl6. blocks C 1 ... S phase r-.. telanoma, pancreatic cancer
·8
DCC D CC - D eleted in C olon C ancer Colon cancer
•9
DPC4/SMAD4 DPC- D eleted in Pancreatic C ancer Pancreatic cancer
• 10
MEN1 !\ lenin !\ lE N I
• 11
NF1 eurofi bromin (Ras GTPase activating protein) 1\eurofibromatosis type 1
• 12
• 13
NF2 Merlin (schwannomin) protein N eurofibromatosis type 2

• 14 PTEN Tyrosine phosphatase of PIP3 (eg, protein kinase Breast cancer, prostate cancer, endometrial
B (AKTJ acti vation) cancer
• 15
Rb Inhibits E2F; blocks C 1 ... S phase Reti noblastoma, osteosarcoma
• 16
• 17 TPS3 p53, ac tivates p21, blocks cl ... s phase lost human cancers, Li-Praumeni syndrome
(multiple malignancies at early age, aka, SBL.\
• 18
cancer syndrome: Sarcoma, Breast, Leukem ia,
• 19
\ drenal gland)
• 20
TSC1 Hamartin protein Tuberous sclerosis
• 21
• TSC2 T 11 ht>ri n nrn iPi n 'l i 1 ht'rn11~ ~I'IPrn~i ~

a
Lock
s
Suspend
8
End Bl ock
Item: 6 of 64 ~ 1 • M k -<:J 1>- Jil ~· !:';-~
QIO: 1662 ..L ar Pre vious Next Lab~lues Notes Calcula tor

1
&

A 27-year-old man presents to his family physician for an annual physical examination. On rectal examination, masses are palpated. The patient Is
referred for a colonoscopy, which reveals adenomatous polyps located diffusely throughout the colon . When asked about his family history, the patient
IA•A] &

2
states that his father passed away from colon cancer.
3
4 Which of the following inheritance patterns is characteristic of this condition?
5
:
•6 A. Autosomal dominant
•7
B. Autosomal recessive
·8
c. Mitochondrial
.9
• 10 o. Sporadic

• 11 E. X-llnked recessive
• 12
• 13
• 14
• 15
• 16
• 17
• 18
• 19
• 20
• 21

a
Lock
s
Su spend
8
End Bl ock
Item: 6 of 64 ~ 1 • M k -<:J 1>- Jil ~· !:';-~
QIO: 1662 ..L ar Previous Next Lab~lues Notes Calculator

& &
1
Th e correct a nsw er i s A . 850/o ch ose this.
2
Familial adenomatous polyposis (FAP) is an autosomal dominant condition characterized by a germline mutation on chromosome s, specifically at the
3 adenomatous polyposis coli (APC) locus. The APC gene is thought to have tumor-suppressive effects, and its loss is associated with more than colonic
cancers. In addition to duodenal neoplasms, for which these patients with FAP must undergo lifelong upper endoscopic surveillance, increased risk exists
4 for developing gastric, liver, thyroid, and central nervous system neoplasms. Another type of hereditary colon cancer is hereditary nonpolyposls colorectal
cancer, which is characterized by microsatellite deletions and is more severe In each subsequent generation.
5
Sma ntestine cancer Bram tumor Familial adenomatous polyposis Adenomatous po yposos co Germ ine mutation Hereditary nonpolyposis colorectal cancer
6 Colorectal cancer Microsatellite Central nervous system Gene Dominance (genetics) Nervous system neoplasm Mutation Neoplasm Chromosome s (human)
0 7 Endo~copy Uver Autosome Chromosome Germline Colon (anatomy) Nervous system Cancer Duodenum Large intestine Thyroid Deletion (genetics)

o8 B i s not correct. 100/o chose this.

.9 Examples of autosomal recessive conditions include cystic fibrosis, sickle cell anemia, and hemochromatosis.
Cystic fib osis Iron overload Sickle-cell disease Autosomal recessive Anemia Dominance genetics) Autosome Recessive Fibrosis
• 10
C i s n ot co rrect. 1 Ofo chose this •
• 11 Examples of conditions with mitochondrial inheritance include Leber hereditary optic neuropathy (LHON); mitochondrial encephalomyopathy, lactic
• 12 acidosis, and stroke-like episodes (MELAS); and myoclonic epilepsy with ragged red fibers (MERRF) .
MELAS syndrome Epilepsy Lactic acidosis Myoclonus Mitochondrial disease MERRF syndrome Leber's hereditary optic neuropathy Optic neuropathy
• 13 Peripheral neuropathy Mitochondrion Acidosis Myoclonic epilepsy Encephalomyopathy Mitochondrial DNA Mitochondrial encephalomyopathy
0 14
D is not co rrect. 20/o chose this.
• 15 Although colon cancer can arise sporadically, this is less likely in this case, given the patient's young age and positive family history.
Colorectal cancer Cancer Colon (anatomy)
0
16
E is not correct. 20/o ch ose this.
0
17
Examples of X-linked recessive conditions include hemophilia A and B, glucose-6-phosphate dehydrogenase deficiency, and Lesch-Nyhan syndrome .
. 18 Lesch-Nyhan syndrome Haemophilia Haemophilia A Glucose-6-phosphate dehydrogenase Glucose-6-phosphate dehydrogenase deficiency

• 19 X-linked recessive inheritance Sex linkage Recessive Dominance (genetics) Glucose 6-phosphate

• 20
• 21 Bottom Line:

a
Lock
s
Su spend
8
End Block
Item: 6 of 64 ~. I • M k <:] t> al ~· ~
QIO: 1662 .l. ar Previous Next lab 'Vfl1ues Notes Calculator

1 Bottom Line:
2 Familial adenomatous polyposis is an autosomal dominant condit ion caused by mutat ion of the APC gene on Sq, leading to increased incidence of
mult isystem m alignancies.
3
Familial adenomatous polyposis Gene Dominance (genetics) Mutation Autosome Adenomatous polyposis coli Malignancy Cancer
4
5
6 I ill ;fi 1!1 I•J for year:[ 2017 ..
FIRST AID FA CT S
.7
•8
FA11 p 370.2
•9
Polyposis syndromes
• 10
Familial adenomatous Autosomal dominant mutation of APC tumor suppressor gene on chromosome 5q. 2-hit hypothesis.
· 11 polyposis Thousands of polyps arise starting after puberty; pancolonic; always involves rectum. Prophylactic
• 12 colectomy or else 100% progress to CRC .
• 13 Gardner syndrome FAP +osseous and soft tissue tumors, congenital hypertrophy of retinal pigment epithelium,
• 14 impacted/supernumerary teeth .
• 15 Turcot syndrome FAP/Lynch syndrome + malignant CNS tumor (eg, medulloblastoma, glioma). Turcot = Turban.
• 16 Peutz-Jeghers Autosomal dominant syndrome featuring numerous hamartomas throughout CI tract, along with
• 17 syndrome hyperpigmentecl mouth, lips, hands, genitalia. Associated with t risk of breast and CI cancers (eg,
• 18 colorectal, stomach, small bowel, pancreatic).
• 19 Juvenile polyposis Autosomal dominant syndrome in children (typically< 5 years old) featuring numerous
syndrome hamartomatous polyps in the colon, stomach, small boweL Associated with t risk of CRC.
• 20
• 21

6
lock
s
Suspend
0
End Block
Item: 6 of 64 ~ 1 • M k -<:J 1>- Jil ~· !:';-~
QIO: 1662 ..L ar Pre vious Next Lab~lues Notes Calcula tor
& &
1
FA17 p 371 .2
2 Colorectal cancer
3 EPIDEMIOLOGY 1\lost patients are > 50 )Cars old.- 25% ha,e a
4 family history.
5 RISK FACTORS Adenomatous and serrated polyps, familial
6 cancer syndromes, lBO, tobacco use, diet of
0 7 processed meat with low fiber.
o8 PRESENTATION Rectosigmoid> ascending > descending. Right side bleeds; left side obstructs.
.9
Ascending- exophrtic mass, iron deficiency
anemia, weight loss.
• 10
Descending- infiltrating mass, partial
• 11 obstruction, colicky pain, hematochezia.
• 12 Rarelr. presents with S bovis (gallolyticus)
• 13 bacteremia.
0 14 DIAGNOSIS Iron deficiency anemia in males (especially> 50
• 15 years old) and postmenopausa l females raises
susp1c1on.
0
16
Screen low-risk patients s1<1rting <lt age 50 with
17
colonoscopy fJ; alternatives include Aexible
0

. 18 sigmoidoscopy, fecal occult blood testing


• 19 (FOBT), fecal immunochemical testing
• 20 (FIT), and CT colonographr. Patients with
a first-degree relati\'e who has colon cancer
• 21
• c h nulrl h., cr-rPPnPrl ui".l l"nl nnncl"nnu #\t "lnP

a
Lock
s
Su spend
8
End Bl ock
Item: 6 of 64 ~ 1 • M k -<:J 1>- Jil ~· !:';-~
QIO: 1662 ..L ar Pre vious Next Lab~lues Notes Calcula tor

1
&
FA17 p55.1 &

2
Modes of inheritance

3 Autosomal dominant Often due to defects in structural genes. 1\ Iany Often pleiotropic (multiple apparent!) unrelated
generations, both males and females are effects) and variably expressive (different
4
affected. between individuals). Family history crucial
5
to diagnosis. With one affected (heteroz)gous)
6 parent. on average, !h of children affected.
0 7
o8

.9
Autosomal recessive Often due to enzyme deficiencies. Usuall} seen Commonh· more se,·ere than dominant disorders;
0
• 10
in only I generation. patients often present in childhood.
• 11 f risk in consanguineous families.
• 12 \Vith 2 carrier (heterozygous) parents, on average:
• 13 V.. of children will be affected (homozygous),
0 14
!lz of children will be carriers, and \4 of ch ildren
will be neither affected nor carriers .
• 15
0
16 X-linked recessive Sons of heterozygous mothers have a 50% Common ly more severe in males. Females
0
17 chance of being affected. o ma le-to-nude u~ual l y must be homozygous to be affected.

. 18 transmission. Skips generations.


• 19
• 20
• 21

a
Lock
s
Su spend
8
End Bl ock
Item: 7 of 64 ~ 1 • M k -<:J 1>- Jil ~· !:';-~
QIO: 1217 ..L ar Pre v ious Next Lab~lues Notes Calcula t o r
& &
1
A mutation In phenylalanine tRNA synthet ase changes the proofreading hydrolytic site so that it cannot differentiat e between phenylalanine and
2 alanine.

3
Which of the following biochemica l results is the most likely outcome of this mutation?
4
5 :
A. Alanine will no longer be incorporated into growing amino acid chains
6
0 7 B. Alan1ne will not be inserted in place of phenyla lanine because of proofreading mechanisms

o8 c. Increased amounts of alanin e wi ll be inserted in place of phenylalanine


.9
D. Increased amounts of phenylalanine will be inserted in place of alanine
• 10
E. Phenylalanine will no longer be incorporated into growing amino acid chains
• 11
• 12
• 13
0 14
• 15
0
16
0
17
• 18
• 19
• 20
• 21

a
Lock
s
Suspend
8
End Bl ock
Item: 7 of 64 ~ 1 • M k -<:J 1>- Jil ~· !:';-~
QIO: 1217 ..L ar Pre v ious Next Lab~lues Notes Calcula t o r
& &
1 Th e co rrect a nswer i s c . 68% chose this.
2 A mischarged tRNA still reads the right codon but inserts the wrong amino acid. Therefore, increased amounts of alanine will be inserted in place of
phenylalanine.
3 Amino acrd Phenylalanine Transfer RNA Alanine Genetic code

4 A i s not correct. 30/o chose t his.

5 Alanine will still be rncorporated into growing amino acid chains. A mischarged tRNA still reads the right codon but inserts t he wrong amino acid.
Therefore, increased amounts of alanine will be inserted in place of phenylalanine.
6 Phenyrala re Amr o acid Transfer RNA Alanine Genetic code

7 B is not correct. 11% chose this.


·8 A mischarged tRNA still reads the right codon but inserts the wrong amino acid. Therefore, increased amounts of alanine will be inserted in place of
phenylalanine.
.9 Amino acid Phenylaoanine Transfer RNA Alanine Genetic code

• 10 0 is not correct. 1 3% chose this.


• 11 A mischarged tRNA still reads the right codon but inserts the wrong amino acid. Therefore, increased amounts of alanine will be inserted in place of
phenylalanine. Because there is no mutation in the alanine tRNA synthetase, Increased amounts of phenylalanine will not be inserted in place of alanine .
• 12 Amino acid Phenylalanine Transfer RNA Alanine Mutation Genetic code

• 13 E is not co rrect. solo chose t his.


• 14 In the situation described, phenylalanine will still be incorporated into the growing amino acid chains. A mischarged tRNA still reads the right codon but
Inserts the wrong amino acid. Both amino acids will be chosen randomly by the tR NA, so both will end up being incorporated . Therefore, increased
• 15 amounts of alanine will be inserted in place of phenylalanine .
Phenylalanine Transfer RNA Amino acid Alanine Genetic code
• 16
• 17
. 18 Bottom Lin e:

• 19 Each tRNA Is charged with an amino acid corresponding to its anticodon by the enzyme aminoacyl-tRNA synthetase. There is only one amlnoacyl-tRNA
synthetase for each amino acid. If a mutation renders an aminoacyl-tRNA unable to distinguish between two amino acids, the wrong amino acid will
• 20 often be added to the growing polypeptide chain. In this case, the phenyla lanine-specific tRNA synthetase cannot distinguish between phenyla lanine
and alanine, so it will insert both peptides into a growing chain. In other words, increased amounts of alanine will appear in the chain rather than solely
• 21 phenyla lanine .

a
Lock
s
Suspend
8
End Bl ock
Item: 7 of 64 ~ 1 • M k -<:J 1>- Jil ~· !:';-~
QIO: 1217 ..L ar Pre v ious Next Lab~lues Notes Calcula t o r
& &
1 FA17 p 40.1
2 tRNA
3 Structure 75-90 nucleotides, 2" structure, cloverleaf form, anticodon end is opposite 3' aminoacyl end. II
4 tRl As, both eukaryotic and prokaryotic, have CC at 3' end along with a high percentage of
5
chemically modified bases. The amino acid is covalently bound to the 3' end of the tRl\t\. CC.\
Can Carry .\ mino acids.
6
T-arm: contains the T'I'C (ribothymidine, pseudouridine, cytidine) sequence necessary for tRl\A-
7 ribosome binding. T-arm Tethers tR A molecule to ribosome.
·8 D-arm: contains dih~drouridine residues necessary for tRNA recognition by the correct aminoacyl-
.9 tR• A S\•nthetase. D-arm Detects the t R t\ b\ aminoacvl-tRN svnthetase.
~ ; • J

• 10
Acceptor stem: the 5'-CC. \ -3' is the amino acid acceptor site.
• 11
Charging Aminoacyl-tRl A synthetase (I per amino acid; "matchmaker"; uses ATP) scrutinizes amino acid
before and after it binds to tRl . If incorrect, bond is hydrolrzcd. The amino acid-tRJ bond
• 12
has energy fo r formation of peptide bond. A mischarged tRN reads usual codon but inserts
• 13
wrong amino acid.
• 14 Aminoacyl-tR A synthetase and binding of charged tR to the codon are responsible for
• 15 accuracy of am ino acid selection .
• 16 Structure Charging Pairing
(aminoacylation} (codon-anticodon}
• 17
Aminoacid , Amtnoaod,
. 18 0 0
Acceptorstem{0 H- ~ 3. ' . 3'
c ' •c 3'
• 19 c c
s· s· s·
• 20 Amonoacyl·tRNA
s nthetase
• 21

T-arm
0-arm ATP AMP+ PP

a
IF2
lonohatonn fartnrl
- s 8
Lock Suspend End Bl ock
Item: 7 of 64 ~ 1 • M k -<:J 1>- Jil ~· !:';-~
QIO: 1217 ..L ar Pre v ious Next Lab~lues Notes Calcula t o r
& &
1
FA17p 41.1
2
Prot ein synthesis
3
Initiation Initiated b) GTP hydrolysis; in itiation factors Eukaryotcs: 40S + 60S ._.. 80S (Even).
4
(eukaryotic IF's) help assemble the -+0 PrOkaryotes: 30S + 50S ..... 70S (Odd).
5
ribosomal subunit with the initiator tR A Synthesis occurs from 1 -terminus to
6 and are released when the m R A and the C-terminus.
7 ribosomal 60S subunit assemble \\ ith the
AT P- tR 'A \ cli\·ation (charging).
·8 complex.
GTP- tRNA Gripping and Going places
.9 Elongation 1. Aminoacyl-tR 'A binds to A site (except for (translocation).
• 10
initiator methionine)
2. rR 'A ("ribozyme") catalyzes peptide bond Think of "going \Pi':":
• 11 i\ site = incoming .\minoacyl-tR A.
formation, transfers grO\\ ing polypeptide to
• 12 amino acid in A site P site = accommodates growing Peptide.
• 13 3. Ribosome advances 3 nuclcotides tO\\'ard 3' E site = holds Empty tR 'A as it Exits.

~
• 14 end of mR1 A, moving pcptidyl tR A to P
Eukaryotic
• 15
site (translocation) nl:>osorn:j vv,
• 16 Termination Stop codon is recognized by release factor, 3'
and completed polypeptide is released from p
• 17 s· E
ribosome .
. 18
• 19
• 20
• 21

a
Lock
s
Suspend
8
End Bl ock
Item: 8 of 64 ~ 1 • M k -<:J 1>- Jil ~· !:';-~
QIO: 3936 ..L ar Pre v ious Next Lab~lues Notes Calcula t o r
& &
1
On fundus examination, a 34-year-old man single man with progressive, bilateral, central visual loss occurring over the past 4 weeks was found to
2 have retinal telangiectasias and increased vascular tortuosity. His mother and older brother became blind at age 35 and 40, respectively. Both have
cardiac arrhythmias and multiple sclerosis-like symptoms. Further testing reveals degeneration of the retinal ganglion and optic nerve in all three
3 affected family members. However, these results, along with previously mentioned features, were not found in the older brother's son and daughter who
were 30 and 28 years old.
4
5
What percentage of this man's four children will be affected by the same disease, assuming the disease has 100% penetrance in this family?
6
:
7 A. 0%
·8
B. 100%
.9
• 10 c. 25%

• 11 o. 50%
• 12
E. 75%
• 13
• 14
• 15
• 16
• 17
• 18
• 19
• 20
• 21

a
Lock
s
Suspend
8
End Bl ock
Item: 8 of 64 ~ 1 • M k -<:J 1>- Jil ~· !:';-~
QIO: 3936 ..L ar Prev ious Next Lab~lues Notes Calculat o r

& &
1
Th e co rrect a nsw er i s A. 4 60/o ch ose this.
2 In addition to this patient's strong family history of acquired visual loss, the subacute progression of his blindness and findings on further testing suggest
3 that he may suffer from a condition known as Leber hereditary optic neuropathy (LHON). LHON is a mitochondrially inherited degeneration of retinal
ganglion cells and their axons, that leads to painless loss of central vision, known as central scotoma. Although visual loss may occur in both eyes
4 simultaneously, as in this patient, it is also common for the disease to manifest in one eye first, followed by the other eye in a few weeks. Because the
disease arises from mutations in mitochondrial DNA, it does not follow a mendelian pattern of inheritance, nor does it always have 100% penetrance. The
5 disease Is transmitted through the mother and not the father because only the egg contributes mitochondria to the fertilized embryo. Therefore, the
older brother's children are unaffected. LHON affects both men and women, although predominantly it manifests in young adult men, typically in the ear1y
6
30s, although symptoms are known to occur at any age. Patients with LHON may also experience extraocular symptoms, such as movement disorders,
7 cardiac arrhythmias, and multiple sclerosis-like symptoms.
l"itochondrion Scotoma Mitochondrial DNA Penetrance Human mitochondrial genetics Leb.,r's hereditary optic neuropathy Embryo Retinal ganglion eel
8
Optic neo opathy Peripheral neuropathy Axon Ganglion cardiac arrhythmia Visual impairment DNA Mendelian inheritance Mutation
.9
B i s n o t correct. 9 0/o chose this .
• 10 This represents a case of autosomal dominant transmission of a trait by a person who is homozygous for a specific allele. An example would be autosomal
dominant polycystic kidney disease .
• 11
Dominance (genetics) Allele Polycystic kidney disease Homozygous Autosomal dominant polycystic kidney disease Autosome Kidney Kidney disease Zygosity
• 12
C is not co rrect. 1 30/o chose this •
• 13 For autosomal recessive traits, 25% of offspring from two carrier parents are affected. An example is cystic fibrosis.
• 14 Cystic fibrosis Autosomal recessive Recessive Autosome Dominance (genetics) Fibrosis

• 15 D is no t co rrect . 260/o ch ose this .


Mothers with X-linked dominant traits will pass those traits to SO% of offspring, as Is the case in X-linked hypophosphat emia (vitamin D-reslstant rickets)
• 16 and about 85% of Alport syndrome cases. This pattern is also seen when an autosomal dominant gene is passed on from a parent who is heterozygous
for the gene .
• 17
Alport syndrome Rickets Dominance (genetics) Gene X-linked dominant inheritance Heterozygous X-linked hypophosphatemia Sex linkage Autosome
• 18 Hypophosphatemia
• 19 E i s n ot correct. 6 0/o ch ose this.
• 20 This Is Impossible, given the amount of information provided in the question stem .

• 21

a
Lock
s
Suspend
8
End Block
Item: 8 of 64 ~. I • M k <:] t> al ~· ~
QIO: 3936 .l. ar Previous Next Lab 'Vfl1ues Notes Calculator

1
Bottom Line:
2
Leber hereditary optic neuropathy (LHON) is a disease of painless, subacute visual loss due to mutations in mitochondrial DNA, which is transmitted
3 exclusively through the mother. A strong family history of adult-onset blindness as well as m anifesta tions of extraocular symptoms such as movem ent
disorders, cardiac arrhythmias, and multiple sclerosis-like symptoms should raise suspicion for LHON .
4 Mitochondrial DNA Leber' s hereditary optic neuropathy Optic neuropathy Peripheral neuropathy Mitochondrion Cardiac arrhythmia DNA Mutation Visual impairment
Family history (medicine)
5
6
7

8
Iiii I;fi IJ I•J for year:[ 2017 ..
FIRST AID FACTS

•9
• 10 FA17 p 55.1
Modes of inheritance
· 11
• 12 Autosomal dominant Often due to defects in structural genes. Many Often pleiotropic (multiple apparently unrelated
generations, both males and females are effects) and variably expressive (different
• 13
affected . between individuals). Family history crucial
• 14
to diagnosis. With one affected (heterozygous)
• 15 parent, on average, Y2 of children affected .
• 16
• 17

• 18
Autosomal recessive Often due to enzyme deficiencies. Usually seen Commonly more severe than dominant disorders;
• 19 in only I generation. patients often present in childhood.
• 20 t risk in consanguineous fam ilies .
• 21 '"' ith 2 ca rrier (heterozygous) parents, on average:

6
lock
s
Suspend
0
End Block
Item: 9 of 64
QIO: 4869
~
..L
1 • M ar k -<:J
Previous
I>
Next Lab
fJ
lues
£!1}>'

Notes
!!":-~
Cal culator

& &
1
Frameshlft mutations are responsible for a variety of severe genetic diseases.
2
3 Which sequence represents a frameshift mutation of the following sequence:
4 TTT GTC GAG ACT GAG
5
:
6 A. TTT ACT GTC GAG ACT GAG

7
B. TTT GAC GAG ACT GAG
8
C. TTT GAG ACT GAG
.9
• 10 D. TTT GTG AGA CTG AG

• 11
• 12
• 13
• 14
• 15
• 16
• 17
• 18
• 19
• 20
• 21

a
Lock
s
Suspend
8
End Block
Item: 9 of 64 ~. I • M k <:] t> al ~· ~
QIO: 4869 .l. ar Previous Next lab 'Vfl1ues Notes Calculator

2
The correct answer is D. 72% chose this.
3
A frameshift mutation is caused by an insertion or deletion of a number of nucleotides that is not evenly divisible by three from a DN A sequence, resulting
4 in a disruption of the rea ding frame and a complet ely different translation from the original. The earlier the insertion or deletion, the more altered is the
protein. In this case, the third nucleotide in the second codon has been delet ed.
5 Frameshift mutation Nucleic acid sequence Nucleotide Protein Mutation Translational frameshift Genetic code Reading frame DNA
6 A is not correct. 11% chose this.
7 Three nucleotides have been inserted; thus, a complet e shift in the rea ding frame will not result.
Nucleotide Reading frame
8
B is not correct. 7% chose this.
9
In this example, a single nucleotide is substituted (the fourth T is replaced by an A). Although this may alter the protein, it will not result in a complet e
0 10 shift in the rea ding frame.
Protein Nucleotide Reading frame
o ll
0
12
c is not correct. 10% chose this.
Three nucleotides have been delet ed; thus, a complet e shift in the rea ding frame will not result.
0
13 Nucleotide Reading frame

0 14
0 15
Bottom Line:
0
16 A frameshift mutation is due to an insertion or deletion of nucleotides that is not evenly divisible by three, thereby disrupting the rea ding frame and
0 17 resulting in a complet ely different translation from the original.
Frameshift mutation Nucleotide Mutation Translational frameshift Reading frame
0
18
0 19
0 20 Iiii I;fi IJ I•J for year:[ 2017 ..
FI RST AID FAC T S
0
21

6
lock
s
Suspend
0
End Block
Item: 9 of 64 ~ 1 • M k -<:J 1>- Jil ~· !:';-~
QIO: 4869 ..L ar Previous Next Lab~lues Notes Calculator
& &
1 FA17 p 36. 1
2
Mutations in DNA Se,·erity of damage: silent << missense < nonsense < frameshift.
3 For point (silent, missense, and nonsense) mutations:
4 • Transition- purine to purine (eg, to C) or pyrimidine to pyrimidine (eg, C toT).
5 Transversion-purine to pyrimidine (cg, A toT) or pyrimidine to purine (eg, C to C).
6 Silent 'ucleotide substitution but codes for same
7 (synonymous) amino acid; often base change
in 3rd position of codon (tR ' \\obble).
8
Missense 'ucleotide substitution resulting in changed Sickle cell disease {substitution of glutamic acid
9
amino acid (called conservative if ne'' amino with valine).
• 10
acid is similar in chemical structure).
• 11
Nonsense i':ucleotide substitution resulting in carlr \ top Stop I he nonsense!
• 12 codon (UAG, UAA, UGA). Usually results in
• 13 nonfunctional protein .
• 14 Frameshift Deletion or insertion of a number of nucleotides Duchenne muscular dystrophy, Tay-Sachs
• 15 not divisible by 3, resulting in misreading of disease.
• 16 all nucleotidcs downstream. Protein may be
shorter or longer, and its function may be
• 17
disrupted or altered.
• 18
Splice site Mutation at a splice site .... retained intron in Rare cause of cancers, dementia, epilepsy, some
• 19
the mRl\IA .... protein with impaired or altered I ypes of ~-thalassemia .
• 20 function .
• 21

a
Lock
s
Suspend
8
End Block
Item: 10 of 64 ~ 1 • M k -<:J 1>- Jil ~· !:';-~
QIO: 2374 ..L ar Pre v ious Next Lab~lues Notes Calcula t o r
&
1 The human leukocyte antigen complex is a 4-m egabase region on chromosome 6 that is densely packed with expressed genes that lead to proteins
2 critical for Immunologic specificity and thus aut oimmune disea ses. Pedigree Is shown in the image: Note that the two DR alleles possessed by the
proband's grandparents are shown above their pedigree symbols.
3 DR5/DR7 DR4/0R8 DR8/DR8 DR3/DR7
4
5
6
7
8
9
• 10
• 11

• 12
/
• 13
• 14
What combination of alleles that could be inherit ed by the proband would confer the greatest risk to the patient of cont racting t ype 1 diabetes?
• 15
:
• 16 A. DR3/DR3
• 17
B. DR3/DR4
. 18
c. DR5/DR3
• 19
• 20 o. OR7/DR3

• 21 E. DRS/DRS

a
Lock
s
Suspend
8
End Bl ock
Item: 10of64 ~. , . M k <:] t> al ~· ~
QIO: 2374 .l. ar Previous Next Lab 'Vfll ues Notes Calculator

1
The co rrect a nswer is B. 69% cho se this.
2
Human leukocyte antigens ( HLAs) DR4 and DR3 molecules are known to confer grea t er -than-average susceptibility to type 1 diabet es. In fact,
3 epidemiologic studies suggest that a carrier of both HLA -DR4 and HLA-DR3 is so times more susceptible to type 1 diabet es than a noncarrier.
Diabetes mellitus type 1 Antigen Human leukocyte antigen HLA-DR4 Diabetes mellitus White blood cell HLA-DR3
4
A is no t co rrect. 8% cho se this.
5
HLA -DR3 is known to confer a grea t er -than-average susceptibility to type 1 diabet es; however; it would not be possible for the patient to inherit two HLA-
6 DR3 alleles, as only one grandparent has it.
Diabetes mellitus type 1 HLA-DR3 Diabetes mellitus
7
c is no t co rrect. 11% cho se this.
8
This answer choice does contain allele HLA -DR3 which confers a grea t er -than -average susceptibility to type 1 diabet es. The second allele, HLA-DRS, is
9 associat ed with pernicious anemia and Hashimoto thyroiditis and not type 1 diabet es.
Hashimoto' s thyroiditis Pernicious anemia HLA-DRS Diabetes mellitus type 1 HLA-DR Allele Anemia Diabetes mellitus HLA-DR3 Thyroiditis
10
D is no t co rrect. 5 % cho se this.
· 11
This answer choice only has 1 allele, HLA -DR3, which confers a grea t er susceptibility to type 1 diabet es. The allele, HLA-DR 7, is not associat ed with type 1
• 12 diabet es but is associat ed with nephrotic syndrome.
Diabetes mellitus type 1 Nephrotic syndrome Allele HLA-DR7 Diabetes mellitus HLA-DR3
• 13
• 14 E is no t co rrect. 7 % cho se this •
This answer choice is not correct because HLA -DR8 is weakly associat ed with primary biliary cirrhosis and not type 1 diabet es.
• 15 Primary biliary cirrhosis Diabetes mellitus type 1 Cirrhosis Diabetes mellitus Bile

• 16
• 17
Botto m Li ne:
• 18 HLA -DR3 and -DR4 are associat ed with type 1 diabet es. Remember HLA -DR4 ( Diabet es and Rheumatoid arthritis) .
• 19 Diabetes mellitus type 1 Human leukocyte antigen HLA-DR3 Arthritis Diabetes mellitus

• 20
• 21
i!!ii.IJ.S'.ii.i ,. I

6
lock
s
Suspend
0
End Block
Item: 10 of 64 ~ 1 • M k -<:J 1>- Jil ~· !:';-~
QIO: 2374 ..L ar Pre v ious Next Lab~lues Notes Calcula t o r
& &
1 FA17 p 96.2
2 HLA subtypes associated with diseases
3 A3 Hemochromatosis
4 BS Addison disease, myasthenia gra' is, Craves
5 disease
6 B27 Psoriatic arthritis, \ nkylosing spondylitis, P \JR. Also known as seronegati,·c arthropathies.
7 IBD-associated arthritis, Reactive arthritis
8 DQ2/ DQ8 Celiac disease I ate (S) too (2) much gluten at Dairy Q ueen.
9 DR2 Multiple sclerosis, hay fever, SLE, lultiple hay pastures have dirt.
10 Coodpashue syndrome
• 11 DR3 Diabetes mellitus type I, SLE, Craves disease, 2-3, S-L-E
• 12 Hashimoto thyroiditis, Addison disease
• 13 DR4 Rheumatoid arthritis, diabetes mellitus type I, There are -f walls in a "rheum" (room).
• 14
ddison disease
• 15 DRS Pernicious anemia - vitamin B12 deficiency,
Hashimoto thyroiditis
• 16
• 17
FA17 p 328.1
• 18
Hypothyroidism
• 19
Hashimoto thyroiditis lost common cause of hypothyroidism in iodine-sufficient regions; an autoimmune disorder with
• 20
antithyroid peroxidase (antimicrosomal) and antithyroglobulin antibodies. Associated ,,·ith t risk
• 21 of non-Hodgkin lymphoma (typicallv of B-cell origin).

a
Lock
s
Suspend
8
End Bl ock
Item: 10 of 64 ~ 1 • M k -<:J 1>- Jil ~· !:';-~
QIO: 2374 ..L ar Pre v ious Next Lab~lues Notes Calcula t o r
& &
1
FA17p337.1
2
Type 1 vs type 2 diabetes mellitus
3
Variable Type 1 Type2
4
1 DEFECT Autoimmune destruction of ~ cells (cg, due to f resistance to insulin, progressi,·e pancreatic
5
glutamic acid decarbox) lase antibodies) ~ -cel l failure
6
INSULIN NECESSARY IN TREATMENT Always Sometimes
7
AGE (EXCEPTIONS COMMONLY < 30 \ "T >40rr
8 ' '
OCCUR)
9
ASSOCIATION WITH OBESITY No Yes
10
GENETIC PREDISPOSITION Relativelr weak (50% concordance in identical Relatively strong (90% concordance in identical
• 11 t\\ ins), polygenic I" ins), polygenic
• 12
ASSOCIATION WI TH HLASYSTEM Yes (HLA-DR3 and -OR4) 'o
• 13
GLUCOSE INTOLERANCE Severe Iild to moderate
• 14
INSULINSENSITIVITY lligh Low
• 15
KETOACIDOSIS Common Rare
• 16
~-CELL NUMBERS IN THEISLETS l Variable (with amyloid deposits)
• 17
SERUMINSULINLEVEL l Variable
• 18
• 19
CLASSIC SYMPTOMSOFPOLYURIA, Common Sometimes
POLYDIPSIA, POLYPHAGIA, WEIGHT
• 20 LOSS
• 21 HISTOLOGY Islet leukocvtic infiltrate Islet am} loid polypeptide (lAPP) deposits
• '
a
Lock
s
Suspend
8
End Bl ock
Item: 11 of 64 ~ 1 • M k -<:J 1>- Jil ~· !:';-~
QIO: 3692 ..L ar Pre v ious Next Lab~lues Notes Calcula t o r
& &
1
A 54-year-old man was recently diagnosed with m edullary thyroid cancer. In his last three hospitalizations, he was admitted for episodes of
2 uncontrollable hypertensive crisis that were associated with severe headache, sweating, and palpitations. The patient subsequently had an
adrenalectomy that corrected his problem .
3
4 What Is the mode of inheritance of this patient's condition?
5
:
6 A. Autosomal dominant inheritance
7
B. Autosoma l recessive inheritance
8
c. Mitochondria l inheritance
9
10 D. X-llnked dominant inheritance

• 11 E. X-llnked recessive inheritance


• 12
• 13
• 14
• 15
• 16
• 17
• 18
• 19
• 20
• 21

a
Lock
s
Suspend
8
End Bl ock
Item: 11 of 64 ~. I • M k <:] t> al ~· ~
QIO: 3692 .l. ar Previous Next Lab 'Vfl1ues Notes Calculator

1 •
The co rrect a nswer is A. 74% cho se this.
2 The patient demonstrat es symptoms of multiple endocrine neoplasia (MEN) IIA/IIB, as both pheochromocytoma and medullary thyroid cancer occur in
about 5 0 % and 90%, respectively, of patients with MEN IIA/IIB. MEN IIA (Sipple syndrome) and MEN liB (now also known as MEN III) involve mutations
3 in the ret proto-oncogene, and are passed on in an autosomal dominant manner.
Pheochromocytoma Multiple endocrine neoplasia type 2 Medullary thyroid cancer Thyroid cancer Dominance (genetics) Proto-oncogene Multiple endocrine neoplasia
4
Endocrine system Neoplasm Thyroid Autosome Cancer Mutation Oncogene
5
B is no t co rrect. 18% cho se this.
6
Multiple endocrine neoplasia (MEN) syndromes are all inherited in an autosomal dominant manner. Autosomal recessive disea ses include cystic fibrosis,
7 thalassemias, sickle cell anemia, and others.
Cystic fibrosis Sickle-cell disease Autosomal recessive Dominance (genetics) Anemia Multiple endocrine neoplasia Endocrine system Autosome Neoplasm Fibrosis
8
Recessive
9
c is no t co rrect. 2% cho se this.
10 Multiple endocrine neoplasia (MEN) syndromes are all inherited in an autosomal dominant manner. Mitochondrial inheritance disea ses include Leber
hereditary optic neuropathy, myoclonic epilepsy, ragged-red fiber disea se, and others.
11
Epilepsy Dominance (genetics) Multiple endocrine neoplasia Peripheral neuropathy Neoplasm Autosome Leber' s hereditary optic neuropathy Myoclonus
• 12 Endocrine system Mitochondrion Optic neuropathy Mitochondrial DNA Myoclonic epilepsy
• 13 D is no t co rrect. 2% cho se this.
• 14 Multiple endocrine neoplasia (MEN) syndromes are all inherited in an autosomal dominant manner. The rare X-linked dominant disea ses include certain
forms of Charcot -Marie-Tooth disea se, and fragile X syndrome .
• 15 Fragile X syndrome Charcot-Marie-Tooth disease Dominance (genetics) Multiple endocrine neoplasia Autosome X-linked dominant inheritance Endocrine system
• 16 Sex linkage Neoplasm

• 17 E is no t co rrect. 4 % cho se this.


• 18 Multiple endocrine neoplasia (MEN) syndromes are all inherited in an autosomal dominant manner. X-linked recessive disea ses Duchenne's muscular
dystrophy, hemophilia A and B, and others .
• 19 Sex linkage Haemophilia Haemophilia A Dominance (genetics) Duchenne muscular dystrophy X-linked recessive inheritance Muscular dystrophy Autosome

• 20 Multiple endocrine neoplasia Recessive Neoplasm Endocrine system

• 21

6
lock
s
Suspend
0
End Block
Item: 11 of 64 ~. I • M k <:] t> al ~· ~
QIO: 3692 .l. ar Previous Next lab 'Vfl1 ues Notes Calculator

1
Bottom Line:
2
MEN IIA/IIB is associated with pheochromocytoma and medullary thyroid cancer; and is passed on via autosomal dominant inheritance.
3 Pheochromocytoma Medullary thyroid cancer Dominance (genetics) Thyroid cancer Thyroid Autosome Cancer

4
5
6 Iiii I;fi IJ I•J for year:[ 2017 ..
FI RST AID FAC T S
7

8 FA17p339.1
9 Multiple endocrine All MEN syndromes have au tosomal dominant inhe ritance.
10 neoplasias "All MEN are dominant" (or so they think).
11 SUBTYPE CHARACTERISTICS COMMENTS

• 12 MEN 1 P ituitary tumors (prolactin or GI-l)


• 13 Pancreatic endocrine tumors- Zollinger-
Pituitary
Ellison syndrome, insulinornas, VIPomas,
• 14
glucagonornas (rare) Pancreas
• 15
Parathyroid adenomas
• 16 Associated with mutation of MENl (men in, a
• 17 tumor suppressor, chromosome 11)
• 18 MEN2A Parathyroid hyperplasia
Parathyroids
• 19 [edullary thyroid carcinoma-neoplasm of
parafollicular or C cells; secretes calcitonin;
• 20
prophylactic thyroidec tomy required
• 21
P heochromocvtoma (secretes catecholamines) Thvrniti

6
lock
s
Suspend
0
End Block
Item: 11 of 64 ~ 1 • M k -<:J 1>- Jil ~· !:';-~
QIO: 3692 ..L ar Pre v ious Next Lab~lues Notes Calcula t o r

1
&
FA17 p 326.1 &

2 Pheochromocytoma

3 ETIOLOGY lost common tumor of the adrenal medulla in Rule of IO's:


4
adults · . Derived from chromaffin cells (arise 10% malignant
from neural crest). 10% bilateral
5
:\lay be associated with germline mutations (eg, 10% extra-adrenal (eg, bladder" all, organ of
6 1 F-1, VHL, RET [\I E 2 . 28]). Zuckerkandl)
7 10% calcify
8 10% kids
9
10
11 SYMPTOMS !\lost tumors secrete epinephrine, Episodic h~ peradrenergic symptoms (5 P's):
• 12 norepinephrine, and dopamine, which can Pressure (t BP)
• 13
cause episodic hypertension. Pain (headache)
Symptoms occur in "spells" -relapse and rc111il. Perspiration
• 14
Palpitations (tachycardia)
• 15 Pallor
• 16 FINDINGS t catecholamines and metancphrincs in urine
• 17 and plasma.
. 18 TREATMENT Irreversible a-antagonists (eg, Phenox} benzamine (16 letters) is given for
• 19 phenoxybenzamine) followed by ~-blockers pheochromocytoma (also 16 letters).
• 20 prior to tumor resection. a-blockade must be
• 21
achieved before giving ~-blockers to a\oid a

a
Lock
s
Suspend
8
End Bl ock
Item: 11 of 64 ~ 1 • M k -<:J 1>- Jil ~· !:';-~
QIO: 3692 ..L ar Pre v ious Next Lab~lues Notes Calcula t o r
& &
1
FA17 p55.1
2 Modes of inheritance
3 Autosomal dominant Often due to defects in structural genes. tvlany Often pleiotropic (multiple apparent!) unrelated
4 generations, both males and females are effects) and variably expressive (different
5 affected. between individuals). Family history crucial
to diagnosis. With one affected (hetero.t)gous)
6
parent. on average, !t2 of children affected.
7
8
9
10 Autosomal recessive Often due to enzyme deficiencies. Usuall) seen Commonh· •
more se,·ere than dominant disorders;
in onlr I generation. patients often present in ch ildhood.
11
f risk in consanguineous families .
• 12
With 2 carrier (heterozygous) parents, on average:
• 13 V.. of children will be affected (homozygous),
• 14 Yz of children wiiJ be carriers, and \4 of ch ildren
• 15 will be neither affected nor carriers .
• 16
X-linked recessive Sons of heterozygous mothers have a 50% Common ly more severe in males. Females
• 17
chance of being affected. o ma le-to-male usual ly must be homozygous to be affected.
. 18 transmission. Skips generations.
• 19
• 20
• 21

a
Lock
s
Suspend
8
End Bl ock
Item: 12 of 64 ~ 1 • M k -<:J 1>- Jil ~· !:';-~
QIO: 3698 ..L ar Pre v ious Next Lab~lues Notes Calcula t o r
& &
1
A 6-year-old Ashkenazi Jewish boy is brought to the emergency department (ED) with severe pain in his right hip. His mother reports that the pain
2 arose suddenly as they were walking home from the park, and the boy has not been able to walk since it began. The boy has been seen in the ED
previously for episodes of "bone pain," but previous tests and examinations have been unremarkable. One year ago, he underwent emergency
3 surgery after his spleen was ruptured during a backyard football game. On examination, an enlarged liver is palpat ed. x-rays of the lower extremities reveal
a fracture at the right femoral head.
4
5
What Is the most likely diagnosis?
6
:
7 A. Child abuse
8
B. Gaucher's disease
9
10 c. Hurler's syndrome

11 o. Nlemann-Pick disease
• 12
E. Tay-Sachs disease
• 13
• 14
• 15
• 16
• 17
• 18
• 19
• 20
• 21

a
Lock
s
Suspend
8
End Bl ock
Item: 12of64 ~. , . M k <:] t> al ~· ~
QIO: 3698 .l. ar Previous Next lab 'Vfl1 ues Notes Calculator

1
The co rrect a nswer is B. 52 % cho se this.
2 This boy most likely has Gaucher's disea se, the most common of the lysosomal storage disea ses. It is an autosomal recessive disea se caused by deficiency
in j3 glucocerebrosidase; it is most prevalent in Ashkenazi Jews. Findings associat ed with Gaucher's disea se are hepatosplenomegaly, aseptic necrosis of
3
the femur; and bone crises. The boy's history is consistent with bone crises and splenomegaly resulting in rupture. A fractured femoral hea d would be a
4 very rare finding without the presence of an underlying disorder. Patients with Gaucher's disea se experience aseptic necrosis of the femoral hea d and are
thus predisposed to such an injury. Another characteristic finding is " Gaucher's cells," macrophages that look like crumpled tissue paper.
5 Gaucher' s disease Splenomegaly Hepatosplenomegaly Ashkenazi Jews Femur Glucocerebrosidase Autosomal recessive lysosomal storage disease

6 Dominance (genetics) lysosome Macrophage Autosome Recessive Necrosis Bone

7 A is no t co rrect. 5 % cho se this.


Child abuse must always be considered when a child has a history significant for multiple visits to the ED. In this case, child abuse is made less likely by
8
the specific findings, such as an incidental fracture of the femoral hea d. Furthermore, you are told this patient is of Ashkenazi Jewish descent, indicating
9 higher likelihood of rare autosomal recessive conditions. Ultimat ely, Gaucher's disea se is the more likely diagnosis given the information in the question
stem .
10 Gaucher' s disease Autosomal recessive Ashkenazi Jews Dominance (genetics) Jews Autosome Bone fracture Judaism

11 c is no t co rrect. 9 % cho se this.


12 Hurler's syndrome is an autosomal recessive disea se caused by deficient a- L-iduronidase. This disea se is associat ed with developmental delay, gargoylism,
airway obstruction, and corneal clouding .
• 13 Hurler syndrome Autosomal recessive Recessive Specific developmental disorder Autosome Dominance (genetics)
• 14 D is no t co rrect. 19 % cho se this.
• 15 Niemann- Pick disea se is an autosomal recessive disea se caused by a defect in sphingomyelinase. It presents in infancy with hepatosplenomegaly (large
belly) with mental ret ardation ; half of patients will have cherry-red maculae and "foamy" macrophages, and will die by age 3 years .
• 16 Hepatosplenomegaly Autosomal recessive Niemann-Pick disease Macrophage Autosome Intellectual disability Dominance (genetics) Recessive
• 17 Sphingomyelin phosphodiesterase

• 18 E is no t co rrect. 15 % cho se this •


• 19 Tay-Sa chs disea se is an autosomal recessive disorder associat ed with deficient hexosaminidase A. It presents in infancy with progressive
neurodegeneration, cherry-red maculae, hyperacusis, and hypotonia. Dea th commonly occurs by age 3.
• 20 Tay-Sachs disease Hypotonia Hyperacusis Dominance (genetics) Autosomal recessive Neurodegeneration Autosome

• 21

6
lock
s
Suspend
0
End Block
Item: 12of64 ~. , . M k <:] t> al ~· ~
QIO: 3698 .l. ar Previous Next lab 'Vfl1 ues Notes Calculator

1
Bottom Line:
2
Gaucher's disease is the most common lysosomal storage disease. Common findings include hepatosplenomegaly, aseptic necrosis of the femur; bone
3 crises, and Gaucher's cells.
lysosomal storage disease Gaucher's disease Hepatosplenomegaly Femur lysosome Necrosis Bone
4
5
6
I iii I;fi 1!1 I•J for year:[ 2017 "
7 FI RST AID FAC TS

8
9 FA17p84.1

10 Lysosomal storage Each is caused by a deficiency in one of the many lysosomal enzymes. Results in an accumulation
11
diseases of abnormal metabolic products.
12 DISEASE FINDINGS DEFICIENT ENZYME ACCUMULATED SUBSTRATE INHERITANCE

• 13 Sphingolipidoses
• 14 Tay-Sachs disease Progressive neurodegeneration, 0 HeXosaminidase A G\11 2 ganglioside AR
• 15 developmental delay, "cherry-red" ("TAy-SaX")
spot on macula fJ, lysosomes with
• 16
onion skin, no hepatosplenomegaly
• 17
(vs liemann-Pick).
• 18
• 19
Fabry disease Early: Triad of episod ic peripheral @ a-galactosidase A Ceramide XR
• 20 neuropathy, angiokeratomas rn, trihexoside
• 21 hypohidrosis. Late: progressive renal

6
lock
s
Suspend
0
End Block
Item: 13 of 64 ~ 1 • M k -<:J 1>- Jil ~· !:';-~
QIO: 3946 ..L ar Pre v ious Next Lab~lues Notes Calcula t o r
& &
1
A 42-year-old woman seeks the advice of her physician because her memory has become progressively worse, she has writhing movements she
2 cannot control, and her friends have told her that she has "changed," particularly that she has become depressed, anxious, and at times aggressive.

3
What Is the most likely underlying cause of this patient's symptoms?
4
5 :
A. A long run of CTG on chromosome 13 in the anti-sense DNA of the gene
6
7 B. A polyg lutamine tract in a cellular protein, encoded by CAG sequences on chromosome 4

8 c. A repetitive sequence of CGG on a chromosome that may be deactivated by Barr-body formation


9
D. Expansion of continuous CTG sequences on chromosome 19, reduci ng the function of the encoded protein kinase
10
E. Repetition of GAA on chromosome 9, leading to dysfunction of Iron removal around mitochondria
11
12
• 13
• 14
• 15
• 16
• 17
• 18
• 19
• 20
• 21

a
Lock
s
Suspend
8
End Bl ock
Item: 13 of 64 ~ 1 • M k -<:J 1>- Jil ~· !:';-~
QIO: 3946 ..L ar Previous Next Lab~lues Notes Calculat or
& &
1 Th e co rrect a nsw er i s B. 730/o chose this.
2 Huntington's disease is an autosomal dominant neurodegenerative disorder In which patients progressively develop uncoordinated movements and chorea
as well as psychiatric symptoms. It is caused by a CAG trinucleotide repeat In the Huntingtin gene on the short arm of chromosome 4. Classically,
3 offspring of patients with diseases involving anticipation present with symptoms at an earlier age than the previous generation as a result of an expansion
of the CAG repeats. The symptoms in the offspring are usually more severe.
4
Huntington s d sease Neurodegeneration Dominance (genetics) Gene Chorea Huntingtin Chromosome 4 (human) Autosome Trinucleotide repeat disorder
5 Chromosome
6 A i s not correct. 6 % chose t his.
7 Spinocerebellar ataxia can arise due to a variety of mutations, one of them being a CTG repeat on chromosome 13 (SCA8 disease). Interestingly, the
expansion affects a non-protein coding reg ion in the anti-sense DNA strand. The patient experiences worsening hori zontal nystagmus but none of the
8 other symptoms described above.
Nystagmu• Sponocerebe ar ataxia Ataxia Chromosome Chromosome 13 1human1 DNA Mutation
9
10 C i s not co rrect. 7 % chose this.
Fragile X syndrome is an X-linked dominant d isease caused by abnormal expansion of the CGG trinucleotide repeat. Individuals with this disease have
11 stereotypical elongated facies with protruding ears, low muscle tone, and macro-orchidism. Patients also have varying degrees of mental retardation.
Although both sexes are affected, males usually suffer a more severe disease.
12
Fragile X syndrome X-linked dominant inheritance Intellectual disability Sex linkage Trinucleotide repeat disorder Hypotonia Muscle tone
13
0 is not co rrect. 90/o chose this •
• 14 Myotonic dystrophy is an autosomal dominant, multisystem disease caused by abnormal expansion of the trinucleotide repea t CTG. Symptoms Include
• 15 progressive proximal lower extremity wea kness, fatigue, and muscle pain secondary to muscle wasting. other extramuscular m anifestations Include
cataracts and arrhythmias.
• 16 Myotonic dystrophy Dominance (genetics) Autosome Trinucleotide repeat disorder Cataract Muscle atrophy Cardiac arrhythmia Fatigue (medical)
Anatomical terms of location Myalgia Muscle
• 17
. 18 E is not co rrect. s oto ch ose this .
Frledrelch's ataxia is an autosomal recessive disease caused by abnormal expansion of the trinucleotide repeat GAA, rendering the frataxin protein
• 19 dysfunctional. Frataxin is believed to clear the peri-mitochondrial cytosol of Iron radicals, thereby preventing oxidative damage. Without this protection,
• 20 the disease causes progressive degeneration of the neural tissue in the spinal cord, leading to gait disturbance and speech difficulty.
Fr'edreich s ataxia Frataxin Autosomal recessive Ataxia Gait abnormality Dominance (genetics) Protein Cytosol Trinucleotide repeat disorder Autosome
• 21 Spinal cord Gait Nervous tissue Oxidative stress Recessive

a
Lock
s
Suspend
8
End Block
Item: 13of64 ~. , . M k <:] t> al ~· ~
QIO: 3946 .l. ar Previous Next lab 'Vfl1 ues Notes Calculator
.. .. . ..
1 Fragile X syndrome is an X-linked dominant disea se caused by abnormal expansion of the CGG trinucleotide repea t . Individuals with this disea se have
2 stereotypical elongat ed facies with protruding ears, low muscle tone, and macro-orchidism. Patients also have varying degrees of mental ret ardation .
Although both sexes are affected, males usually suffer a more severe disea se.
3 Fragile X syndrome X-linked dominant inheritance Intellectual disability Sex linkage Trinucleotide repeat disorder Hypotonia Muscle tone

4 D is no t co rrect. 9 % cho se this.


5 Myotonic dystrophy is an autosomal dominant, multisystem disea se caused by abnormal expansion of the trinucleotide repea t CTG. Symptoms include
progressive proximal lower extremity weakness, fatigue, and muscle pain secondary to muscle wasting. Other extra muscular manifestations include
6 cat aracts and arrhythmias.
Myotonic dystrophy Dominance (genetics) Autosome Trinucleotide repeat disorder Cataract Muscle atrophy Cardiac arrhythmia Fatigue (medical)
7
Anatomical terms of location Myalgia Muscle
8
E is no t co rrect. 5 % cho se this.
9 Friedreich's at axia is an autosomal recessive disea se caused by abnormal expansion of the trinucleotide repea t GAA, rendering the frat axin protein
10 dysfunctional. Frat axin is believed to clear the peri-mitochondrial cytosol of iron radicals, thereby preventing oxidative damage. Without this protection,
the disea se causes progressive degeneration of the neural tissue in the spinal cord, lea ding to gait disturbance and speech difficulty.
11 Friedreich' s ataxia Frataxin Autosomal recessive Ataxia Gait abnormality Dominance (genetics) Protein Cytosol Trinucleotide repeat disorder Autosome

12 Spinal cord Gait Nervous tissue Oxidative stress Recessive

13
• 14 Botto m Li ne:
• 15 Some hereditary disea ses are caused by abnormal expansion of a specific trinucleotide repea t . CAG is associat ed with Huntington's disea se, GAA with
Friedreich's at axia, CTG with myotonic dystrophy, and CGG with fragile X syndrome. There are a multitude of other trinucleotide repea t disorders that
• 16 are not commonly t ested on the USMLE.
Huntington' s disease Fragile X syndrome Friedreich' s ataxia Myotonic dystrophy Trinucleotide repeat disorder Ataxia Genetic disorder
• 17

• 18
• 19
lijj ;fi IJ l•l fo r yea r:l 2017 ..
• 20 FI RST AI D FA CTS

• 21

6
lock Suspend
s 0
End Block
Item: 14 of 64 ~ 1 • M k -<:J 1>- Jil ~· !:';-~
QIO: 5159 ..L ar Pre v ious Next Lab~lues Notes Calcula t o r

1
&

A 39-year-old man with a history of hypertension presents to the emergency department with gross hematuria shortly after being hit in the flank with
a ball during a tennis match. An ultrasound performed in the emergency department shows numerous cysts in both kidneys and the liver. On further
IA•A] &

2
questioning he states that his mother has had a renal transplant. While Informing the patient of the diagnosis, the physician explains that the disease
3 Increases the risk of an aneurysm.
4
5 Assuming the patient's wife does not carry the gene, what are the chances that their newborn child has the disease?

6 :
A. 0%
7
8 B. 25%

9 c. 50%
10
o. 75%
11
E. 100%
12
13
• 14
• 15
• 16
• 17
• 18
• 19
• 20
• 21

a
Lock
s
Suspend
8
End Bl ock
Item: 14 of 64 ~. I • M k <:] t> al ~· ~
QIO: 5159 .l. ar Previous Next lab 'Vfl1ues Notes Calculator

1
The co rrect a nswer is c. 74% cho se this.
2 Gross hematuria after trauma is a classic presentation of autosomal dominant polycystic kidney disea se ( ADPKD), which is confirmed by the presence of
3 numerous bilat eral renal cysts at a young age. Liver or pancrea tic cysts are associat ed with AD PK D but are not required for diagnosis. Brain aneurysms
occur in about 8% of patients with AD PK D, which is twice the ra t e of the normal population, but asymptomatic screening for berry aneurysms is only
4 recommended in AD PK D patients if it has been previously found in the family. Because the disea se is autosomal dominant and only the patient is affected
(wife is homozygous for the normal gene), his newborn son has a 5 0 % chance of inheriting the disea se.
5 Hematuria Polycystic kidney disease Gene Homozygous Dominance (genetics) Autosomal dominant polycystic kidney disease Autosome liver Asymptomatic
6 Pancreas Kidney Kidney disease Major trauma Aneurysm Brain Human brain

7 A is no t co rrect. 7% cho se this.


8 Because the patient is affected with an autosomal dominant disea se, there is a 5 0 % chance of his newborn having the disea se. A 0 % chance would occur
in this scenario if the disea se were X-linked.
9 Dominance (genetics) Autosome Sex linkage

10 B is no t co rrect. 1 0 % cho se this.

11 If the disea se were passed in an autosomal recessive manner and the wife of the patient was also a carrier; then the chance of their son having the
disea se would be 2 5 % .
12 Autosomal recessive Recessive Autosome Dominance (genetics)

13 D is no t co rrect. 3 % cho se this.


14 If both parents carried the autosomal dominant gene, then the chance of the son having the disea se would be 75 % .
Gene Dominance (genetics) Autosome
• 15
E is no t co rrect. 6 % cho se this •
• 16
If the disea se was Y-linked, then the chance of the son having the disea se would be 100% .
• 17 Y linkage

• 18
• 19 Botto m Line:
• 20 AD PK D is autosomal dominant as the name implies, and classically manifests with bilat eral renal cysts, liver cysts, and hypertension at a young age .
Dominance (genetics) liver Hypertension Autosome Cyst
• 21

6
lock
s
Suspend
0
End Block
Item: 14 of 64 ~. I • M k <:] t> al ~· ~
QIO: 5159 .l. ar Previous Next lab 'Vfl1ues Notes Calculator

1 •

2 FA17 p 573.1
Renal cyst disorders
3
4 Autosomal dominant a
lumerous cysts in cortex and medulla causing bilateral enlarged kidneys ultimately destroy
polycystic kidney kidney parenchyma. Presents with Aank pain, hematuria, hypertension, urinary infection,
5
disease progressi,·e renal fai lure in - 50% of individuals.
6 Mutation in PKDl (85% of cases, chromosome 16) or PKD2 (15% of cases, chromosome 4). Death
7 from complications of chronic kidney disease or hypertension (caused by t renin production).
8 Associated with berry aneurysms, mitral ,·alve prolapse, benign hepatic cysts, diverticulosis.
9 Treatment: ACE inhibitors or ARBs.
10 Autosomal recessive Cystic di lation of collecting ducts [i]. Often presents in infancy. ssociated with congenital
polycystic kidney hepatic fibrosis. Significant oliguric renal failure in utero can lead to Potter sequence. Concerns
11
disease beyond neonatal period include systemic hypertension, progressive renal insufficiency, and portal
12
hypertension from congenita I hepatic fibros is.
13
Medullary cystic Inherited disease causing tubulointerstitial fibrosis and progressive renal insufficiency with inability
14 disease to concentrate urine. Medullary cysts usually not visualized; shrun ken kidneys on ultrasound.
• 15 Poor prognosis.
• 16 Simple vs complex Simple cysts are filled with ultra filtrate (anechoic on ultrasound [!t). Very common and account for
• 17 renal cysts majority of all renal masses. Found incidentally and typically asymptomatic.
• 18 Complex cysts, including those that arc septated, enhanced, or have solid components on imaging
• 19
require follow-up or removal due to risk of renal cell carcinoma .
r: . .'
~
• 20 • ::. I

• 21

I _... ,. _..~

6
lock
s
Suspend
0
End Block
Item: 15 of 64 ~ 1 • M k -<:J 1>- Jil ~· !:';-~
QIO: 1194 ..L ar Pre v ious Next Lab~lues Notes Calcula t o r
& &
1
Therapeutic proteins such as growth hormone can be produced using recombinant DNA techniques. DNA sequenced from humans is inserted Into
2 plasmlds used to transform Escherichia coli. These transformed bacteria produce large amounts of growth hormone chains, which are purified and
allowed to fold into active proteins.
3
4 Which of the following features of the genetic code makes this technique possible?
5
:
6 A. It is bidirectional
7
B. It Is commaless
8
c. It Is degenerative
9
10 D. It Is semiconservative

11 E. It is universal
12
13
14
0 15
0
16
0
17
0
18
• 19
0 20
• 21

a
Lock
s
Suspend
8
End Bl ock
Item: 15of64 ~. , . M k <:] t> al ~· ~
QIO: 1194 .l. ar Previous Next Lab 'Vfll ues Notes Calculator

1
The co rrect a nswer is E. 61% cho se this.
2 The genetic code is universal. Even extremely distant organisms (eg, mycobacteria, Archaebacteria) have only minute changes in the meaning of the
genetic code. It remains almost entirely conserved among species. This fea ture of the genetic code allows bacteria to make growth hormone using DNA
3
sequenced from humans.
4 Genetic code Mycobacterium DNA sequencing Growth hormone Bacteria DNA Archaea Hormone Species Conserved sequence Whole genome sequencing

5 A is no t co rrect. 9 % cho se this.

6 Bidirectionality is not a fea ture of the genetic code, but rather a fea ture of DNA replication, where synthesis begins at the origin and moves in both
directions at the same time. DNA is always synthesized in the 5 ' -3 ' direction .
7 DNA replication DNA Genetic code

8 B is no t co rrect. 5 % cho se this.

9 Although the genetic code is commaless and without punctuation, this is not the fea ture that allows Escherichia coli to produce human growth hormone.
Ea ch nucleotide is rea d only once in groups of three called cod o ns. With the exception of some viruses, codons do not overlap.
10 Growth hormone Genetic code Nucleotide Hormone Virus

11 c is no t co rrect. 6 % cho se this.


12 Although the genetic code is degenerative, or redundant, this is not the fea ture that allows Escherichia coli to produce human growth hormone. An amino
acid can be coded for by more than one codon . The variation occurs in the third base of the codon .
13 Growth hormone Amino acid Genetic code Hormone

14 D is no t co rrect. 19% cho se this.

15 Semiconservative is not a fea ture of the genetic code. It describes a characteristic of the DNA replication process, where ea ch daughter chromosome
contains a parental strand and a new complementary strand .
• 16 Semiconservative replication DNA replication Genetic code Chromosome DNA Complementary DNA
• 17

• 18 Botto m Li ne:
• 19 The universal nature of the genetic code makes it possible for other organisms, such as bacteria, to synthesize human gene products.
Gene Genetic code Bacteria
• 20
• 21

6
lock
s
Suspend
0
End Block
Item: 15 of 64 ~ 1 • M k -<:J 1>- Jil ~· !:';-~
QIO: 1194 ..L ar Pre v ious Next Lab~lues Notes Calcula t o r
& &
1
FA17 p 34.2
2
Genetic code features
3
Unambiguous Each codon specifies on I~ I amino acid.
4
Degenerate/ ~ lost amino acids are coded b) multiple codons. Exceptions: methionine and tryptophan encoded
5
redundant Wobble- codons that differ in 3rd, "wobble" b) on!) 1 codon (AUG and UGG, rcspccti,ely).
6
position may code for the same tR l /amino
7 acid. Specific base pairing is usuall) onl)
8 requi red in the first 2 nucleotide positions of
9 m RNAcodon.
10 Comma less, Read from a fixed starting point as a continuous Exceptions: some vimses.
11 nonoverlapping sequence of bases.
12 Universal Genetic code is conserved throughout Exception in humans: mitochondria.
e\·olution.
13
14
FA17 p 51 .4
15
• 16 Cloning methods C lon ing is the production of a recombinant DNA molecule that is self perpetuating.
Steps:
• 17
I. Isolate eukaryotic mR A (post-R A processing) of interest.
. 18
2. Expose mR 'A to reverse transcriptase to produce eDNA (lacks introns).
• 19 3. Insert cOl A fragments into bactcriill plasmids contilining antibiotic resistance genes.
• 20 4. Transform recombinant plasmid into bacteria .
• 21 5. Surviving bacteria on an tibiotic medium produce cloned D l (copies of cO 'A).

a
Lock
s
Suspend
8
End Bl ock
Item: 16 of 64 ~ 1 • M k -<:J 1>- Jil ~· !:';-~
QIO: 3697 ..L ar Pre v ious Next Lab~lues Notes Calcula t o r
& &
1
An 8-year-old boy in the clinic has upslanted eyes with epicanthal folds, a flat nasal bridge, a transverse palmar crease, a subendocardial cushion
2 defect, and intellectual disability.

3
A defect In which step of cell division is the most likely cause of the patient's defects?
4
5 :
A. MeiOSIS I anaphase
6
B. Meiosis 1 metapha se
7
8 c. Meiosis 1 prophase

9
D. Meiosis II anaphase
10
E. Meiosis II metaphase
11
12 F. Meiosis II prophase

13
14
15
• 16
• 17
• 18
• 19
• 20
• 21

a
Lock
s
Suspend
8
End Bl ock
Item: 16 of 64 ~ 1 • M k -<:J 1>- Jil ~· !:';-~
QIO: 3697 ..L ar Prev ious Next Lab~lues Notes Calculat o r

& &
1 Th e co rrect a nsw er i s A. 390/o chose this.

2 The most common error giving rise to Down syndrome is nondisjunction of chromosome 21 during segregation of homologous chromosomes In maternal
meiosis I. In normal meiosis, diploid germ cells undergo one round of replication followed by two consecutive cell divisions to produce haploid gametes.
3 Following replication, each chromosome consists of two sister chromatids. In prophase of meiosis I , homologous chromosomes pair and undergo
recombination. In metaphase of meiosis I , paired homologs align at the metaphase plate. In anaphase of meiosis I , t he homologs segregate. Each
4 homolog still consists of two sister chromatids. A segregation error (nondisj unction ) during meiosis I anaphase may ca use an abnormal number of
chromosomes in the daughter cells. Ninety-five percent of Down syndrome cases result from nondisjunction, and remaining cases are caused either by a
5 Robertsonlan translocation or by trisomy 21 mosaicism.
6 Meiosis Rl "'rts01 .an translocation Down syndrome Nondisjunction Haplood Anaphase Diploid Prophase Metaphase Ploidy Chromosome 21 \human 1
Sister chromatids Homology (biology) Meiosis I Chromosome Homologous chromosome Mosaic (genetics) Trisomy Gamete Genetic recombination Chromatid
7
Germ eel Spindle apparatus Chromosomal translocation
8
B i s not co rrect. 15% chose this.
9
In meiosis I metaphase, homologous pairs of chromosomes align on the equator.
10 Meiosis Metaphase Chromosome Homology (biology) Meiosis I

11 C is not correct. 9% ch ose this.

12 In meiosis I prophase, the chromatids condense, homologous chromosomes pair, and recombination occurs. Spindle fibers begin to form, and the nuclear
membrane disappears.
13 Meiosis Prophase Meiosis I Homologous chromosome Nuclear membrane Spindle apparatus Chromosome Chromatid Homology {biology) Genetic recombination

14 Biological membrane

15 D is not co rrect. 180/o chose this.


In meiosis II anaphase, the centromeres separate, and the two chromatids of each chromosome migrate to opposite poles.
16
Meiosis Anaphase Meiosis II Centromere Chromosome Chromatid
• 17
E i s n ot co rrect. 130/o chose this .
• 18 In Meiosis II metaphase, homologous chromosomes line up on the metaphase plate.
• 19 Meiosis Metaphase Homologous chromosome Chromosome Homology (biology) Spindle apparatus

• 20 F is not correct. 60/o chose this•


In meiosis II prophase, chromosomes condense, and meiotic spindles re-form. There is no additional replica t ion or recombination .
• 21
• Meios•s P oohase Meiosis II Chromosome Genetic recombination

a
Lock
s
Suspend
8
End Block
Item: 16of64 ~. , . M k <:] t> al ~· ~
QIO: 3697 .l. ar Previous Next lab 'Vfl1 ues Notes Calculator

1
Bottom Line:
2
In the vast majority of cases, Down syndrome is due to nondisjunction during anaphase of meiosis I.
3 Down syndrome Nondisjunction Meiosis Anaphase

4
5
6 lijj ;fi IJ l•l for year:l 2017 ..
FI RST AI D FAC TS
7

8 FA17 p 59.1
9 Autosomal trisomies
10 Down syndrome Findings: intellectual disability, flat facies, Incidence 1:700.
11 (trisomy 21) prominent epicanthal folds, single palmar Drinking age (21).
12 crease, gap between 1st Z toes, duodenal Most common viable chromosomal disorder and
atresia, Hirschsprung disease, congenital heart most common cause of genetic intellectual
13
disease (eg, atrioventricular septal defect), disability.
14
Brush field spots. Associated with early-onset First-trimester ultrasound commonly sho,,·s
15 Alzheimer disease (chromosome 21 codes for t nuchal translucency and hypoplastic nasal
16 amyloid precursor protein) and t risk of ALL bone; l serum PAPP-A, t free ~-hCG.
• 17 andAML. Second-trimester quad screen shows
95% of cases due to meiotic nondisjunction l a -fetoprotein, t ~ -h CG, l estriol,
• 18
(t with advanced maternal age; from 1:1500 in t inhibin A.
• 19
women< 20 to 1:25 in women> 45 rears old).
• 20 4% of cases clue to unbalanced Robertson ian
• 21 translocation, most typically between

6
lock
s
Suspend
0
End Block
Item: 16 of 64 ~ 1 • M k -<:J 1>- Jil ~· !:';-~
QIO: 3697 ..L ar Pre v ious Next Lab~lues Notes Calcula t o r

1

2 Patau syndrome Findings: severe intellectual clisabilit), rocker- Incidence 1:1 5,000.
(trisomy 13) bottom feet, microphthalmia, microcephaly, Puberty (13).
3
cleft liP/Palate, holoProsencephaly, First-trimester pregnancy screen shows 1 free
4
Pol) dact) ly, cutis a Plasia, congenital heart P-hCG, 1 PAPP-A.
5 disease. Death usually occurs by age I.
6
Nondisjunction in meiosis I Nondisjunction in meiosis II
7
8
9 Meiosis I

10
11 Nondisjunction

12
13 c' c' c'
14 c' Meiosis II
c'
15 c' '
) ~''"'"i"""'"
16
• 17 A A A
• 18
• 19
Ill Ill I J Gametes
II II I Ill
n+l n+ l n-1 n- 1 n n n- 1 n+l
• 20
Trisomy Monosomy Normal Monosomy Trisomy lil
• 21

a
Lock
s
Suspend
8
End Bl ock
Item: 17 of 64 ~ 1 • M k -<:J 1>- Jil ~· !:';-~
QIO: 1193 ..L ar Pre v ious Next Lab~lues Notes Calcula t o r
&
1
A 66-year-old woman with a history of cigarette smoking and extensive sun exposure is found to have a basal cell carcinoma. The cancer is excised,
2 with clear margins, and genotyped so the the causative mutation may be Identified. The mutation of interest is shown in the DNA-coding region and
corresponding polypeptides shown below.
3
Patients' protein
4
5 GTC ACC GAG GAA GAG TAC GGT
6
va: Thr Glu Glu Glu Tyr Gly
7
8 Reference DNA
9
GTC ACC GAG AAG AGT ACG GTG
10
11 va -_ Thr Glu Lys Ser Arg Val
12
13
Which of the following diseases is caused by the same type of mutation as this patient's mutation?
14
:
15 A. Cystic fibrosis
16
B. Duchenne muscular dystrophy
• 17
. 18 c. Huntington disease

• 19 D. Sickle cell disease


• 20
E. Williams syndrome
• 21

a
Lock
s
Suspend
8
End Bl ock
Item: 17 of 64 ~ 1 • M k -<:J 1>- Jil ~· !:';-~
QIO: 1193 ..L ar Prev ious Next Lab~lues Notes Calculat o r

& &
1 The co rrect a nsw er i s B. 480/o chose this.
2 This Is an example of a frameshift mutation, a nucleotide insertion or deletion that alters the reading frame . Because each codon comprises three bases,
any Insertion or deletion that is not a multiple of three bases results In a frameshlft mutation. In this case, a guanine residue was inserted, shifting the
3 reading frame downstream from the insertion. The diagram shown here provides a nice example of a frameshift mutation and how it garbles the
downstream message. Frameshift mutations commonly result in premature stop codons and protein truncations.
4
Dystrophin Is a very large protein found in skeletal and cardiac muscle and In neurons, although its role in the last of these is largely unknown at this
5 time. Loss of function mutations in the dystrophin gene located on the x-chromosome result in progressive muscular injury from birth, and the severity of
6 the symptoms correlate with the significance of the mutation. Duchenne muscular dystrophy (DMD) presents at a younger age and is more severe than
Becker muscular dystrophy (BMD). This is largely because in DMD the most common cause is a frameshift mutation that results in a premature stop
7 codon, leading to the inability to produce any functional dystrophin. In comparison, the molecular basis of BMD is often an in-frame (or nonframeshlft)
deletion, which allows for the production of dystrophin that retains some function.
8
Frameshifl mutation Dystrophin Duchenne muscular dystrophy Stop codon Bee e s muscular dystrophy Gene Muscular dystrophy cardiac muscle X chromosome
9
Mutation Translational frameshift Reading frame Guanine Nucleotide Protein Genetic code Neuron Residue {chemistry) Amino acid Nonsense mutation
10
11 CANYOUDIGHERNEWHAT . . CAN YOU DIG HER NEW HAT
12
CANXYOUDIGHERNEWHAT. . CAN XYO UDI GHE RNE WHAT
13
A is not co rrect. 160/o chose this.
14
Although cystic fibrosis can be caused by a variety of mutations of the CFTR gene on chromosome 7, it is most commonly caused by a three-nucleotide
15 deletion of codon 508, which normally would encode for phenylalanine. This patient's genetic sequencing does not display such a deletion, and moreover
one would not expect the reading frame to be altered with the t.-FS08 mutation.
16
Cystic fibrosis Phenylalanine Gene Chromosome 7 {human) Cystic fibrosis transmembrane conductance regulator Mutation Genetic code Reading frame
17 Chromosome Fibrosis DNA sequencing Whole genome sequencing

. 18 C i s n ot correct. 90/o chose this.


. 19 Huntington disease is caused by an expansion of trinucleotide repeats (CAG) on chromosome 4. When DNA polymerase encounters trinucleotide repeats,
It Is prone to slippage and insertion of additional repeats. As a result, Huntington disease displays genetic anticipation, in which subsequent generations
• 20 have earlier onset and more severe disease.
Huntington·~ disease DNA polymerase Anticipation (genetics) Chromosome 4 (human) Trinucleotide repeat disorder Chromosome DNA Polymerase
. 21

a
Lock
s
Suspend
8
End Block
Item: 17 of 64 ~. I • M k <:] t> al ~· ~
QIO: 1193 .l. ar Previous Next Lab 'Vfll ues Notes Calculator

1 D is no t co rrect. 18% cho se this.


2 Sickle cell disea se is caused by a missense mutation in which valine is replaced with glutamic acid . Recall that a missense mutation is a mutation within a
coding region that results in a different amino acid . Missense mutations may be conservative (if an amino acid with similar biochemical characteristics is
3 substituted) or may drastically change the phenotype.
Missense mutation Sickle-cell disease Glutamic acid Valine Amino acid Phenotype Mutation Coding region Biochemistry
4
E is no t co rrect. 9 % cho se this.
5
Williams syndrome is caused by a microdeletion in 7q, resulting in intellectual disability, elfin facies, strong language skills, and cardiovascular
6 abnormalities. A microdeletion is a deletion so small that it cannot be det ected by light microscopic evaluation of chromosomal banding patterns. This
patient's genetic sequence does not display a microdeletion .
7 Williams syndrome Intellectual disability Deletion (genetics) Circulatory system
8
9
Bo tto m Line:
10
A frameshift mutation is a nucleotide insertion or deletion that alters the rea ding frame. Duchenne muscular dystrophy is one example of a frameshift
11 mutation .
Frameshift mutation Duchenne muscular dystrophy Muscular dystrophy Mutation Translational frameshift Nucleotide Reading frame
12
13
14
I ill ;fi 1!1 I•J f o r year:[ 20 17 ..
FI RST AID FA CTS
15
16
FA17 p 36.1
17
• 18
Mutations in DNA Severi ty of damage: silent<< m issense< nonsense< frameshift.
For point (silent, missense, and nonsense) mutations:
• 19
• Transition - purine to purine (eg, A to G) or prrimidine to pyri m idine (eg, C toT).
• 20 • Transversion- purine to pyrimidine (eg, A to T ) or pyrim id ine to purine (eg, C to C) .
• 21
Silent Nucleotide substitution but codes for same
6
lock
s
Suspend
0
End Block
Item: 17 of 64 ~ 1 • M k -<:J 1>- Jil ~· !:';-~
QIO: 1193 ..L ar Pre v ious Next Lab~lues Notes Calcula t o r
& &
1

2
FA17 p 33.1
3
Nucleotides lucleoSide = base + {deoxy)ribose (Sugar).
4
NucleoT ide = base + (deoxy)ribose + phosphaTe; 5' end of incoming nucleotide bears the
5 linked by 3'-)' phosphodiester bond. triphosphate (energy source for the bond).
6 Triphosphate bond is target of 3' hydrO\) I
7 attack.
P U Rines (.-\,C )-2 rings. PU Re \ s C old.
8
PYrimidines (C ,U,T )-1 ring. CUT the PY (pic).
9
10
.
Th,. mine has a methvl.
Dcamination of crtosinc makes uracil. C-C bond (3 H bonds) stronger than A-T bond
11 Deamination of adenine makes guanine. (2 II bonds). f C -C content .... f melting
12 Uracil found in R 'A; thymine in D temperature of DNA. "C -C bonds are like
13
Methylation of uracil m akes thymine. C razy C lue."

14 Purine (A. G) Pyrimidine (C. U. n


GAG -Am ino acids necessary for purine
15 C02 synthesis:
Carbamoyl
16
Aspartate I .......-Glycine phosphate \
{ Aspartate
G lycine
\ C N c
17 N C N c Aspartate
C- NlO.Formyl· G lutamine
. 18 c c tetrahydrofolate c c
• 19 I N N N
NIO.Foonyt· ~ \ .
• 20 tetrahydrololate Glutamine

• 21

a
Lock
s
Suspend
8
End Bl ock
Item: 17 of 64 ~ 1 • M k -<:J 1>- Jil ~· !:';-~
QIO: 1193 ..L ar Pre v ious Next Lab~lues Notes Calcula t o r
& &
1 FA17 p 57. 1
2 Muscular dystrophies
3 Duchenne X-linked disorder typically due to framcshift Duchenne = deleted dystrophin.
4 or nonsense mutations - truncated or Oystrophin gene (DMD) is the largest
5
absent dystrophin protein - progrcssi\'C protein-coding human gene - f chance of
myofiber damage. Weakness begins in pelvic ~ponlaneous mutation. Dystrophin helps
6
gircUe muscles and progresses superiorly. anchor muscle fibers, primarily in skeletal and
7 Pseudohypertrophy of calf muscles due to cardiac muscle. It connects the intracellular
8 •••• fibrofatty replacement of muscle . \ addling crtoskeleton (actin) to the transmembrane
9
f!_
.
· rl
D
gait. Onset before 5 years of age. Dilated proteins a- and ~-dystroglycan, which are
10
cardiomyopathy is common cause of death. connected to the extracellular matrix (ECt.. l).
l .oss of dystrophin results in myonecrosis.
11
f CK and aldolase are seen; genetic testing
12 confirms diagnosis.
13 Becker X-linked disorder typically due to non- Deletions can cause both Duchennc and
14 frameshift deletions in dystrophin gene Becker muscular dystrophies. 1'\ of cases have
15 {partially functional instead of trunc:ltccl). Less large deletions spanning one or more exons.
16 severe than Duchcnnc. Onset in adolescence
or early adulthood.
17
Myotonic type 1 Autosomal dominant. CTG trinucleotide repeal C.1taracts, Toupee (early balding in men),
. 18
expansion in the DMPK gene - abnormal Gonadal atroph)'.
. 19
expression of myotonin protein kinase
• 20 -+ myotonia, muscle wasting, cataracts,

. 21 testicular atrophy, frontal balding, arrhythmia .



a
Lock
s
Suspend
8
End Bl ock
Item: 18 of 64 ~ 1 • M k -<:J 1>- Jil ~· !:';-~
QIO: 2683 ..L ar Pre v ious Next Lab~lues Notes Calcula t o r

1 &

A 20-year-old man comes to the physician for a physical examination before beginning college . He states that he has been in good health except for a
complaint of bloody stools that he attributes to occasional constipation. On further questioning, the physician discovers that the patient's mother died
lA• AI ·
2
of colorectal cancer at the age of 42 years. The physician sends the patient for a colonoscopy. The results shown in the image reveal hundreds of
3 polyps throughout the large intestine.

4
5
6
7
8
9
10
11
12
13
14
15
16
17
. 18
. 19
• 20
. 21

a
Lock
s
Suspend
8
End Bl ock
Image courtesy of Wikimedia Commons
12
13 What other malignancy may be associated with this finding?
14
:
15 A. Medulloblastoma

16
B. Melanoma
17
c. Osteomas
. 18
• 19 o. Endometrial cancer

• 20 E. Chronic myelogenous leukemia


• 21

a
Lock
s
Suspend
8
End Block
Item: 18of64 ~. , . M k <:] t> al ~· ~
QIO: 2683 .l. ar Previous Next lab 'Vfl1 ues Notes Calculator

1
The co rrect a nswer is A. 28% cho se this.
2 This patient has familial adenomatous polyposis (FAP), which is an autosomal dominant disea se caused by a mutation of chromosome 5. FAP classically
manifests with rectal bleeding in a 20 - to 40 -year-old people with a family history of colon polyps and/or colorectal cancer. On colonoscopy, a patient with
3 FAP will have hundreds of polyps. FAP has primary syndromes with which it may be associat ed, ea ch with risks of other cancers : Turcot syndrome (risk of
4 CNS tumors, including medulloblastoma) and Gardner syndrome (thyroid, liver; kidney cancers, osteomas, and dental abnormalities).
Gardner' s syndrome Familial adenomatous polyposis Medulloblastoma Mismatch repair cancer syndrome Colorectal cancer Colonoscopy Dominance (genetics)
5
lower gastrointestinal bleeding Colon (anatomy) liver Chromosome 5 (human) Adenoma Chromosome Mutation Osteoma Polyp (medicine) Cancer
6 Central nervous system Thyroid Rectum
7 B is no t co rrect. 13 % cho se this.
8 Melanoma is not generally associat ed with familial adenomatous polyposis or the APC gene. The classic mutation for melanoma is an activating somatic
mutation in BRAF kinase.
9 Familial adenomatous polyposis Somatic mutation Gene Melanoma BRAF (gene) Mutation Adenomatous polyposis coli Somatic (biology)
10 c is no t co rrect. 28% cho se this.
11 Osteomas are commonly seen in patients with Gardner syndrome, which is connected to familial adenomatous polyposis and APC mutations. However;
osteomas are not considered a malignancy as these are benign bone tumors. Dental abnormalities are also seen as well in Gardner syndrome.
12 Gardner' s syndrome Familial adenomatous polyposis Malignancy Cancer Adenomatous polyposis coli Osteoma Bone Benign tumor Mutation

13 D is no t co rrect. 24% cho se this.


14 Endometrial cancer is not classically associat ed with familial adenomatous polyposis (FAP). However; it is associat ed with Lynch syndrome (hereditary
nonpolyposis colorectal cancer) which is caused by mutations in DNA mismatch genes (eg, MSH2, MLHl ). Lynch syndrome classically fea tures colorectal
15 cancers (but not the diffuse polyps seen in FAP), endometrial cancers, ovarian cancers, and others.
MLHl Familial adenomatous polyposis Endometrial cancer MSH2 Hereditary nonpolyposis colorectal cancer Colorectal cancer Endometrium Cancer DNA Mutation
16
17 E is no t co rrect. 7% cho se this.
Chronic myelogenous leukemia (CML) is not associat ed with APC gene mutations like those implicat ed in familial adenomatous polyposis. The classic DNA
18 mutation associat ed with CML is the Philadelpha chromosome, a translocation between the breakpoint cluster region (BCR ) on chromosome 22 and the
ABL gene on chromosome 9. This results in the myeloid proliferation that we see in CML.
• 19
Familial adenomatous polyposis Chronic myelogenous leukemia Gene leukemia Chromosome 22 (human) Chromosome Mutation BCR (gene) ABl (gene)
• 20 Chromosome 9 (human) Myeloid Chromosomal translocation Adenomatous polyposis coli DNA
• 21

6
lock
s
Suspend
0
End Block
Item: 18of64 ~- , . M k <:] t> al ~· ~
QIO: 2683 .l. ar Previous Next lab 'Vfl1 ues Notes Calculator

1
Bottom Line:
2
Fa milial adenomatous polyposis results from a mutat ion on chromosome 5 of the APC gene. It is associated with Gardner syndrome and Turcot
3 syndrome, each of which have other cancer risks.
Gardner' s syndrome Familial adenomatous polyposis Mismatch repair cancer syndrome Gene Chromosome s (human) Adenomatous polyposis coli Mutation
4
Chromosome Cancer
5
6
7 lijj ;fi IJ l•l for year:l 2017 ..
FIRST AID FACTS
8
9
FA17 p 218.2
10
Tumor suppressor Loss of function -+ f cancer risk; both (two) alleles of a tumor suppressor gene must be lost for
11
genes expression of disease.
12 GENE GENE PRODUCT ASSOCIATED CONDITION
13 APC 1'\cgative regulator of ~-cateni n/WNT pathway Colorcctal cancer (associated with FAP)
14 BRCA 1/ BRCA2 D A repair protein Breast, ovarian, and pancreatic cancer
15 CDKN2A pl6, blocks C 1 -+ S phase lelanoma, pancreatic cancer
16
DCC D CC- D eleted in C olon C ancer Colon cancer
17
DPC4/SMAD4 DPC - Deleted in Pancreatic C ancer Pancreatic cancer
18
MEN1 Men in ME 1
• 19
NF1 eurofibromin (Ras GTPase activating protein) Neurofibromatosis type 1
• 20
NF2 Merlin (schwannomin} protein Neurofibromatosis type 2
• 21
• ---·· ' . • • I • ..'
6
lock
s
Suspend
0
End Block
Item: 18 of 64 ~ 1 • M k -<:J 1>- Jil ~· !:';-~
QIO: 2683 ..L ar Pre v ious Next Lab~lues Notes Calcula t o r
& &
1
2 FA17 p56.1
3 Autosomal dominant Achondroplasia, autosomal dominant polycystic kidney disease, familial adenomatous polyposis,
4 diseases familial hypercholesterolemia, hereditar) hemorrhagic telangiectasia, hereditar) spherOC) tosis,
5 Huntington disease, Li-Fraumeni S) ndrome, \ larfan syndrome, multiple endocrine neoplasias,
6
neurofibromatosis type I (von Recklinghausen disease), neurofibromatosis type 2, tuberous
sclerosis, von Hippel-Lindau disease.
7
8
FA17 p 370. 2
9
Polyposis syndromes
10
Familial adenomatous Autosomal dominant mutation of APC tumor suppressor gene on chromosome 5q. 2-hit hypothesis.
11 polyposis Thousands of polyps arise starting after puberty; pan colonic; always im·olves rectum. Prophylactic
12 colectomy or else 100% progress to CRC.
13 Gardner syndrome FAP +osseous and soft tissue tumors, congenital hypertrophy of retinal pigment epithelium,
14 impacted/supernumerary teeth.
15 Turcot syndrome F P/Lynch syndrome+ malignant C S tumor (eg, medulloblastoma, gl ioma). Turcot= Turban.
16 Peutz-Jeghers Autosomal dominant syndrome featuring numerous hamartomas throughout CI tract, along with
17 syndrome hyperpigmented mouth, lips, h;mcls, gcnitalin. Associnted with t risk of breast and C I cancers (cg,
18
colorectal, stomach, small bowel, pancreatic).
. 19 Juvenile polyposis Autosomal dominant syndrome in children (typically< 5 years old) featuring numerous
• 20
syndrome hamartomatous polyps in the colon, stomach, small bo,,el. Associated with t risk ofCRC.

. 21

a
Lock Suspend
s 8
End Bl ock
Item: 19 of 64 ~ 1 • M k -<:J 1>- Jil ~· !:';-~
QIO: 4820 ..L ar Pre v ious Next Lab~lues Notes Calcula t o r

1 &

A 10-year-old boy suffers from recurrent bacterial infections, eczema, and nosebleeds. His maternal grandfather had similar symptoms, but the boy's
mother does not. The boy's platelet count is 60,000/mm•.
IA•A] &

2
3
If his mother becomes pregnant with another son, what are the odds that this next child will be affected by the same disorder?
4
5 :
A. Oo/o
6
B. 12.5%
7
8 c. 25%

9
o. 50%
10
E. 100%
11
12
13
14
15
16
17
18
. 19
• 20
. 21

a
Lock
s
Suspend
8
End Bl ock
Item: 19 of 64 ~. I • M k <:] t> al ~· ~
QIO: 4820 .l. ar Previous Next lab 'Vfl1 ues Notes Calculator

1 The co rrect answer is D. 63% cho se this.


This boy's symptoms are classic for Wiskott-Aidrich syndrome, which characteristically involves recurrent pyogenic infections, thrombocytopenia, purpura,
2
and eczema. It is characterized by low IgM levels and high IgE levels. The syndrome's inheritance pattern is X-linked recessive, so this boy must have
3 inherited it from his mother. His mother must be het erozygous for the disea se allele because she herself is not affected. If a disea se is inherited in an X-
linked recessive fashion, it will affect all males who receive the mutant allele but only females who are homozygous. These disea ses can get passed from
4 an affected male through a carrier female to an affected grandson and grea t grandson. If a male is affected with the disea se, he will pass his mutant X
allele to all of his daughters, and therefore ea ch female child will be het erozygous. Now, any of her sons will have a 5 0 % chance of inheriting her mutant
5 X allele and therefore a 5 0 % chance of having the disea se, which is the case here.
6 Thrombocytopenia Allele Wiskott-Aidrich syndrome Dermatitis Zygosity Heterozygous Immunoglobulin E Homozygous Immunoglobulin M Dominance (genetics)
X-linked recessive inheritance Sex linkage Mutant Recessive
7

8 A is no t co rrect. 3 % cho se this.


The son has a 5 0 % chance of being affected. Since the mother carries one mutant X allele, the son has a 5 0 % chance of inheriting either the mutant X or
9 the normal X from the mother. If the mother were pregnant with a female child, this female offspring would have the same 5 0/ 5 0 chance of inheriting
either the mutant or the normal X alllele but, this would give her a 0 % chance of having the disea se. This is becuase a male with one mutant X allele will
10 have the disase but a female with 1 mutant X alelle will be het erozygous, and not show signs of disea se.
11 Allele Heterozygous Mutant Mutant {Marvel Comics)

12 B is no t co rrect. 4 % cho se this.


Ea ch son has a 5 0 % chance of inheriting an X-linked recessive disea se if his mother is het erozygous.
13
Heterozygous X-linked recessive inheritance Recessive Dominance (genetics) Sex linkage
14
c is no t co rrect. 22% cho se this.
15 The son has a 5 0 % chance of being affected, since he has a 5 0 % chance of inheriting the mutant X allele from his het erozygous mother. If this disea se
were inherited in an autosomal recessive fashion, and both parents were het erozygous, then ea ch child would have a 2 5 % chance of inheritng the
16 disea se. However; this is not the case here, as Wiskott-Aidrich syndrome follows an X-linked recessive inheritance pattern .
17 Allele Wiskott-Aidrich syndrome Autosomal recessive Heterozygous X-linked recessive inheritance Autosome Dominance (genetics) Recessive Sex linkage Mutant

18 E is no t co rrect. 8 % cho se this.


The male offspring has a 5 0 % chance of being affected. Since the mother is het erozygous and not affected by the disea se, she only carries one mutant
19
allele and the son can inherit either the mother's normal X or her mutant X. In order for the son to have a 100% chance of inheriting the disea se, he
0 20 would have to have a 100% chance of inheriting a mutant X allele from his mother; which would mean that the mother would have 2 mutant X alleles.
This is not the case here, since we are told the mother is not affected by the disea se.
0
21 Allele Heterozygous Mutant

6
lock
s
Suspend
0
End Block
Item: 19 of 64 ~ 1 • M k -<:J 1>- Jil ~· !:';-~
QIO: 4820 ..L ar Pre v ious Next Lab~lues Notes Calcula t o r
& &
1
T cell disorders
2
Thymic aplasia 22qll deletion; failme Tetany (hypocalcemia). l T cells, l PTH, l Ca2+.
3
(DiGeorge syndrome) to de,·elop 3rd and 4th recurrent viral/fungal Absent thymic shadow on
4 pharyngeal pouches - absent infect ions (T-cell deficiency), CXR.
5 thymus and parath~Toids. conol runcal abnormalities
6 (eg, tetralogy of Fallot,
truncus arteriosus).
7
8
IL-12 receptor l Th I response. utosomal Disseminated mycobacterial l IFI\-y.
deficiency recess!\ e. and fungal infections; may
9
present after administration of
10 BCC ,·accine.
11
Autosomal dominant Deficiency ofTh 17 cells due to l<i \Ti'.D : coarse Facies, cold t lgE, l IF -y.
12 hyper-lgE syndrome $TAT3 mutation - impaired (noninnamed) staphylococcal t eosinophils.
13 (Job syndrome) recruitment of neutrophils to \ bsccsscs, retained primary
14 sites of infection. Teeth, t lgE , D ermatologic
problems (eczema). Bone
15
fr;~clurcs from mi nor trauma.
16
Chronic T-cell dysfunc tion. Many on invasive Candida albicans Absent in vitro T-cell
17
mucocutaneous causes. infect ions of skin and mucous proliferation in response lo
18 candidiasis membranes. Candida antigens.
19 Absent cutaneous reaction to
• 20 Candida antigens.
. 21 B- and T-cell d isorders

a
Lock
s
Suspend
8
End Bl ock
Item: 19 of 64 ~ 1 • M k -<:J 1>- Jil ~· !:';-~
QIO: 4820 ..L ar Pre v ious Next Lab~lues Notes Calcula t o r
& &
1
B- and T-cell disorders
2
Severe combined se,·eraltypes including Failure to thrive, chronic l T-cell receptor excision
3
immunodeficiency defecti,·e IL-2R gamma chain diarrhea, thrush. Recurrent circles (TRECs).
4 (most common, X-I inked), viral, bacterial, fungal, and Absence of thymic shadow
5 adenosine dcaminasc protozoal infections. (CXR), germinal centers
6 deficienc\' (autosomal Treatment: a,·oid Ji,·e vaccines. (lymph node biopsy), and
recessi,·e). gi,·c antimicrobial prophylaxis T cells (Aow cytometry).
7
and IVIC; bone marrow
8
transplant curati,·e (no
9 concern for rejection).
10 Ataxia-telangiectasia Defects in ATM gene ~ failure Triad: cerebellar defects t AFP.
11 to repair OJ A double strand (Ata\ia), spider Angiomas l IgA, IgC, and IgE.
12
13
14
"' D
breaks ~ cell cycle a rrcst. (telangiectasia · ), Ig. \.
de~ciency.
Lymphopenia, cerebellar
atrophy.
t risk of lymphoma and
leukemia.
15
16
Hyper-lgM syndrome Most commonly due to Severe pyogenic infections ormal or t lgM.
defective C040L on Th cells early in life; opportunistic ! l lgG, lgA, lgE.
17
~ class switching defect; infection with Pneumocystis, Failure to make germin<1l
18 X-linked rccessi,·e. Cryptosporidium, Cl\ IV. centers.
19
Wiskott-Aidrich J\ lutation in WASp gene; \\'ATER: W iskott- \ ldrich: l to normal IgG, Ig f.
• 20 syndrome leukocytes and platelets T hrombOC) topcnia, E czema, t IgE, IgA.
. 21 unable to reorganize actin Recurrent (pyogenic) Fewer and smaller platelets.

a
Lock
s
Suspend
8
End Bl ock
Item: 19 of 64 ~ 1 • M k -<:J 1>- Jil ~· !:';-~
QIO: 4820 ..L ar Pre v ious Next Lab~lues Notes Calcula t o r

1 &
antigen presentation. X-linked t risk of autoimmune disease &

2 recessl\·e. and malignancy.


3 Phagocyte dysfunction
4 leukocyte adhesion Defect in LFA-1 integrin Rccmrent skin and mucosal t neutroph ils.
5
deficiency (type 1) (CDlS) protein on bacterial infections, ab~cnt Absence of neutrophils at
phagocytes; impaired pus, impaired wound healing, infection sites.
6
migration and chemotaxis; delayed (> 30 days) separation
7 autosomal recessive. of umbilical cord.
8 Chediak-Higashi Defect in lysosoma I trafficking Recurrent pyogenic Giant granules ( : , arrO\\S) in
9 syndrome regulator gene (LYST). in feet ions by staph~ lococci granulocytes and platelets.
10 ~ licrotubuJe dysfunction in and streptococci. partial Pancytopenia.
11 phagosome-lysosome fusion; albinism, peripheral Mild coagulation defects.
autosomal recessive. neuropathy, progressive
12
neurodegeneration, infiltrati\'e
13
lymphoh istiocytosis.
14
15
Chronic Defect of ADPH oxidase t susceptibility to catalase$ Abnormal dihydrorhodam ine
16 -+ l reactive oxygen
granulomatous org<lll 1Sl11S. (How cytometry) test (l green
17 disease species (eg, superoxide) fluorescence).
18 and l respiratory burst Nitroblue tetrazolium dye
19 in neutrophils; X-I inked reduction test (obsolete) fai ls
• 20
recessi,·e most common. to turn blue.

. 21

a
Lock
s
Suspend
8
End Bl ock
Item: 20of64
QIO: 3972 ~
~ , .1 • Mar k <1
Previous
t>
Next Lab
d\ ues
•.

Notes
~
calculator

1
A 2 -year-old child has no red reflex in the r ight eye , He is subsequently found to have an eye tumor, as shown in the image, that is caused by dysfunction of a
2 specific cell- cycle regulatory gene product,

3
4
5
6
7
8
9
10
11
12
13
Image courtesy of the National Cancer Institute
14
15 What is the normal function of this gene product in a quiescent cell?
16
:
17 A. Prevents cell- cycle progression past the G1/ S checkpoint
18
B. Prevents cell- cycle progression past the G2/M checkpoint
19
• 20 C. Promotes DNA damage repair

• 21 D. Promotes histone acetylation


• 22
E. Regulates apoptosis and angiogenesis
• 23
• 24

a
Lock
s
Suspend
o
End Block
Item: 20of64
QIO: 3972
~ , .1 • Mar k
~
<1
Previous
t>
Next Lab
d\ ues
•.

Notes
~
calculator

1 Th e correct a nswer is A. 720/o ch ose t his.

2 This child most likely has r etinoblastoma, a r apidly progressive neoplastic gr owth in the r etina , Retinoblastoma may present in one eye, as in this patient, or bilater ally,
as in appr oximately 30% of cases , This vignette does not allude to any family histor y, in which case the r etinoblastoma is called sporadic, in contr ast with the familial
3 form, which is associated with a family histor y, I n either case, the disease is believed to ar ise from a loss -of-function mutation in the RB1 gene , RB1 is a tumor
suppr essor gene that normally binds the E2F transcription factor complex in quiescent cells, which prevents the cell from progressing through the G1/ S checkpoint. Under
4 appr opr iate conditions for cell r eplication, E2F is r eleased when the Rb protein becomes phosphorylated by cyclins and their cyclin -dependent kinases , loss of function of
5 the RB1 gene is associated with osteosar coma and r etinoblastoma ,
Osteosarcoma Transcription factor E2F Retina Gene Retinoblastoma protein Tumor suppressor gene Mutation Retinoblastoma Phosphorylation Transcription (genetics) Protein
6
Cyclin-dependent kinase Neoplasm Cyclin Kinase loss-of-function mutation
7
B is not correct. 110/o ch ose t his.
8 The G2fM checkpoint is another important cell- cycle r egulator y checkpoint. It provides another opportunity to prevent the cell from under going m itosis should the
9 envir onment be inappropriate for cell r eplication or there is DNA damage , Some important r egulator y proteins at the G2fM checkpoint include the CHK l and CHK2 kinases
through interactions with Grspecific cyclin A and CDK2 ,
10 Mitosis Cyclin A CHEK2 Cyclin-dependent kinase 2 Cyclin Cell cycle DNA repair DNA damage DNA Kinase CHEKl Protein
11 C is not correct. ]OJo ch ose t his.
12 The Rb protein does not promote DNA damage r epair. This function is often associated with p53, which is a protein that like Rb, promotes cell- cycle arrest in the
presence of DNA damage ( or other factors that do not constitute a favorable envir onment for successful cell r eplication ) , I n addition, it also initiates the apoptotic
13 cascade in the presence of overwhelming DNA damage that cannot be r epair ed by the ceiL
14 PS3 Protein DNA Retinoblastoma protein Cell cycle DNA repair Apoptosis DNA damage

15 D is not correct. 30/o ch ose t his.


The Rb protein does not promote histone acetylation , I n fact, the Rb-E2F complex promotes histone deacetylation on chr omatin ,
16
Acetylation Chromatin Histone Protein Histone acetylation and deacetylation Retinoblastoma protein Histone acetyltransferase
17
E is not correct. ]OJo ch ose t his.
18 Pr otein 53, not Rb protein, plays an important r egulator y r ole in apoptosis and angiogenesis of multiple types of cancer s,
19 Apoptosis Angiogenesis Protein Retinoblastoma protein PS3 Rubidium

20
• 21 Bottom Li ne :

• 22 loss of function of both copies of the RB1gene r esults in r etinoblastoma and osteosar coma , The RB1 gene codes for a protein that normally binds the E2F transcription
factor, thus preventing the cell from progr essing past the G1/ S r estr iction point in quiescent cells,
• 23 Osteosarcoma Gene Retinoblastoma Transcription factor Restriction point E2F Retinoblastoma protein Transcription (genetics) Protein GO phase
• 24

a
Lock
s
Suspend
o
End Block
Item: 20of64
QIO: 3972
~ , .1 • Mar k
~
<1
Previous
t>
Next Lab
d\ ues
•• .
Notes
~
calculator

1
FA17 p 218,2
2
Tumor suppressor Loss of function - f cancer risk; both (t"o) alleles of a tu mo r suppressor gene must be lost for
3
genes expression of disease,
4
GENE GENE PRODUCT ASSOCIATED CONDITION
5
APC egative regulator of ~-caten i n/WNT pathway Colorectal ca ncer (associated with FAP)
6
7
BRCA 1/ BRCA2 D1 A repair protein Breast, ovarian, and pancreatic cancer
8 CDKN2A pl6, blocks G 1 -+ S phase lelanoma, pancreatic cancer
9 DCC D CC- D eleted in C olon C ancer Colon cancer
10
DPC4/SMAD4 DPC - D eleted in Pancreatic C ancer Pancreatic cancer
11
MEN1 M en in lEl
12
13
NF1 eurofibromin (Ras GTPase activating protein) N eurofibromatosis type l
14 NF2 Merlin (schwannomin) protein N eurofibromatosis type 2
15 PTEN Tyrosine phosphatase ofPIP3 (eg, protein kinase Breast cancer, prostate cancer, endometrial
16 8 [AKT ] activation) cancer
17 Rb Inhibits EZF; blocks G 1 - S phase Retinoblastoma, osteosarcoma
18
TP53 p53, activates p21, blocks G 1 - S phase lost human cancers, Li-Fraumeni syndrome
19
(multiple malignancies at early age, aka, SBLA
20 cancer syndrome: Sarcoma, Breas t, Leukemia,
• 21 Adrenal gland)
• 22
TSC1 1-lamartin protein Tuberous sclerosis
• 23
TSC2 Tuberin protein Tuberous sclerosis
• 24

a
Lock
s
Suspend
o
End Block
Item: 20of64
QIO: 3972
~ , .1 • Mar k
~
<1
Previous
t>
Next Lab
d\ ues
•.

Notes
~
calculator

1 FA17 p 42 ,1
2
Cell cycle phases Checkpoints control transitions between phases of cell cycle. Th is process is regulated by cyclins,
3
cyclin-dependent kinases (CDKs), and tumor suppressors. M phase (shortest phase of cell cycle)
4
includes mitosis (prophase, prometaphase, metaphase, anaphase, telophase) and cytokinesis
5 (cyl·oplasm splits in two). C 1 and C 0 arc of variable duration.
6
REGULATION OF CEll CYCLE
7
Cyclin-dependent Constitutive and inactive.
8
kinases
9
10 Cyclins Regulatory proteins that control cell cycle
11
events; phase specific; activate CDKs.
12 Cyclin-CDK complexes Phosphorylate other proteins to coordinate
13 cell cycle progression; must be activated and
14
inactivated at appropriate times for cell cycle
15
to progress.
16 Tumor suppressors p53 induces p2l, which inhibits CDKs F
G-o G
-+ hypophosphorylation (activation) of Rb
17
18
- inh ibition of G 1-S progression. Mutations
in tumor suppressor genes can result in
19 Rb. p53 modulate
unrestrained cell division (eg, Li-Fraumen i G1 restriction point
20
syndrome).
• 21
CELL TYPES
• 22
Permanent Remain in G 0, regenerate from stem cells. Neurons, skeletal and cardiac muscle, RBCs.
• 23
• 24 Stable lauiescentl Enter G. from G, when stimulated. Heoatocvtes. lvmohocvtes.
a
Lock
s
Suspend
o
End Block
Item: 21 of 64 ~ 1 • M k -<:J 1>- Jil ~· !:';-~
QIO: 1195 ..L ar Pre v ious Next Lab~lues Notes Calcula t o r
A A
12 A baby girl is born with an RBC abnormality. The physician explains to her mother that the hemoglobin in her daughter's RBCs is folded abnormally,
13 and subsequently aggregates in a way that distorts the shape of the cells. He further explains that these misshapen cells have a harder time making
It through the smallest vessels in the body and often get stuck, which deprives downstream tissues of oxygen.
14
15 Which of the following types of mutation is the cause of this patient's disease?
16
:
17 A. Framesh1ft mutation

18 B. Insertion mutation
19
c. Nonsense mutation
20
• 21 o. Point mutation

• 22 E. Silent mutation

• 23
• 24
• 25
• 26
• 27
• 28
• 29
• 30
• 31
• 32

a
Lock
s
Suspend
8
End Bl ock
Item: 21 of 64 ~. I • M k <:] t> al ~· ~
QIO: 1195 .l. ar Previous Next Lab 'Vfll ues Notes Calculator

12
The co rrect a nswer is D. 8 2% cho se this.
13 Sickle cell anemia is an autosomal recessive disea se caused by a point mutation in the sixth position of the 13 chain of hemoglobin. This mutation causes a
14 polar amino acid, glutamat e, to be replaced with a hydrophobic amino acid, valine. Absence of a polar amino acid in the 13-6 position lea ds to abnormal
hydrophobic interactions with neighboring hemoglobin molecules and subsequent aggregation . Evolutionary biologists theorize that sickle cell anemia has
15 persisted because het erozygous carriers of the trait have some protection against malaria.
Point mutation Sickle-cell disease Amino acid Valine Malaria Heterozygous Autosomal recessive Hemoglobin Dominance (genetics) Glutamic acid Hydrophobe
16
Mutation Recessive Anemia Autosome Hydrophobic effect Evolutionary biology
17
A is no t co rrect. 7% cho se this.
18 A frameshift mutation is an insertion or deletion of nucleotides of any number of nucleotides not divisible by 3. It results in a misrea ding of all codons
downstream during messenger RNA translation . Tay-Sa chs disea se is an example of a four-nucleotide insertion frameshift mutation, which results in a
19
defective hexosaminidase.
20 Tay-Sachs disease Frameshift mutation Messenger RNA Mutation Nucleotide Translational frameshift Genetic code Translation (biology) RNA

21 B is no t co rrect. 3 % cho se this.


• 22 An insertion mutation is an addition of one or more nucleotides to the DNA. This results in a frameshift when the number of nucleotides in the inserted
element is not a multiple of 3. One example of an insertion mutation resulting in a frameshift is Tay-Sa chs disea se, in which the most common va riant is
• 23 due to a four base pair insertion in the 13-hexosaminidase gene .
Tay-Sachs disease Base pair Gene Insertion (genetics) Mutation Nucleotide Frameshift mutation Translational frameshift DNA
• 24
c is no t co rrect. 6 % cho se this •
• 25
A nonsense mutation occurs when a point mutation results in an early stop codon . This type of mutation results in truncat ed proteins that usually are
• 26 nonfunctional and eventually are degraded by the cell. 13-Thalassemias can be caused by nonsense mutations in the 13-globin gene, which decrea se the
overall amount of functional hemoglobin 13 chains.
• 27 Nonsense mutation Point mutation Stop codon Gene Hemoglobin Mutation Genetic code Protein

• 28 E is no t co rrect. 2% cho se this.


• 29 A silent mutation occurs when a point mutation does not change the amino acid sequence of the protein. The protein is not altered, and thus there is no
associat ed disea se stat e. Often the point mutation is in the third position of the codon .
o30 Silent mutation Point mutation Amino acid Protein Mutation Peptide sequence Genetic code
•31
• 32 Rntt-n rn 1 i n .ca.•

6
lock
s
Suspend
0
End Block
Item: 21 of 64 ~. I • M k <:] t> al ~· ~
QIO: 1195 .l. ar Previous Next Lab 'Vfll ues Notes Calculator

12
Bottom Line:
13
Sickle cell anemia is caused by a point mutation . Glutamat e is switched for a valine in the j3 -6 position of the hemoglobin molecule.
14 Sickle -cell disease Point mutation Valine Hemoglobin Anemia Glutamic acid Mutation Sickle Molecule

15
16
17 Iiii I;fi IJ I•J for year:[ 2017 ..
FI RST AI D FA CTS
18
19 FA17 p 36.1

20 Mutations in DNA Severity of damage: silent << missense < nonsense < frameshift.
21 For point (silent, missense, and nonsense) mutations:
• 22 • Transit ion -purine to purine (eg, A to G) or prrimidine to pyrimidine (eg, C toT).
• 23
• Transversion- purine to pyrimidine (eg, A to T) or pyrimid ine to purine (eg, C to C) .
• 24 Silent Nucleotide substitution but codes for same
(synonymous) amino acid; often base change
• 25
in 3rd position of codon (tRNA wobble).
• 26
Missense Nucleotide substitution resulting in changed Sickle cell disease (substitution of glutamic acid
• 27
amino acid (called conservative if new amino with valine).
• 28 acid is similar in chemical structure).
• 29 Nonsense Nucleotide substitution resulting in early stop Stop the nonsense!
o30 codon (UAG, UAA, UGA). Usually results in
• 31 nonfunctional protein .
• 32 Frameshift Deletion or insertion of a number of nucleotides Duchenne muscular dystrophy, Tay-Sachs

6
lock
s
Suspend
0
End Block
Item: 21 of 64 ~. I • M k <:] t> al ~· ~
QIO: 1195 .l. ar Previous Next Lab 'Vfll ues Notes Calculator

• •
12 FA17 p400.1
13 Intrinsic hemolytic anemia
14 DESCRIPTION FINDINGS

15 Hereditary Extravascular hemolysis due to defect in Splenomegaly, aplastic crisis (parvovirus Bl9
spherocytosis proteins interacting with RBC membrane in fection).
16
skeleton and plasma membrane (eg, ankyrin, Labs: osmotic fragility test ® . lormal to
17
band 3, protein 4.2, spectrin). lostly l MC V with abundance of cells.
18 autosomal dominant inheritance. Treatment: splenectomy.
19 Results in small, round RBCs with less surface
20 area and no central pallor (t MC I !C)
- premature removal by spleen.
21
• 22
G6PD deficiency Most common enzymatic disorder of RBCs. Back pain, hemoglobinuria a few days after
Causes extravascular and intravascular oxidant stress.
• 23
hemolysis. X-linked recessive. Labs: blood smear shows RBCs with Heinz
• 24 Defect in C6PD ... l glutath ione ... t RBC bodies and bite cells.
• 25 susceptibility to oxidant stress. Hemolytic "Stress makes me eat bites of fava beans with
• 26 anemia following oxidant stress (eg, sulfa Heinz ketchup."
• 27 drugs, antimalarials, infections, fava beans).
• 28 Pyruvate kinase Autosomal recessive pyruvate kinase defect Hemolytic anemia in a newborn.
deficiency - l ATP - rigid RBCs - extravascular
• 29
hemolysis. Increases levels of 2,3-BPG
o30 - l hemoglobin affinity for 0 2.
• 31
Paroxysmal nocturnal t complement-mediated intravascular RBC Associated with aplastic anemia.
• 32 hemoqlobinuria lvsis (imoaired svnthesis of GPI anchor Triad: Coombs 8 hemolvtic anemia. •

6
lock
s
Suspend
0
End Block
11
12
expansion from t erythropoiesis (also seen in Treatment: hydroxyurea (t HbF), hydration.
13
thalassemias).
14
HbC disease Glutamic acid- to-lyC ine (I) sine) mutation in Patients with ll bSC (1 of each mutant gene) ha, c
15
~-gl obi n. Causes extravascular hemolysis. mi lder disease than HbSS patients.
16
Blood smear in homozygotes: hemoglobin
17 C ry~t a ls inside RBCs, target cells.
18
19 FA17 p 77.3
20 Amino acids Only L-amino acids arc found in proteins.
21 Essential C lucogenic: methionine (\let), histidine (I Ii\), I met hi\ ,·a Ientine, she is so sweet (glucogenic).
• 22 valine (Val). All essential amino acids need to be supplied in
• 23 Clucogenic/ketogenic: isoleucine (lie), the diet.
• 24 phenylalanine (Phe), threonine (Thr),
tryptophan (Trp).
• 25
Ketogenic: leucine (Leu), lysine (l.ys).
• 26
Acidic Aspartic acid (Asp) and glutamic acid (C iu).
• 27
Negatively charged at body pH.
• 28
Basic l listidine (I lis), lysine ( L)~). arginine (Arg). II i ~ I)~
(lies) are basic.
• 29 Arg is most basic. Arg and His are required during periods of
• 30 His has no charge at body pH . growth. Arg and Lys are t in histones, which
• 31 bind negati\'ely charged DNA .
• 32

a
Lock
s
Suspend
8
End Block
Item: 22 of 64 ~ 1 • M k -<:J 1>- Jil ~· !:';-~
QIO: 2858 ..L ar Pre v ious Next Lab~lues Notes Calcula t o r

12
A

A child Is born with a flat facial profile, prominent epicanthal folds, duodenal atresia, and an atrial septal defect. His parents are advised that he very
likely will suffer from some degree of intellectual disability and has an Increased risk for developing acute leukemia and Alzheimer disease at a young
IA•A] A

13
age.
14
15 At which of the following steps of cell division did this child's defect most likely occur?
16
:
17 A. Anaphase I

18 B. Metaphase I
19
c. Prophase I
20
21
o. Telophase I

• 22 E. Telophase II

• 23
• 24
• 25
• 26
• 27
• 28
• 29
• 30
• 31
• 32

a
Lock
s
Suspend
8
End Bl ock
Item: 22 of 64 ~ 1 • M k -<:J 1>- Jil ~· !:';-~
QIO: 2858 ..L ar Prev ious Next Lab~lues Notes Calculat o r

A A
12 Th e co rrect a nswer i s A. 46% ch ose this.
This child has Down syndrome, or trisomy 21, most likely due to nondisjunction of homologous chromosomes during anaphase I or II of meiosis
13 (Illustrated in the diagram). During anaphase I and II, a microtubule spindle separates homologous pairs and sister chromatids to opposite poles of the
14 dividing cell. Down syndrome results when the spindle unequally separates chromosome 21, creating a final gamete with two copies of chromosome 21
(during fertilization, one more chromosome 21 is added, resulting in trisomy). Other possible causes include Robertsonian translocation (a translocation
15 between chromosome 21 and the long arm of a second acrocentric chromosome, generally chromosomes 14 or 22) and Down mosaicism. Although the
risk of having an infant with Down syndrom e increases with advancing maternal age, the majority of infants with Down syndrome are born to younger
16 mothers, as overall more infants are born t o younger mothers.
17 Gamete Rubt!rtso 1an translocation Centromere Down syndrome Microtubule NondiSJUI ction Me1osis Anaphase Mosaic (genetics) Trisomy
Homologous chromosome Chromosome Chromosome 21 (human) Homology (biology) Fertilisation Chromatid Sister chromatids Spindle apparatus
18
Chromosomal translocation Advanced materna l a ge
19
20 polarbodyl

21
meiosis 2
22
. 23
. 24
• 25
• 26
• 27 trisomy

• 28
. 29
• 30

. 31
. 32

a
Lock
s
Suspend
8
End Block
14 I mage courtesy of w. Peissner, Wikimedia Commons

15 B is no t co rrect. 27% cho se this.


16 Met aphase I of meiosis is the step during which the homologous pairs line up along the central met aphase plat e of the dividing cell, preceding anaphase.
Nondisjunction does not occur during met aphase; this problem usually occurs during the actual separation of chromosomes during anaphase.
17 Nondisjunction Meiosis Anaphase Metaphase I Metaphase Spindle apparatus Homology (biology) Chromosome

18 c is no t co rrect. 1 2% cho se this.


19 Prophase I is the initial stage of meiosis when the replicat ed homologous pairs join to form a t etrad. Crossover; or recombination of genetic mat erial
between alleles, is most likely to occur here and usually does not result in trisomy.
20 Meiosis Genetic recombination Prophase Prophase I Trisomy Homology (biology) Allele Genome DNA

21 D is no t co rrect. 8 % cho se this.


22 Telophase I is the first physical division of the cell, with ea ch daughter cell containing one homologous pair. A defect in t elophase I would affect all
chromosomes, not just chromosome 2 1.
• 23 Telophase Homologous chromosome Chromosome Cell division Homology (biology) Chromosome 21 (human)
• 24 E is no t co rrect. 7% cho se this.
• 25 Telophase II is the second physical division of the cell, with ea ch daughter cell containing one sister chromatid . A defect in t elophase II would affect all
chromosomes, not just chromosome 2 1.
• 26 Telophase Chromatid Chromosome Cell division Sister chromatids Chromosome 21 (human)
• 27

• 28
Bo tto m Line:
• 29 A flat facial profile, prominent epicanthal folds, duodenal atresia, and atrial septal defect are characteristic of Down syndrome, or trisomy 2 1. This
o30 condition is most likely due to nondisjunction of homologous chromosomes during anaphase I or II of meiosis.
Atrial septal defect Down syndrome Nondisjunction Meiosis Anaphase Duodenal atresia Trisomy Chromosome Homologous chromosome Epicanthic fold Atresia
• 31 Homology (biology) Duodenum

• 32

6
lock
s
Suspend
0
End Block
Item: 22 of 64 ~ 1 • M k -<:J 1>- Jil ~· !:';-~
QIO: 2858 ..L ar Pre v ious Next Lab~lues Notes Calcula t o r
A A
12 FA17 p 59. 1

13 Autosomal trisomies
14 Down syndrome Findings: intellectual d isab i l i t~, fl at facies, Incidence 1:700.
15 (trisomy 21 ) prominent epicanthal folds, single palmar Drinking age (21).
crease, gap between 1st 2 toes, duodenal lost common ,·iable chromosomal disorder and
16
atresia, Hirschsprung disease, congenital heart most conunon cause of genetic intellectual
17
disease (eg, atriO\·entricular septal defect), disability.
18 Brushfield spots. Associated with early-onset First-trimester ultrasound commonly shows
19 Alzheimer disease (chromosome 21 codes for t nuchal translucency and hypoplastic nasal
20 amyloid precursor protein) and t risk of ALL bone; ! serum PAPP-A, t free ~-hCC.
and rL. Second-trimester quad screen sho\\'S
21
95% of cases due to meiotic nond isjunction ! a -fetoprotein, t ~-hCC, ! estriol,
22
(t with advanced maternal age; from 1:1500 in t inh ibin A.
• 23 women< 20 to 1:25 in \\ Omen > 45 years old).
• 24 4% of cases due to unbalanced Robertson ian
• 25 translocation, most typically between
• 26
chromosomes 14 and 21. 1% of cases due
to mosaicism (no association with maternal
• 27
nondisjunction; postfertilization mitotic error).
• 28
Edwards syndrome Find ings: PRIKCE Edward- Prominent Incidence 1:8000.
• 29
(trisomy 18) occiput, Rocker-bottom feet, Intellectual Election age (18).
• 30 disability, l':ondisjunction, Clenched fists 2nd most common autosomal trisomy resulting
• 31 (\\ ith O\'erlapping fin gers), low-set Ears, in ]i,e birth (most common is Do'' n S) ndrome).
micrognathia (small jaw), congenital heart P . PP-A and free ~-hCC are ! in first trimester.
• 32
• . ..... --
a
Lock
s
Suspend
8
End Bl ock
Item: 22 of 64 ~ 1 • M k -<:J 1>- Jil ~· !:';-~
..L Lab~lues
QIO: 2858

12
A
ar Pre v ious Next Notes
. .
Calcula t o r
A

13
14
15
16
c) c) c :>
Meiosis II
17
c' ~
v
18
19
20
A A A
) "-" Nondisjunction

21
22
Ill Ill I
nt1 n+l n-1
I
n- 1
Gametes
II II I Ill n n-1 ntl
n
I I I
. 23 I
Trisomy Monosomy Normal Monosomy Trisomy li!
. 24
• 25
FA17 p60.2
• 26
Robertson ian Chromosomal translocation that commonly involves chromosome pairs 13, 14, 15, 21, and 22.
• 27
translocation One of the most common types of I ranslocalion. Occurs when the long arms of 2 acroccnl ric
• 28 chromosomes (chromosomes with cenlromeres near thei r ends) fuse at the centromere and the
. 29 2 short arms are lost Balanced translocalions normally do not cause any abnormal phenotype.
• 30 Unbalanced translocations can result in miscarriage, stillbirth, and chromosomal imbalance (eg,
. 31 Down syndrome, Palau syndrome).
. 32

a
Lock
s
Suspend
8
End Bl ock
Item: 23 of 64 ~ 1 • M k -<:J 1>- Jil ~· !:';-~
QIO: 3353 ..L ar Pre v ious Next Lab~lues Notes Calcula t o r
A A
12 A genetics counselor is consulted by a newlywed couple for a preconception evaluation. The husband is concerned because his father passed away In
13 his 40s. Genetic analysis of a specific part of one of the husband's chromosomes reveals the following composition: 16.7% adenine, 16.7% thymine,
33.3% guanine, and 33.3% cytosine .
14
15 Which of the following diseases best fits the description in this scenario?
16
:
17 A. Cystic fibrosis

18 B. Duchenne's mu scular dystrophy


19
c. Fragile X syndrome
20
D. Huntington's disease
21
22 E. Tay-Sachs disease

. 23
. 24
• 25
• 26
• 27
• 28
. 29
• 30
. 31
. 32

a
Lock
s
Suspend
8
End Bl ock
Item: 23 of 64 ~ 1 • M k -<:J 1>- Jil ~· !:';-~
QIO: 3353 ..L ar Pre v ious Next Lab~lues Notes Calcula t o r
A A
12 Th e co rrect a nswer i s D. 600/o chose this.
The disease is Huntington's. other examples of trinucleotide repeat disorders Include Friedrich's ataxia, myotonic dystrophy, and fragile X syndrome. In
13
each of these diseases, a single gene is interrupted by a series of three-base-pa ir repeating sequences. I n Huntington's disease, this trinucleotide repeat
14 Is CAG, so the codons in the involved gene on chromosome 4 would be 5'-CAG CAG CAG ...-3' and on the complementary strand would be 3'-GTC GTC
GTC ...-5'. Thus, there are 2 G, 2 c, 1 A, and 1 T codon per trinucleotide repeat when considering both strands of DNA, resulting in the nucleotide
15 proportions seen in the stem. These inserted elements result in cellular dysfunction by differing mechanisms. Due to errors in DNA replication caused by
the repeating sequence, there is a tendency for each subsequent generation to inherit a longer series of repeats. As t he number of these trinucleotide
16 repeats Increases, the severity of disease increases and age at onset decreases (anticipation).
17 Huntington s d sease Fragi e X syndrome Myotonic dystrophy Fr;edreich's ata .a - n1 ~reotide repeat disorder Gene DNA replication Ataxia Genetic code Nucleotide
Chromosome c: romosome 4 human) DNA
18
A is not correct. 6% chose this.
19
Cystic fibrosis is an autosomal recessive disease caused by a defective CFTR gene on chromosome 7. The most common mutation is a deletion of three
20 base pairs corresponding to the amino acid phenylalanine, at position 508. Autosomal recessive disorders do not exhibit anticipation.
Cystic fib osis Gene Amino acid Phenylalanine Autosomal recessive Dominance {genetics) Chromosome Mutation Autosome
21
Cystic fibrosis transmembrane conductance regulator Fibrosis Base pair Recessive
22
B is not co rrect. 7% chose t his.
23
Duchenne's muscular dystrophy results from a frameshift null mutation of the dystrophin gene, which leads to accelerated muscle breakdown. There Is no
• 24 genetic anticipation associated with this disorder.
Oystrophin Ouchenne muscular dystrophy Gene Muscular dystrophy Mutation Frameshift mutation Anticipation (genetics) Translational frameshift Null allele
• 25
C is not co rrect . 230/o ch ose this .
• 26
Fragile X syndrome is an X-linked syndrome of mental disability attributed to the expansion of a CGG triplet repeat in the FMRl gene. It would show a
• 27 paucity of adenine and thymine in the above analysis. The classic features are an elongated face, protruding ears, macro-orchidism, and problems In
social or Intellectual development. The inheritance pattern is X-linked dominant, and it Is not possible for the husband to have inherited this condition
• 28 from his father, from whom he received his Y, not X, chromosome .
Fragile X syndrome Gene FMRl Thymine Sex linkage Chromosome Adenine Multiple birth Triplet state X-linked dominant inheritance
• 29
E i s n ot correct. 40/o ch ose this .
• 30
Tay-Sachs disease is an autosomal recessive disease caused by a defect In the gene encoding the lysosomal enzyme hexosaminidase A. It does not
• 31 Involve trinucleotide repeat expansion and t hus does not lead to anticipation.
Trinucleotide repeat disorder Gene Enzyme Autosomal recessive Dom1nance (genetiC~) Trinucleotide repeat expansion Autosome Lysosome
• 32

a
Lock
s
Suspend
8
End Bl ock
Item: 23 of 64 ~. I • M k <:] t> al ~· ~
QIO: 3353 .l. ar Previous Next lab 'Vfl1 ues Notes Calculator

12
Bottom Line:
13
Huntington's disease is a CAG trinucleotide repeat disorder that can be expressed at an ea rlier age in the patient than in the patient's parent, a
14 characteristic called anticipation.
Trinucleotide repeat disorder Huntington' s disease
15
16
17
I ill ;fi 1!1 I•J for year:[ 2017 ..
18 FI RST AI D FA CTS

19
FA11 p 58.2
20
21 Trinucleotide repeat Huntington disease, myotonic dys trophy, Try (tri nucleotide) hunting for my fragile cage-
expansion diseases fragile X syndrome, and Friedreieh ataxia. free eggs (X).
22
lay show genetic anticipation (disease severity
23
f and age of onset ! in successive generations).
• 24 Huntington disease = (CAG), C auda te has ! ACh and GABA
• 25 lyoton ic dystrophy= (CTG)11 C ataracts, Toupee (early balding in men),
• 26 G onadal atrophy
• 27
Fragile X syndrome = (CGG)11 C hin (protruding), G ian t G onads
Friedreich ataxia = (GAA)11 Ataxic GAAit
• 28

• 29
FA17p56.1
o30
Autosomal dominant Achondroplasia, autosomal dominan t polycystic kidney disease, fam ilial adenomatous polyposis,
• 31
diseases familial hyperc holesterolemia, hereditary hemorrhagic telangiectasia, hereditary spherocytosis,
• 32 Hnntineton disease_ Li-Franmeni svndrome_Marfan svndrome_ mnltinle endocrine neonlasias_

6
lock
s
Suspend
0
End Block
Item: 24 of 64 ~ 1 • M k -<:J 1>- Jil ~· !:';-~
QIO: 5148 ..L ar Pre v ious Next Lab~lues Not es Calcula t o r

12
A

A 45-year-old man goes to his primary care physician complaining of recent headaches. They rarely persist for more than an hour, but are sporadic
and often accompanied by sudden sweating and palpitations. "Doc," he says, "It feels like my heart is racing ." Acetaminophen provides minimal relief
IA•A] A

13
from the pain. He is afebrile and his blood pressure is 128/70 mm Hg. His physical examination is unremarkable with the exception of moist palms
14 and pale skin. A urinalysis is notable for elevated metanephrines.

15
This patient's most likely diagnosis is often associated with which of the following conditions?
16
17 :
A. Hashimoto thyroiditis
18
19 B. Insullnoma

20 c. Parathyroid hyperplasia
21
o. Prolactinoma
22
E. Zollinger-EIIison syndrome
23
. 24
• 25
• 26
• 27
• 28
. 29
• 30
. 31
. 32

a
Lock
s
Suspend
8
End Bl ock
Item: 24 of 64 ~. I • M k <:] t> al ~· ~
QIO: 5148 .l. ar Previous Next lab 'Vfl1ues Notes Calculator

12 The co rrect a nswer is c. 5 8% cho se this.


Intermittent hea daches, swea ting, and palpitations in an otherwise healthy man are suggestive of a pheochromocytoma, a cat echolamine-secreting
13 tumor most commonly found in the adrenal glands. Episodes are limited in duration, but blood pressure during these events can rea ch dangerously high
14 levels. High urinary cat echolamines, met anephrine, and vanillylmandelic acid confirm the diagnosis. Pheochromocytomas are associat ed with primary
hyperparathyroidism, resulting from parathyroid hyperplasia, as part of the multiple endocrine neoplasia (MEN ) type II syndrome. This syndrome also
15 includes medullary carcinomas of the thyroid .
Pheochromocytoma Vanillylmandelic acid Metanephrine Hyperparathyroidism Multiple endocrine neoplasia Palpitations Endocrine system
16
Primary hyperparathyroidism Adrenal gland Perspiration Catecholamine Parathyroid hormone Parathyroid gland Blood pressure Neoplasm Thyroid Hyperplasia
17
A is no t co rrect. 9 % cho se this.
18
Hashimoto thyroiditis is an autoimmune disorder resulting in hypothyroidism. Antimicrosomal and antithyroglobulin antibodies are present and diagnostic.
19 During the initial phase of glandular injury, a transient stat e of hyperthyroidism may result from cellular rupture. Hashimoto thyroiditis is not associat ed
with pheochromocytomas.
20 Hashimoto' s thyroiditis Hyperthyroidism Hypothyroidism Autoimmune disease Thyroiditis Antibody Autoimmunity Pheochromocytoma Gland

21 B is no t co rrect. 10 % cho se this.


22 Insulinomas are associat ed with parathyroid and pituitary tumors as part of the multiple endocrine neoplasia type I complex. Insulinomas are insulin -
secreting tumors of the pancrea s that produce symptoms due to hypoglycemia. They are not associat ed with pheochromocytomas.
23 Hypoglycemia Pancreas Multiple endocrine neoplasia Endocrine system Parathyroid gland Pituitary adenoma Pituitary gland Pheochromocytoma Neoplasm
24 D is no t co rrect. 10 % cho se this.
• 25 Prolacintomas are associat ed with pancrea tic tumors (Zollinger-EIIison syndrome, insulinomas, and VI Pomas) and parathyroid tumors as part of multiple
endocrine neoplasia type I syndrome. Prolactinomas cause excessive secretion of prolactin, resulting in secondary amenorrhea in women and
• 26 galactorrhea . Prolactinomas are not associat ed with pheochromocytomas.
Galactorrhea Prolactin Zollinger-EIIison syndrome Amenorrhoea Endocrine system Pheochromocytoma Multiple endocrine neoplasia Neoplasm Parathyroid gland
• 27
E is no t co rrect. 13 % cho se this •
• 28
Zollinger- EIIison syndrome is associat ed with parathyroid and pituitary tumors as part of the multiple endocrine neoplasia type I complex. Zollinger- EIIison
• 29 syndrome is caused by a gastrin-secreting tumor (gastrinoma ), resulting in recurrent upper gastrointestinal ulcers that are resistant to medical trea tment.
Zollinger- EIIison syndrome is not associat ed with pheochromocytomas.
o30
Gastrinoma Zollinger-EIIison syndrome Multiple endocrine neoplasia Endocrine system Parathyroid gland Neoplasm Pituitary adenoma Pituitary gland
•31 Pheochromocytoma Human gastrointestinal tract Gastrointestinal tract Peptic ulcer

• 32

6
lock
s
Suspend
0
End Block
Item: 24 of 64 ~ 1 • M k -<:J 1>- Jil ~· !:';-~
QIO: 5148 ..L ar Prev ious Next Lab~lues Not es Calculat o r

A A
12 Bottom Line:
13 MEN type II syndrome includes medullary carcinoma of the thyroid, pheochromocytoma, and primary hyperparathyroidism. This primary
hyperparathyroidism typically stems from a parathyroid hyperplasia.
14
Pheochromocytoma Medullary thyroid cancer Parathyroid hormone Pr;mary hyperparathyroidism Thyroid Hyperplasia Hyperparathyroidism Parathyroid gland
15 Carcinoma

16
17
18
I@ l;fi 1;1i•J for yea r: 2017 •
FIRST AIO FACTS
19
20 FA17 p 326.1

21 Pheochromocytoma

22 ETIOLOGY !\lost common tumor of the <~drcnalmcdulla in Rule of 10'!.:


23
adults rJ. Derived from chromaffin cel ls (arise 10% malignant
from neural crest). 10% bi lateral
24
May be associated with germ line mutations (eg, 10% extra-adrenal (eg, bladder wall, organ of
• 25
NF-I, VHL, RET [MR ' ZA, ZB]). Zuckerkand l)
• 26 10% calcify
• 27 10% kids
• 28
• 29
• 30 SYMPTOMS l\losllumors secrete epinephrine, Episodic hypcradrenergic symptoms (5 P'!t):
• 31 norepinephrine, and dopamine, which can Pressure (t BP)
cause episodic hypertension . Pain (headache)
• 32
• . ..
'

a
Lock
s
Suspend
8
End Bl ock
Item: 24 of 64 ~ 1 • M k -<:J 1>- Jil ~· !:';-~
QIO: 5148 ..L ar Pre v ious Next Lab~lues Not es Calcula t o r

12
A
FA17 p 339.1 A

13 Multiple endocrine All MEN syndromes have autosoma I dominant inheritance.


14 neoplasias "All l\lEN are dominant" (or so they think).
SUBTYPE CHARACTERISTICS COMMENTS
15
MEN 1 Pituitary tumors (prolactin or G i l)
16
Pancreatic endocrine tumors-Zollinger- Pituitary
17
EIIison srndrome, insulinomas, I Pomas,
18 glucagonomas (rare) Pancreas
19 Parathyroid adenomas
20 Associated with mutation of i\IEt 1 (menin, a
21
tumor suppressor, chromosome II )

22 MEN2A Parathyroid hyperplasia


Parathyroids
~ led ullary thyroid carcinom<1-ncoplasm of
23
parafollicular or C cells; secretes ca lcitonin;
24 prophylactic thyroidectomy required
• 25 Pheochromocytoma (secretes catecholamines) Thyroid
Associated with mutation in RET (codes for (medullary carcinoma)
• 26

• 27 receptor tyrosine kinase) in cells of neura I crest


ongm Pheochromocytomas
• 28
• 29
MEN2B ledullary thyroid carcinoma
Pheochromocytoma
• 30
lucosal neuromas (oral/intestinal
Mucosal neuromas
• 31 ganglioneuromatosis)
• 32
• Associated with marfanoid habitus; mutation in

a
Lock
s
Suspend
8
End Bl ock
Item: 25 of 64
QIO: 3373
~ , .1 •
~
Mar k <1
Previous
t>
Next Lab
d\ ues
•.

Notes
~
calculator

• Within a normal seminiferous tubule obtained from a testis biopsy of a healthy man, there are several layer s of germ cells at various stages of development, Such a
12
biopsy specimen is shown in the image ,
13
14
15
• •, 1
16
17

18 ~-
19
20
21

22
23
..
24
• 25
• 26
'~
-
... .
.:
• •
'
• 27
I I •
•,.
~ ,
• 28
• 29
• ...•
~, •
• 30
• 31
.. ... '
• 32
• 33
• 34

a
Lock
s
Suspend
o
End Block
Item: 25 of 64 ~ 1 • M k -<:J 1>- Jil ~· !:';-~
QIO: 3373 ..L ar Pre v ious Next Lab~lues Notes Calcula t o r
6
Within a normal seminiferous tubule obtained from a testis biopsy of a healthy man, there are several layers of germ cells at various stages of
12
development. Such a biopsy specimen is shown in the image.
13
14
15
16
.......
••
-
17
18
19
20
21
22
23

0
24
25

0
26
0
27
0

. 29
28
• •
o30

. 31
. 32 •
a
Lock
s
Suspend
8
End Bl ock
Item: 25 of 64 ~ 1 • M k -<:J 1>- Jil ~· !:';-~
QIO: 3373 ..L ar Pre v ious Next Lab~lues Notes Calcula t o r

12
13
14
15
• '
16 •
17
18
19
20
21
22
23 The genetic complements of the most basal (peripheral) germ cell and the most apical (or central) germ cell are, respectively:
24
:
0 25 A. Nand N
• 26
B. Nand 2N
• 27
c. 2N and N
• 28
. 29 D. 2N and 2N

o30 E. 4N and 2N
. 31
. 32 •
a
Lock
s
Suspend
8
End Bl ock
Item: 25 of 64 ~. I • M k <:] t> al ~· ~
QIO: 3373 .l. ar Previous Next Lab 'Vfll ues Notes Calculator

12
The co rrect a nswer is c. 70 % cho se this.
13
Spermatogenesis proceeds in a highly organized fashion in the seminiferous tubule. As illustrat ed by the diagram here, spermatogonia (2N), supported by
14 the adjacent Sertoli cells, begin m eiosis in the most basal part of the basal compartment, dividing and moving apically into the adluminal compartment
until they become haploid spermatids (N) locat ed in the most apical part of the seminiferous tubule. From here they begin their journey away from the
15 t estis.
Seminiferous tubule Meiosis Spermatogonium Haploid Spermatogenesis Ploidy Testicle Sertoli cell Spermatid Nephron Basal (phylogenetics)
16
17 A is no t co rrect. 5 % cho se this.
Haploid spermatids (N) are locat ed more apically. Cells in the basal segment have not yet undergone m eiosis and are still diploid (2N).
18 Meiosis Haploid Diploid Spermatid Ploidy Basal (phylogenetics)

19 B is no t co rrect. 1 2 % cho se this.


20 Spermatids (N) are locat ed most apically and spermatogonium (2N) most basally, not the other way around .
Spermatogonium Spermatid
21
D is no t co rrect. 5 % cho se this.
22
Spermatogonia (2N) are diploid; they are locat ed in the most basal region of the basal compartment. However; cells in the apical region have complet ed
23 m eiosis and have a haploid chromosome complem ent (N).
Meiosis Spermatogonium Diploid Haploid Chromosome Ploidy Basal (phylogenetics)
24
E is no t co rrect. 8% cho se this.
25
Cells with a 4N complem ent would be t etraploid and are not observed in normal spermatogenesis. At the completion of m eiosis, normal spermatids are
• 26 haploid (N) not diploid (2N) .
Meiosis Haploid Spermatogenesis Diploid Tetraploid Polyploid Spermatid Ploidy
• 27

• 28
Botto m Li ne:
• 29
Male sperm develop from the outside of the tubule inward, so basal-side cells are 2 N and apical-side cells are N.
o30 Sperm Spermatozoon Nephron Tubule
•31
• 32

6
lock
s
Suspend
0
End Block
Item: 25 of 64 ~ 1 • M k -<:J 1>- Jil ~· !:';-~
QIO: 3373 ..L ar Pre v ious Next Lab~lues Notes Calcula t o r
A A

12 FA17 p601.1

13 Spermatogenesis Spermatogenesis begins at pubert y with "Con iu m" is going to be a sperm; "Zoon" is
14 spermatogonia. Full dc\'clopment ta~es 2 "Zooming" to egg.
months. O ccurs in seminiferous tubules.
15
Produces spermatids that undergo
16
spermiogenesis (loss of cytoplasmic contents,
17 gain of acrosomal cap) to form mature
18 spermatozoon.
19 N=ploidy
20 C= # of chromatids

21 Spermiogenesis
22 Spermatogootum t• spermatocyte z· spermatocyte Spermatid _ _ _ _ _ _,.. Mature spermatozoon
Diploid Diploid Haploid HaplOid Haploid
23 (2N, 2C) (2N, 4C) (IN, 2C) (IN, IC) {IN.IC)
24
Sperm
25
23 Acrosome' \ r Tatl
• 26 single
23 sex• X) Head ~
• 27 ( sister
Blood-testis Nucleus-
• 28 chromatids
barrier (sex• X.Xl Neck -
23
• 29 stngle
o3 0
46 ~ (sex• X) Middle _ /
piece
single sister
• 31 chromo- chromatids
somes
• 32 • (sex= X-Yl !sex= X-XJ
y.y Note: lmoaired tail mobiUtv can lead to

Lock
a s
Suspend
8
End Bl ock
Item: 25 of 64 ~. I • M k <:] t> al ~· ~
QIO: 3373 .l. ar Previous Next Lab 'Vfll ues Notes Calculator

• •
12 FA17 p 594.1
13 Seminiferou s tubules
14 CELL FUNCTION LOCATION/ NOTES

15 Spermatogonia Maintain germ cell pool and produce 1° Line seminiferous tubules rJ
(germ cells) spennatocytes.
16
17 Sertoli cells Secrete inhibin B .... inhibit FSH. Line seminiferous tubules
(non-germ cells) Secrete androgen-binding protein - maintain Convert testosterone and androstenedione to
18
local levels of testosterone. estrogens via aromatase
19
Produce MIF. Sertoli cells Support Sperm Synthesis
20 Tight junctions between adjacent Sertoli cells Homolog of female granulosa cells
21 form blood-testis barrier .... isolate gametes
22 from autoimmune attack.
Support and nourish developing spermatozoa.
23
Regulate spermatogenesis.
24
Temperature sensitive; I sperm production and t temperature seen in varicocele,
25 I inhibin B with t temperature. cryptorchidism
• 26 Leydig cells Secrete testosterone in the presence of LI I; Interstitium
• 27 (endocrine cells) testosterone production unaffected by Homolog of female theca interna cells
• 28 temperature.
• 29 HYPOTHALAMUS

o30 ~
GnRH
• 31
• 32 • ~ ~ •

6
lock
s
Suspend
0
End Block
Item: 25 of 64 ~ 1 • M k -<:J 1>- Jil ~· !:';-~
QIO: 3373 ..L ar Pre v ious Next Lab~lues Notes Calcula t o r
A A

12 l inhibin B with t temperature. cryptorchidism


13 Leydig cells Secrete testosterone in the presence of Ll I; lnlcrslilium
14 (endocrine cells) testosterone production unaffected by llomolog of female theca intern a cells
temperature.
15
16 HYPOTHALAMUS

17 ~
GnRH

~
18
19
J
20
21
22
23

J\
24
25 le)'dtg a ll

• 26 Testosterone

• 27
• 28
• 29
Sertoll<ell
o3 0 nudous

• 31
• 32 SPERMATOGENESIS

a
Lock
s
Suspend
8
End Bl ock
Item: 26 of 64 ~ 1 • M k -<:J 1>- Jil ~· !:';-~
QIO: 5 0 52 ..L ar Pre v ious Next Lab~lues Notes Calcula t o r

12
A

A 2-year-old boy is being bathed by his mother when she notices a palpable left flank mass on her son. He was born at full term via a normal
spontaneous vaginal delivery without complications and has no significant past medical history. His mother t akes him to the pediatrician, who suspects
IA•A] A

13
the boy may have Wilms' tumor.
14
15 On which chromosome is the genetic abnormality located that is most likely responsible for the boy's condition?
16 :
17 A. Chromosome 4

18 B. Chromosome s
19
c. Chromosome 11
20
21
o. Chromosome 16

22 E. Chromosome 22

23
24
25
• 26
• 27
• 28
. 29
o30
. 31
. 32 •
a
Lock
s
Suspend
8
End Bl ock
Item: 26 of 64 ~ 1 • M k -<:J 1>- Jil ~· !:';-~
QIO: 5 0 52 ..L ar Prev ious Next Lab~lues Notes Calculat o r

A A

12
The co rrect an sw er i s c . 560/o chose this.
13 Wilms' tumor is the most common renal malignancy of childhood . It usually presents in children 2-4 years old as a palpable flank mass with
14 hemihypertrophy. Wilms' tumor is most commonly associated with a deletion of the tumor suppressor gene WTl on chromosome 11, but deletions on
chromosomes 1p and 16 have also been seen. Wilms' tumor may be part of the WAGR complex that consists of Wilms' t umor, Aniridia, Genitourinary
15 malformation, and mental-motor Retardation.
W' ms' tumo WTl Tumor suppressor gene Gene Hemihypertrophy Chromosome 11 (human} Malognancy Neoplasm cancer Kidney Chromosome
16
Deletion ge 1etics
17
A i s not correct. 14% chose thi s.
18 Huntington's disease, an inherited autosomal dominant disorder that results In progressive dementia, choreiform movements, depression, personality
19 changes, and ultimately, death, is caused by excessive repeats of a trinucleotide on chromosome 4.
Huntington's d1sease Dominance (genetics) Chromosome 4 (human) Dementia Cho e Autosome Chromosome Major depressive disorder Depress1on 1mood 1
20
B is n ot correct. 1 30/o chose this.
21
Familial adenomatous polyposis (FAP) is an inherited condition in which the colon becomes covered with adenomatous polyps soon after puberty. Unless
22 the colon Is resected, 100% of patients with this condition will progress to cancer. FAP is due to a deletion of the APC tumor suppressor gene located on
chromosome 5.
23 Familial adenomatous polyposis Tumor suppressor gene Gene Chromosome 5 (human) Neoplasm Colon (anatomy) Adenomatous polyposis coli Chromosome

24 Polyp (medicine) Cancer Colorectal polyp Adenomatous polyps

25 0 is not co rrect. 11 Ofo chose this.


Adult polycystic kidney disease is an autosomal dominant disorder in which the kidneys develop multiple large cysts, ca using massive enlargement . It
26
often presents in adults 30-60 yea rs old wit h pain, hematuria, hypertension, and progressive renal failure. This condit ion is associat ed with polycystic liver
• 27 disease, berry aneurysms, and mitral va lve prolapse. Ninet y percent of cases are caused by a mutation in t he APKDl gene on chromosome 16 .
Hematuria Polycystic kidney disease Gene Dominance (genetics) Polycystic liver disease Mitral valve prolapse Mitral valve Hypertension Kidney Liver Autosome
• 28
Kidney disease Mutation Chromosome Aneurysm Cyst Liver disease Chromosome 16 (human)
. 29
E i s n ot correct. 60fo chose this.
o30 Neurofibromatosis type 2, which presents wit h bilat eral acoustic neuromas, meningiomas, and juvenile cat ara cts, is ca used by a mutation in the NF2 gene
on chromosome 22 .
. 31
NeurofibromatOSIS type II Neurofibromatosis Gene Chromosome 22 (human) Meningioma Vestibular schwannoma Mutation Merlin (protein) Chromosome cataract
. 32 •
a
Lock
s
Suspend
8
End Block
Item: 26 of 64 ~. I • M k <:] t> al ~· ~
QIO: 5052 .l. ar Previous Next lab 'Vfl1 ues Notes Calculator

12 Bottom Line:
13 Wilms' tumor is associated with a deletion of the tumor suppressor gene WTl on chromosome 11 and m ay be part of the WAGR complex, which
consists of W ilms' tumor; Aniridia, Genitourinary m alformation, and m ental-motor Reta rdation.
14
Wilms' tumor WTl Tumor suppressor gene Gene Chromosome 11 (human) Neoplasm Chromosome
15
16
17 Iiii I;fi IJ I•J for year:[ 2017 ..
FI RST AID FAC T S
18
19
FA17 p 569.1
20
Nephroblastoma lost common renal malignancy of early childhood (ages 2-4). Contains embryon ic glomerular
21
(Wilms tumor) structures. Presents with large, palpable, unilateral Rank mass rJ and/or hematuria.
22 "Loss of function" mutations of tumor suppressor genes WTl or WT2 on chromosome l L
23 lay be a part of several syndromes:
24 • WACR complex: Wilms tumor, Aniridia (absence of iris), Genitourinary malformations, mental
25
Retardation/intellectual disability (WTl deletion)
• Denys-Drash: Wilms tumor, early-onset nephrotic syndrome, male pseudohermaphroditism
26
(WTI mutation)
• 27
• Beckwith-Wicdemann: Wilms tumor, macroglossia, orga nomcg~lly, hcmihyperplasia (WT2
• 28 mutation)
• 29
o30 FA17p56.1

• 31 Autosomal dominant Achondroplasia, autosomal dominant polycystic kidney disease, fam ilial adenomatous polyposis,
• 32 diseases familial hypercholesterolemia, hereditary hemorrhagic telangiectasia, hereditary spherocytosis,

6
lock
s
Suspend
0
End Block
Item: 27 of 64 ~ 1 • M k -<:J 1>- Jil ~· !:';-~
QIO: 2647 ..L ar Pre v ious Next Lab~lues Notes Calcula t o r
A A

12 A 22-year-old man is diagnosed with medullary thyroid carcinoma, and a comprehensive metabolic panel is significant for hypercalcemia . He notes
that he has episodes of dizziness accompanied by sweating and feeling llghtheaded. He says he remembers his mother having her thyroid gland
13
removed when he was a young child but cannot remember the exact reason.
14
15 These findings suggest a possible mutation in which of the following genes?
16 :
17 A. brat
18 B. erb-82
19
C. MENl
20
21
o. ras
22 E. ret
23
24
25
26
• 27
• 28
. 29
o30
. 31
. 32 •
a
Lock
s
Suspend
8
End Bl ock
Item: 27 of 64 ~ 1 • M k -<:J 1>- Jil ~· !:';-~
QIO: 2647 ..L ar Prev ious Next Lab~lues Notes Calculat or
A A

12 Th e co rrect a nswer i s e. 530/o chose this .


13 The characteristics described in the question are found in patients with multiple endocrine neoplasia (M EN ) type 2A, also known as Sipple syndrome. MEN -
2A Is associated with parathyroid hyperplasia or tumor, leading to hypercalcemia, medullary carcinoma of the thyroid, and pheochromocytoma (which
14 commonly causes elevated plasma catecholamine levels). The patient's episodes of lightheadedness and sweating m ay be attributed to the elevated
15 catecholamine level found in pheochromocytoma. The MEN syndromes follow an autosomal dominant pattern of inheritance, and MEN-2A and MEN-28 are
both linked to distinct mutations in the ret proto-oncogene. MEN-26 is characterized by pheochromocytoma, medullary carcinoma of the thyroid,
16 marfanold hab1tus, and ganglioneuromas in the mouth or gastrointestinal tract.
Catecholam •e P oeochromocytoma Medullary thyroid cancer Marfanood Hypercalcaemia Dominance \genetics) Multiple endocrine neoplasia Parathyroid hoo mone
17
Endocrine system Thyroid Parathyroid gland Gastrointestinal tract Multiple endocrine neoplasia type 2 Proto-oncogene Blood plasma Human gastrointestinal tract
18 Oncogene Neoplasm Hyperplasia Autosome Marfan syndrome Perspiration Carcinoma Morphology (biology) Ughtheadedness
19 A is not correct. 5% chose this.
20 Mutations In braf have been associated with papillary carcinoma of the thyroid. This is the most common thyroid malignancy, and it has been associated
with a remote history of radiation exposure. It is not associated with any of the MENs.
21 Malignancy Carcinoma Thyroid Cancer Papillary thyroid cancer Mutation Acute radiation syndrome
22 B is not co rrect. 5% c h ose this.
23 The erb-62 oncogene is associated with breast, ovarian, and gastric carcinomas.
Oncogene Carcinoma
24
C is not co rrect . 260/o c hose this .
25
Mutations In the MENl tumor suppressor gene are found in patients with MEN type 1, also known as Wermer syndrome. This disea se is characterized by
26 hyperplasia or tumor of the " 3 Ps" : Pituitary, Parathyroid, and Pancreas. It may manifest its pancreatic component by the Zollinger-EIIison syndrome,
hyperlnsullnemla, or pancreatic cholera . Although this patient does have hypercalcemia that m ay be attributed to hyperplasia or tumor of the parathyroid
27 gland, he does not have any pancreatic symptoms. MEN-1 syndrome Is not associated with medullary thyroid carcinoma .
• 28 Parathyroid gland Zollinger-EIIison syndrome Multiple endocrine neoplasia type 1 Hypercalcaemia Tumor suppressor gene MENl Gene Medullary thyroid cancer
Thyroid cancer Carcinoma Neoplasm Thyroid Hyperplasia Mutation Thyroid neoplasm
. 29
D i s n ot co rrect. 11Ofo ch ose this .
o30
The ras oncogene mutation is associated with follicular thyroid carcinoma, not medullary thyroid carcinoma .
. 31 Medullary thyroid cancer Follicular thyroid cancer Oncogene Thyroid Thyroid neoplasm Mutation Thyroid cancer carcinoma
. 32 •
a
Lock
s
Suspend
8
End Bl ock
Item: 27 of 64 ~. , . M k <:] t> al ~· ~
QIO: 2647 .l. ar Previous Next lab 'Vfl1 ues Notes Calculator

12 Bottom Line:
13 MEN -2A (Sipple syndrome) is an autosomal dominant predisposition to medullary carcinoma of the thyroid, pheochromocytoma, and parathyroid
hyperplasia/tumors that arises from a mutation in the ret proto-oncogene.
14
Pheochromocytoma Medullary thyroid cancer Dominance (genetics) Proto -oncogene Multiple endocrine neoplasia type 2 Carcinoma Autosome Thyroid Mutation
15 Parathyroid gland Oncogene

16
17
18
lijj ;fi IJ l•l for year:l 2017 ..
FI RST AID FAC T S

19
20 FA17p339.1

21 Multiple endocrine All MEN syndromes have autosomal dominant inhe ritance.
22 neoplasias "All MEN are dominant" (or so they think).
23 SUBTYPE CHARACTERISTICS COMMENTS

24 MEN 1 P ituitary tumors (prolactin or GI-l)


Pancreatic endocrine tumors- Zollinger-
25 Pituitary
Ellison syndrome, insulinomas, VIPomas,
26
glucagonomas (rare) Pancreas
27 Parathyroid adenomas
• 28 Associated with mutation of MENJ (men in, a
• 29 tumor suppressor, chromosome 11)
o30 MEN2A Parathyroid hyperplasia
Parathyroids
• 31
ledullary thyroid carcinoma-neoplasm of
parafollicular or C cells; secretes calcitonin;
• 32 ---.-. ..... k . .t ........ ,.; .............. _ .... :.J ........... ..........~ •. -.-..-•• : _ .... ..)

6
lock
s
Suspend
0
End Block
Item: 27 of 64 ~ 1 • M k -<:J 1>- Jil ~· !:';-~
QIO: 2647 ..L ar Pre v ious Next Lab~lues Notes Calcula t o r
A A

12 FA17 p 330.2
13
Thyroid cancer Typically diagnosed with fine needle aspiration; treated with thyroidectomy. Complications of
14 surgery include hoarseness (due to recurrent Jar) ngeal nerve damage), hypocalcemia (due to
15 removal of parathyroid glands), and transection of recurrent and superior laryngeal nen·es (during
16 ligation of inferior thyroid artef) and superior laryngeal artery, respectively).
17 Papillary carcinoma lost common, excellent prognosis. Empty-appearing nuclei with central clearing (M Orphan
18 \ nnic" eyes) rJ, psam \I om a bodies, nuclear groo\·es (Papi and \lorna adopted Orphan \nnic).
t risk with RET and BRAF mutations, childhood irradiation.
19
20
21
22
23
24
Follicular carcinoma Good prognosis. Invades thyroid capsule and vascu lature (unlike follicular adenoma), uniform
25
foll icles; hematogenous spread is common. ssociated with RAS mutation.
26
Medullary carcinoma From parafollicular "C cells"; produces calcitonin, sheets of cells in an amyloid stroma (stains with
27 Congo red [l]). Associated with ME ' 2A and 2B (RET mutations).
• 28
. 29
o30
. 31
. 32 •
a
Lock
s
Suspend
8
End Bl ock
Item: 28 of 64
QIO: 3932
~
..L
1 • Ma r k -<:J
Prev ious
I>
Next Lab
fJ
lues
£!1}>'

Notes
!!":-~
Cal culat o r

A A

12 A particular mRNA has the codon 5'-AGG, which codes for the amino acid arginine.
13
14 Which nucleotide sequence is the corresponding tRNA anticodon?

15 :
A. S'·UCC
16
17 8 . S'·CCT

18 C. S'·CCU
19
0. S'·TCC
20
21
22
23
24
25
26
27
• 28
. 29
o30
. 31
. 32 •
a
Lock
s
Suspend
8
End Bl ock
Item: 28 of 64 ~. I • M k <:] t> al ~· ~
QIO: 3932 .l. ar Previous Next Lab 'Vfll ues Notes Calculator

12
13
The co rrect a nswer is c. 59 % cho se this.
14 The anticodon portion of the tRNA molecule binds to the complementary codon on the mRNA molecule. Transfer RNA characteristically contains many
modified bases as a result of posttranscriptional modifications. Ea ch tRNA molecule is specific for only one amino acid; thus, a family of tRNAs is necessary
15 to carry all 20 possible amino acids. Codons and anticodons adhere to the same nucleotide rules that are true in DN A and RN A in general. Adenine pairs
16 with thymine (in DN A) or uracil (in RN A), wherea s cytosine pairs with guanine. The codon and anticodon are lined up in an anti-parallel manner. In this
case, the anticodon is 5' -AGG . Of the answer choices, only 5' -CCU is the corresponding codon .
17 Transfer RNA Anticodon Amino acid Thymine Uracil Cytosine Guanine Adenine Genetic code Molecule DNA RNA Nucleotide Messenger RNA Anticodons

18 A is no t co rrect. 1 6 % cho se this.


19 This answer choice is incorrect because it does not match up with the anticodon in an anti-parallel manner.
Anticodon Transfer RNA
20
B is no t co rrect. 1 6 % cho se this.
21 This answer choice is incorrect because thymine is not present in RN A.
22 Thymine RNA

23 D is no t co rrect. 9 % cho se this.


This answer choice is incorrect for two rea sons. First, the codon contains thymine, which should be uracil because this is an RN A molecule. Second, the
24 anticodon does not match with the codon in an anti-parallel manner.
25 Anticodon Thymine Uracil Transfer RNA Genetic code RNA Molecule

26
27 Botto m Li ne:

28 Codons are three -nucleotide RN A molecules on mRN A that code for a particular amino acid . Codons are paired to corresponding anticodons on tRN A
according to the A-U and C-G pairing rule in an anti-parallel manner.
• 29 Amino acid Transfer RNA Messenger RNA Genetic code RNA Anticodons

o30
•31
• 32

6
lock
s
Suspend
0
End Block
Item: 28 of 64 ~ 1 • M k -<:J 1>- Jil ~· !:';-~
QIO: 3932 ..L ar Pre v ious Next Lab~lues Notes Calcula t o r
A A

12 FA17 p 40.1
13 tRNA
14 Structure 75-90 nucleotides, zo structure, cloverleaf form, anticodon end is opposite 3' aminoacyl end. II
15 tRl As, both eukaryotic and prokar)otic, have CC at 3' end along with a high percentage of
16
chemically modified bases. The amino acid is covalently bound to the 3' end of the tRl\A. CC \
Can Carry \ mino acids.
17
T-arm: contains the T'f'C (riboth} midinc, pseudouridine, cytidine) sequence necessary for tR~A­
18 ribosome binding. T-arm Tethers tR A molecule to ribosome.
19 0-arm: contains dihydrouridine residues necessary for tR~A recognition by the correct aminoacyl-
20 tRJ A svnthetase.
. D-arm Detects theIR A b\ aminoacd-tRNA "'
svnthetase.
~ ~

21
Acceptor stem: the 5'-CC -3' is the am ino acid acceptor site.

22
Charging Aminoacyl-tR1 A synthetase (I per amino acid; "matchmaker''; uses ATP) scrutinizes amino acid
before and after it binds to tR1 A. If incorrect, bond is hydrolyzed. The amino acid-tR A bond
23
has energy for formation of peptide bond. A mischarged tR 1 reads usual codon but inserts
24 wrong amino acid.
25 Aminoacyl-tR A synthetase and binding of charged IRN to the codon are responsible for
26 accuracy of am ino acid selection.
27 Structure Charging Pairing
(aminoacylation) (codon-anticodon)
28
• 29
Aminoacid, Amino acid,
0
o3 0
Acceptor stem{OH- i 3. 0
' c. 3'
c
' c. 3'
c
s· s· s·
• 31 Am1noacyHRNA
etase
• 32 • T·arm ~ --
synU
....... .. ... . .""'""" IF2
..
a
Lock
s
Suspend
8
End Bl ock
Item: 28 of 64 ~ 1 • Ma rk -<:J 1>- Jil ~· !:';-~
QIO: 3932 ..L Pre v ious Next Lab~lues Notes Calcula t o r

FA17p 41.1 •
12
13
Protein synthesis

14 Initiation Initiated b) GTP hydrolysis; initiation factors Eukarrotcs: 40S + 60S .... 80S (Even).
(eukaryotic IFs) help assemble the -+0 PrOkaryotes: 30S + 50S .... 70S (Odd).
15
ribosomal subunit with the initiator tR t\ Synthesis occurs from 1 -terminus to
16
and are released when the mR A and the C-terminus.
17 ribosomal 60S subunit assemble" ith the
AT P-tRN \ cti,·ation (charging).
18 complex.
GTP-tfu'-:A Gripping and Going places
19 Elongat ion 1. Aminoacyl-tR 'A binds to A site (except for (translocation).
20 initiator methionine)
2. rR A ("ribozyme") catalyzes peptide bond Think of"going \Pi<:":
21
format ion, transfers grO\\ ing polypeptide to A site = incoming Aminoacyl-tRNA.
22 P site = accommodates growing Peptide.
amino acid in A site
23 3. Ribosome advances 3 nucleot ides to\\ard 3' I~ site= holds Empty tR 'A as it Exits.
24 end of ml A, moving pept idyl t RNA to P
Eukaryotic
25 site (translocation) ribosome
26 Termination Stop codon is recognized by release factor, 3'
27 and completed polypeptide is released from

ribosome.
28
• 29
o3 0
• 31
D
• 32 •
a
Lock
s
Suspend
8
End Bl ock
Item: 29 of 64 ~ 1 • M k -<:J 1>- Jil ~· !:';-~
QIO: 1212 ..L ar Pre v ious Next Lab~lues Notes Calcula t o r
A A

12 The drawing below represents a newly transcribed strand of RNA being processed Into a mature mRNA.
13
14
15
16 X
AUG~ ...._ _~
:...:::::
5' .....
17
18
19
20
21 Which of the following is a most likely sequel of a nonsilent mutation in the component of X of the RNA?

22 :
A. A different functional version of the protein
23
24 B. Abnormality or loss of protein function

25 c. Abnormally high rate of protein translation


26
D. Inability to translate the mRNA to a protein
27
E. Rapid degradation of the mRNA
28
. 29
o30
. 31
. 32 •
a
Lock
s
Suspend
8
End Bl ock
Item: 29 of 64 ~. I • M k <:] t> al ~· ~
QIO: 1212 .l. ar Previous Next lab 'Vfl1ues Notes Calculator

12 The co rrect a nswer is B. 57 % cho se this.


13 X is an exon within the RNA. Exons are spliced together to form the mature mRNA transcript. A nonsilent mutation within one the mRNA exons is likely to
cause an abnormal amino acid sequence in the translat ed protein, lea ding to abnormality or loss of function . Alternatively, it may also lea d to a truncat ed
14 protein that is not functional and is degraded by the cellular proteosomal complex.
Amino acid Exon Messenger RNA Protein Mutation Proteasome Mature messenger RNA RNA Peptide sequence
15
A is no t co rrect. 1 9 % cho se this.
16
A mutation affecting splicing sites in the premature RN A may lea d to a different splice variant of the protein. Splicing sites are not parts of the exonic
17 RN A sequences pointed to in X.
Alternative splicing Protein RNA Mutation RNA splicing
18
19
c is no t co rrect. 5 % cho se this.
Commonly, mutations in promotor regions of the gene that usually occur in the intronic sequences or at 5 ' and 3 ' untranslat ed ends of the mRN A are
20 likely to affect the regulation of mRN A translation, lea ding to higher or lower levels of the protein. In this case, a mutation of X affects the exonic region
and is less likely to affect the level of protein translation .
21 Gene Messenger RNA Protein Mutation Protein biosynthesis Translation (biology)
22 D is no t co rrect. 1 3 % cho se this.
23 Mutations within the exons of an RN A are more likely to cause abnormality in the amino acid sequence of the protein product and are less likely to cause
abnormalities in the translation process of the mature mRN A.
24 Amino acid Protein Exon RNA Peptide sequence Messenger RNA Mutation Mature messenger RNA
25 E is no t co rrect. 6 % cho se this.
26 The lifetime of a mature mRN A is a key aspect of its regulation and is usually det ermined by the poly-A t ail at the 3 ' end of the mRN A (in addition to
alternative mechanisms). A mutation in the exon of the mRN A is less likely to directly affect the lifetime of a mature mRN A.
27 Exon Polyadenylation Mutation Messenger RNA Directionality (molecular biology) Mature messenger RNA
28
29
Botto m Li ne:
o30
The diagram is showing the process of splicing, in which intervening segments of noncoding DN A, called an intron, are t aken out to form a continuous
•31 strand of coding DN A (exons) . These rea ctions are cat alyzed by a group of small nuclear ribonucleoproteins called a spliceosome .
Spliceosome Noncoding DNA Intron Exon SnRNP DNA RNA splicing Ribonucleoprotein
• 32

6
lock
s
Suspend
0
End Block
Item: 29 of 64 ~ 1 • M k -<:J 1>- Jil ~· !:';-~
QIO: 1212 ..L ar Pre v ious Next Lab~lues Notes Calcula t o r
A A

12 FA17 p 38.4

13 RNA processing Initial transcript is called heterogeneous nuclear mRI is transported out of the nucleus into the
14 (eukaryotes) RNA (hnR lA). hnR lA is then modified and C) tosol, \\here it is translated.

15 becomes mR 'A. mR quality control occurs at cytoplasmic


T he following processes occur in the nucleus: processing bodies (P-bodies), \\'hich contain
16 Cap Coding
s· Capping of 5' end (addition of cxonucleases, decapping enzymes, and
17
Gppp <b/ ~ 7-meth}lguanosine cap) microRl'!As; mRNAs may be stored in P-bodies
18 3' I : "' Polyadenylation of 3' end (= 200 A's) for futme translation.
H().AAAAA
...___...
19 Ta1l G!l Splicing out of introns Poly-A polymerase does not require a template.
20 Capped, tailed, and spliced transcript is called \UAAt\ = polyadenylation signal.
mRNA.
21
22
FA17 p 39.2
23
24 lntrons vs exons Exons contain the actual genetic information lntrons are intervening sequences and stay
coding for protein. in the nucleus, whereas exons e-..it and are
25
Introns are intervening noncoding segments of expressed.
26
DNA. Abnormal splicing variants are impl icated in
27 Different exons are frequently combined by oncogenesis and many genetic disorders (eg,
28 alternative splicing to produce a larger number ~-tha l assemia).
29 of unique proteins.
o3 0 5
_ onl
.,_Ex Exon 2 Exon 3 Exon4 _ _:::; on~5:;,.__......;:Ex;;;;o;,;;n~6.....,. 3•
Ex:::;
DNA 3'-- -------------~s·
• 31
• 32 Tramcription

a
Lock
s
Suspend
8
End Bl ock
Item: 29 of 64 ~ 1 • Ma rk -<:J 1>- Jil ~· !:';-~
QIO: 1212 ..L Pre v ious Next Lab ~lues Notes Calcula t o r

f . . ;.
6

12 hnRNA

I
S' 3'
2 5 6
~,.,~
1 3
13
Spllong
14
( )
15
16
17
,,,_
mRNA 5'
1 2 4

1
5 6
3' S'
1 3

1
5 6
3' 5'
1 3 4

1
5 6
3'

18
Proteins 6
19 4
2
20
21
FA17 p 39.1
22
lariat
23
Splicing of pre-mRNA 0 Primary transcript combines "it h small A

'
nuclear ribonucleoproteins (snRi Ps) and (
24
other proteins to form spliceosome. snRNPs-
25
f) Lariat-shaped (looped) intermediate is ~ <PrGU-A-AG~
26 0
generated.
27 E) Lariat is released to precisely remove intron
28 and join 2 exons. f)
29 Antibodies to spliccosomal snR Ps (anti-
Smith antibodies) are highly specific for
o30
SLE. Anti-U l R1 P antibodies arc highly
. 31
associated with mixed connective tissue li!l
. 32 • disease (~JCTD).
a
Lock
s
Suspend
8
End Bl ock
Item: 30 of 64 ~ 1 • M k -<:J 1>- Jil ~· !:';-~
QIO: 4867 ..L ar Pre v ious Next Lab~lues Notes Calcula t o r
A A

12 A 22-year-old woman of European descent presents with progressive exercise Intolerance. She has a brother who has epilepsy and hearing loss. Her
mother has hearing loss and poor night vision . Other members of her family also have a history of epilepsy. In the pedigree shown, the affected 22-
13
year-old Is indicated by an arrow.
14
15
16
17
18
19
20
21
22 Based on the clinical description and pedigree, which inheritance pattern Is most consistent?
23 :
24 A. Autosomal dominant

25 8. Autosomal recessive
26
c. Mitochondrial inheritance
27
D. X-llnked dominant
28
29 E. X-linked recessive

o30
. 31
. 32 •
a
Lock
s
Suspend
8
End Bl ock
Item:30of64 ~. , . M k <:] t> al ~· ~
QIO: 4867 .l. ar Previous Next lab 'Vfl1 ues Notes Calculator

12
13 The co rrect a nswer is c. 73% cho se this.
14 This pedigree demonstrat es mitochondrial inheritance. Disea ses inherited in this fashion are transmitted only through the mother. Thus, all offspring of
affected women may show signs of disea se. Het eroplasmy refers to the presence of both normal and mutat ed mitochondrial DNA, thus resulting in
15 variable expression of mitochondrial inherited disea ses, as in our patient, who has myoclonic epilepsy with ragged red fibers (MERRF), which manifests
with a combination of progressive myoclonic epilepsy, poor night vision, short stature, hearing loss, and lactic acidosis.
16
Heteroplasmy Epilepsy lactic acidosis Mitochondrial DNA Myoclonus Mitochondrion Mitochondrial disease Hearing loss Progressive myoclonus epilepsy
17 Short stature Expressivity (genetics) Night vision DNA Genetic disorder

18 A is no t co rrect. 13 % cho se this.


19 Autosomal dominant disea ses affect multiple generations. Both males and females can be affected. Although there is a small possibility that this is
autosomal dominant inheritance, the fact that the brother's three children are all unaffected makes this answer choice less likely.
20 Dominance (genetics) Autosome
21 B is no t co rrect. 2% cho se this.
22 In autosomal recessive disea se, 2 5 % of the offspring from two carrier parents will be affected with the disea se. Typically, only one generation is affected,
and both males and females can be affected. This pedigree is unlikely to represent autosomal recessive inheritance since every person marrying into the
23 family would have to be a carrier.
Recessive Autosome Dominance (genetics) Autosomal recessive Purebred Breed registry
24
25 D is no t co rrect. 9 % cho se this.
X-linked dominant disea ses are transmitted through both parents, and either male or female offspring of the affected mother may be affected. In
26 contrast, all female offspring of the affected father are affected. This pedigree does not represent X-linked dominant inheritance, since the healthy
daughters in the third generation did not inherit their affected father's disea se.
27
X-linked dominant inheritance Sex linkage
28
E is no t co rrect. 3 % cho se this.
29 In X-linked recessive inheritance, males are more likely to be affected. Affected males inherit a mutation from their unaffected carrier mothers. This
pedigree is unlikely to represent X-linked recessive inheritance, since there are multiple affected females.
30
X-linked recessive inheritance Recessive Dominance (genetics) Mutation Sex linkage
•31
• 32

6
lock
s
Suspend
0
End Block
Item:30of64 ~. , . M k <:] t> al ~· ~
QIO: 4867 .l. ar Previous Next lab 'Vfl1 ues Notes Calculator

12
Bottom Line:
13 Mitochondrial inheritance is characterized by m at ernal t ra nsmission. The risk to the children of an affected woman is as high as 100%. Affected m ales
14 never transmit the disease. Variable expressivity m ay be common and m ay be explained by het eroplasmy- the presence of both normal and mutant
mitochondrial DNA.
15 Heteroplasmy Mitochondrial DNA Mutant Mitochondrion DNA Expressivity (genetics)

16
17
18 I iii I;fi 1!1 I•J for year:[ 2017 "
FIRST AID FAC T S
19
20 FA17 p 55.1
21 Modes of inheritance
22 Autosomal dominant Often due to defects in structural genes. Many Often pleiotropic (multiple apparently unrelated
23 generations, both males and females are effects) and variably expressive (different
24 affected. between individuals). Family history crucial
to diagnosis. With one affected (heterozygous)
25
parent, on average, Y2 of children affected.
26

27
28

29
Autosomal recessive Often due to enzyme deficiencies. Usual ly seen Common ly more severe than dominant disorders;
in only I generation. patients often present in childhood.
30
t risk in consanguineous fam ilies .
• 31 With 2 ca rrier (heterozygous) parents, on average:
• 32 ~ of ch ildren will be affected (homozygous),

6
lock
s
Suspend
0
End Block
Item:30of64 ~. , . M k <:] t> al ~· ~
QIO: 4867 .l. ar Previous Next lab 'Vfl1 ues Notes Calculator

• •
12 FA17 p 52.2
Genetic terms
13
TERM DEFINITION EXAMPLE
14
Codominance Both alleles contribute to the phenotype of the Blood gro ups A, B, AB; a 1-antitrypsin
15
heterozygote. deficiency.
16
Variable expressivity Patients with the same genotype ha ve varying 2 patients with neurofibromatosis type 1 ( 1Fl)
17
phenotypes. may have va rying disease severity.
18
Incomplete ot all individuals with a mutant genotype BRCA.l gene mutations do not always result in
19
penetrance show the mutant phenotype. breast or ovarian cancer.
20
Pleiotropy One gene contributes to multiple phenotypic Untreated phenylketonuria (PKU) manifests with
21
effects. light skin, intellectual disability, and musty body
22 odor.
23
Anticipation Increased severity or earlie r onset of disease in Trinucleotide repeat diseases (cg, Huntington
24 succeeding generations. disease).
25
Loss of heterozygosity If a patient inherits or develops a mutation in Retinoblastoma and the "two-hit hypothesis,"
26 a tumor suppressor gene, the complementary Lynch syndrome (Hl PCC), Li-Fraumcni
27 allele must be deleted/mutated before cancer syndrome.
28 develops. This is not true of oncogenes.
29 Dominant negative Exerts a dominant effect. A heterozygote Mutation of a transcription factor in its allosteric
30 mutation produces a nonfunctional altered protein that site. onfunctioning muta nt can still bind
• 31 also prevents the normal gene product from DNA, preventing wild-type transcription factor
fu nctioning. from binding.
• 32 • •

6
lock
s
Suspend
0
End Block
Item: 31 of 64 ~ 1 • M k -<:J 1>- Jil ~· !:';-~
QIO: 4 0 18 ..L ar Pre v ious Next Lab~lues Notes Calcula t o r
A A

12 Genetic karyotypes are obtained for two siblings, and they are found to have the same deletion on a particular region of chromosome 13. No other
abnormalities are found on either child's DNA. One child has severe intellectual disability, with an IQ near 25, but the other child has only mild
13
disability, with an IQ of so.
14
15 Which of the following principles could explain the difference in IQ between these siblings?

16 :
17 A. AntiCipation

18 B. I mprinting
19
c. Loss of heterozygosity
20
21
o. Pleiotropy

22 E. Variable expressivity

23
24
25
26
27
28
29
30
. 31
. 32 •
a
Lock
s
Suspend
8
End Bl ock
Item: 31 of 64 ~. I • M k <:] t> al ~· ~
QIO: 4018 .l. ar Previous Next lab 'Vfl1ues Notes Calculator

12
The co rrect a nswer is E. 8 3 % cho se this.
13 Variable expressivity describes the phenomenon that occurs when the clinical fea tures (type and severity) of a genetic disorder vary between individuals
with the same gene alteration . The children in this vignette have the same genetic alteration with differing phenotypes. Disea ses with variable
14 expressivity include Wiskott -Aidrich syndrome and neurofibromatosis type 1.
15 Another possible explanation would be mosaicism, which is the presence of genetically distinct cell lines in the same individual. Although less likely in this
scenario, mosaicism can lea d to variability in the expressed phenotype. Disea ses such as Down syndrome can be caused by mosaicism in rare cases.
16
Down syndrome Neurofibromatosis type I Neurofibromatosis Gene Mosaic (genetics) Genetic disorder Phenotype Expressivity (genetics)
17
A is no t co rrect. 1% cho se this.
18
Anticipation describes a situation in which individuals in successive generations exhibit increa singly severe disea se symptoms and/or an earlier age of
19 onset . Disea ses that exhibit anticipation include Huntington chorea, fragile X syndrome, and myotonic dystrophy. Anticipation arises with certain
mutations involving trinucleotide repea ts, in which a grea t er number of copies of the mutant trinucleotide is passed along to the next genera tion .
20
Fragile X syndrome Myotonic dystrophy Chorea Mutant Trinucleotide repeat disorder Mutation
21
B is no t co rrect. 6 % cho se this.
22 Imprinting describes parent-of-origin effects in which differences in phenotype depend on whether the mutation is of mat ernal or pat ernal origin.
Angelman syndrome and Prader -Willi syndrome are examples of imprinting disorders.
23
Angelman syndrome Prader-Willi syndrome Phenotype Genomic imprinting Imprinting (psychology) Mutation
24
c is no t co rrect. 5 % cho se this.
25 Loss of het erozygosity refers to loss of an allele through deletion of either part of the gene or the entire gene. This event is common in the progression of
cancer. During tumorigenesis, both alleles of a tumor supressor gene need to be inactivat ed in order to allow uncontrolled cell division to occur. In
26
het erozygosity, one functional copy of a tumor suppressor is present, a cancer phenotype will not present itself. A tumor will only manifest after loss of
27 the second functional allele.
Tumor suppressor gene Gene loss of heterozygosity Carcinogenesis Phenotype Zygosity Allele Heterozygosity Cell division Neoplasm Cancer
28
D is no t co rrect. 5 % cho se this.
29 Pleiotropy occurs when one genetic defect exerts multiple, seemingly unrelat ed phenotypic effects. Pleiotropy is typical in many genetic disea ses. For
30 example, Marfan syndrome is characterized by abnormalities of the ocular; skelet al, and cardiovascular systems.
Marfan syndrome Pleiotropy Genetic disorder Disease Phenotype Circulatory system
31
• 32

6
lock
s
Suspend
0
End Block
Item: 31 of 64 ~. I • M k <:] t> al ~· ~
QIO: 4018 .l. ar Previous Next lab 'Vfl1ues Notes Calculator

12
Bottom Line:
13 Variable expressivity = the same genotype results in different phenotypes in affected individuals.
14 Genotype Expressivity (genetics) Phenotype

15
16
17
I ill ;fi 1!1 I•J for year:[ 2017 ..
FIRST AID FAC TS

18
19 FA17 p 52.2
Genetic term s
20
TERM DEFINITION EXAMPLE
21
Codominance Both alleles contribute to the phenotype of the Blood groups A, B, AB; a 1-antitrypsin
22
heterozygote. defici ency.
23
Variable expressivity Patients with the same genotype have varying 2 patients with neurofibromatosis type 1 ( JFl)
24
phenotypes. may have varying disease severity.
25
Incomplete 'ot all individua ls with a mutant genotype BRCAJ gene mutations do not always result in
26
penetrance show the mutant phenotype. breast or ovarian cancer.
27
Pleiotropy One gene contributes to multiple phenotypic Untreated phenylketonuria (PKU) manifests with
28
effects. light skin, intellectual disability, and musty body
29
odor.
30
Anticipation Increased severity or earlier onset of disease in Trinucleotide repeat diseases (cg, Huntington
31 succeeding generations. disease).
• 32 . --- _, .. _._ _- _ __ ... 'r .. . . . '• . .. 1'\ , • I 1 , 1 , I U, I •, I • 1 • ,

6
lock
s
Suspend
0
End Block
Item: 32 of 64 ~ 1 • M k -<:J 1>- Jil ~· !:';-~
QIO: 3470 ..L ar Pre v ious Next Lab~lues Notes Calcula t o r

12
A

A 49-year-old woman presents to her primary care physician for a lower abdominal pain that has worsened over the course of several weeks. ACT
scan shows a drastically enlarged spleen. The spleen is surgically removed and pathologic sections show a lymph node-like appearance with abnormal
lA• A] A

13
germinal centers. The results of immunohistochemical staining are:
14
CD20: High
15 CDlS: Low
bcl-2: High
16 c-Myc: Low
17 Tartrate-resistant acid phosphatase: Low

18
19 Which gene translocation has occurred in this cance r ce ll?
20
:
21 A. t(8;14)

22 B. t(9;22)
23
c. t(l1;14)
24
25 o. t( l1; 22)

26 E. t(l4;18)

27
28
29
30
31
. 32 •
a
Lock
s
Suspend
8
End Bl ock
Item: 32 of 64 ~ 1 • M k -<:J 1>- Jil ~· !:';-~
QIO: 3470 ..L ar Prev ious Next Lab~lues Notes Calculat or
A A

12 T h e correct a n sw er i s E. 490/o chose this .


13 This patient has follicular lymphoma, a type of non-Hodgkin lymphoma (NHL). Follicular lymphoma is
recognizable by its attempt to recapitulate lymph node architecture by forming follicles. Ninety percent of
14 follicular lymphomas have a translocation between chromosomes 14 and 18, which leads to the
overexpression of the antiapoptotic BCL -2 gene. A representative image of an lmmunohistochamically
15 stained spedmen demonstrating follicular lymphomas is shown here. ,,.
16 Non-Hodg in lymphoma Lymph node Follicular lymphoma Gene Bcl-2 Lymphoma Chromosome Lymph
Chromosomal translocation
17
18
19 -'
20
Image courtesy of Dr. Michael Bonert
21
22 A is n ot co rrect. 120/o c h ose this.
Burkitt lymphoma is a rare cancer of young adults that is associated with Epstein-Barr virus infection. Translocation between chromosome a and the heavy
23 arm of the Ig gene is highly prevalent, leading to the classic overexpresslon of c-myc.
24 Burkitt's lymphoma Epstein-Barr virus Gene Chromosome 8 (human) Cancer Chromosome Lymphoma Virus Chromosomal translocation Infection

25 8 is n o t co rrect . 160/o c h ose this .


Philadelphia chromosome is considered a hallmark for chronic m yelogenous leukemia (CML). The t(9;22) translocation generates a fused bcr-abl hybrid
26 (not to be confused with bel) that leads to a constitutively active tyrosine kinase oncoprotein. CML is one of the first human diseases to be treated with a
27 tyrosine kinase inhibitor, imatinib.
Chronic myelogenous leukemia Philadelphia chromosome Oncogene Imatinib ;yrosine·kinase inhibitor Leukemia Tyrosine kinase Tyrosine Chromosome
28 Protein kinase inhibitor Chromosomal translocation Enzyme inhibitor Philadelphia Kinase
29 C i s n o t co rrect. 150/o ch o se this .
30 Mantle cell lymphoma is a rare and aggressive type of NHL. Diagnostic features include CDS positivity and a t(11 ; 14) translocation. This translocation
leads to overexpression of cyclin D1, a known proto-oncogene.
31 Mantle eel lymphoma Cyc! n Dl Lymphoma cyclin CDS (protein) Chromosomal translocation Oncogene
32 • n it:: ... ~ . f'V"t.rro~ Q~ rhnco • hie

a
Lock
s
Suspend
8
End Block
Item:32of64 ~. , . M k <:] t> al ~· ~
QIO: 3470 .l. ar Previous Next lab 'Vfl1 ues Notes Calculator


12 I mage courtesy of Or. Michael Bonert
13
A is no t co rrect. 12% cho se this.
14
Burkitt lymphoma is a rare cancer of young adults that is associat ed with Epstein -Barr virus infection . Translocation between chromosome 8 and the hea vy
15 arm of the Ig gene is highly prevalent, lea ding to the classic overexpression of c-myc.
Burkitt' s lymphoma Epstein-Barr virus Gene Chromosome 8 (human) Cancer Chromosome lymphoma Virus Chromosomal translocation Infection
16
B is no t co rrect. 16% cho se this.
17
Philadelphia chromosome is considered a hallmark for chronic myelogenous leukemia (CML). The t(9;22) translocation generat es a fused bcr -abl hybrid
18 (not to be confused with bel) that lea ds to a constitutively active tyrosine kinase oncoprotein. CML is one of the first human disea ses to be trea t ed with a
tyrosine kinase inhibitor; imatinib .
19 Chronic myelogenous leukemia Philadelphia chromosome Oncogene Imatinib Tyrosine-kinase inhibitor leukemia Tyrosine kinase Tyrosine Chromosome
20 Protein kinase inhibitor Chromosomal translocation Enzyme inhibitor Philadelphia Kinase

21 c is no t co rrect. 15% cho se this.


22 Mantle cell lymphoma is a rare and aggressive type of NH L. Diagnostic fea tures include CDS positivity and a t(11 ; 14) translocation . This translocation
lea ds to overexpression of cyclin Dl , a known proto -oncogene.
23 Mantle cell lymphoma Cyclin Dl lymphoma Cyclin CDS (protein) Chromosomal translocation Oncogene

24 D is no t co rrect. 8 % cho se this.

25 Ewing sarcoma is an aggressive bone tumor that belongs to the group of small-round cell tumors. The differential diagnosis of this tumor group has been
highly enhanced by the use of molecular genetics. CD20 and bcl-2 levels should not be elevat ed in Ewing sarcoma. The t(11 ;22) translocation generat es a
26 fusion between the Ewing sarcoma gene with a transcription factor that behaves as an oncogene.
Gene Transcription factor Ewing' s sarcoma Bone tumor Sarcoma CD20 Bcl-2 Neoplasm Oncogene Molecular genetics Genetics Differential diagnosis
27
Chromosomal translocation Transcription (genetics) Bone
28
29
Bo tto m Line:
30
Follicular lymphoma is associat ed with t(14; 18) and BCL-2 overexpression.
31 Follicular lymphoma Bcl-2 lymphoma

32 •

6
lock
s
Suspend
0
End Block
Item: 32 of 64 ~ 1 • M k -<:J 1>- Jil ~· !:';-~
QIO: 3470 ..L ar Pre v ious Next Lab~lues Notes Calcula t o r
A A

12 FA17 p 4 08.1

13 Non-Hodgkin lymphoma
TYPE OCCURS IN GENETICS COMMENTS
14
Neoplasms of mature Bcells
15
Burkitt lymphoma Adolescents or young t(8; H)-translocation "Starry Sk)., appearance, sheets of I) mphocytes
16
adults of c-myc (8) and with interspersed "tingible body" macrophages
17
hea'')·Chain lg (14) (arrows in fJ). ssociated with EBV.
18 Ja" lesion lll in endemic fom1 in Africa; pch is
19 or abdomen in sporadic form.
20 Diffuse large B-cell Usually older adults, Alterations in Bcl-2, lost common type of non-Hodgkin lymphoma
21 lymphoma but 20% in children Bcl-6 in adults.
22 Follicular lymphoma Adults t( 14; 18) -translocation Indolent course; Bcl-2 inhibits apoptosis.
23 of heavy-chain lg ( 14) Presents with painless "waxing and waning"
and BCL-2 (18) lymphadenopathy. Follicular architecture:
24
small cleaved cells (grade l), large cells (grade
25
3), or mixture (grade 2).
26
Mantle cell lymphoma Adult males t(l l;l4)- translocalion Very aggressive, patients typically present wit h
27 ofcyclin Dl ( II ) and late-stage disease.
28 heavy-chain lg ( 14),
29 CD5+
30 Marginal zone Adults t(11 ,18) Associated with chronic inflammation (eg,
31 lymphoma Sjogren ~rndrome, chronic gastritis (.1\IALT
lymphoma]).
32 •
a
Lock
s
Suspend
8
End Bl ock
18
19 FA17 p412.1
20 Chromosomal translocations
21 TRANSLOCATION ASSOCIATED DISORDER

22 t(8;14) Burkitt lymphoma (c-myc activation)


23 t(9;22) (Philadelphia CML (BCR-A BL hybrid), ALL (less common, Philadelphia C real\IL cheese.
24 chromosome) poor prognostic factor) The Ig heavy chain genes on chromosome 14
are constitutively expressed. When other
25
genes (eg, c-myc and BCL-2) are translocated
26
next to this heavy chain gene region, they are
27 overexpressed.
28 t(ll;l4) !antic cell lymphoma (cyclin Dl activation)
29 t(l4;18) Follicular lymphoma (BCL-2 activation)
30
t(l5;17) APL (M3 type of AML) Responds to all-trans retinoic acid.
31
32

6
lock
s
Suspend
0
End Block
Item: 33 of 64 ~ 1 • M k -<:J 1>- Jil ~· !:';-~
QIO: 1219 ..L ar Pre v ious Next Lab~lues Notes Calcula t o r

13
A

A 2-year-old child has required frequent transfusions throughout his life because of anemia . A peripheral blood smear demonstrates microcytic,
hypochromic red cells with target cells and anisopoikilocytosis. After further genetic testing, the child's spleen is removed to lessen the need for
IA•A] A

14
transfusions. A segment of the suspected gene in this patient is removed and sequenced, and the corresponding RNA is experimentally reconstructed
15 (shown below), demonstrating a mutation at the 5' end of the segment, as shown below.

16
17 Normal 5' UUCGUUCCGACU 3'
18
19 Mutant 5' UUCAUUCCGACU 3'
20
21 This change is most likely to affect which of the following processes?
22
:
23 A. Capping
24
B. Hybridization
25
c. Polyadenylation
26
27 D. Splicing

28 E. Translation
29
30
31
32
. 33 •
a
Lock
s
Suspend
8
End Bl ock
Item:33of64 ~. , . M k <:] t> al ~· ~
QIO: 1219 .l. ar Previous Next Lab 'Vfll ues Notes Calculator

13 The co rrect a nswer is D. 39% cho se this.


Introns are noncoding regions of RNA that are spliced out of pre-m RNA. Almost all introns begin and end with 5'-GU------AG -3' . A mutation in one of those
14 nucleotides affects splicing. As the mutant shown in the diagram changes from the wild 5' -GU to a 5'-AU, the intron is prevented from being properly
spliced at the appropriate site. This type of mutation can cause thalassemia .
15
Intron Thalassemia Mutation Nucleotide RNA Mutant Precursor mRNA RNA splicing
16
A is no t co rrect. 1 4 % cho se this.
17 Capping of the m RNA occurs at the 5' end, as it is being transcribed from DNA into RNA. A mutation at the 5' end of an intron would not affect capping .
18 Intron Messenger RNA Mutation DNA RNA Directionality (molecular biology)

19 B is no t co rrect. 5 % cho se this.


Hybridization is a process in which single -stranded DNA base -pairs with a complementary sequence. A mutation at the 5' end of an intron would not affect
20 hybridization.
21 Intron Complementarity (molecular biology) Mutation DNA Directionality (molecular biology) Hybrid (biology) Base pair Nucleotide

22 c is no t co rrect. 7 % cho se this.


A poly(A) tail is added to the 3' end of heterogeneous nuclear RNA (hnRNA, the initial RNA transcript) after transcription . Poly(A) polymerase uses ATP as
23 a precursor for adding adenosine one molecule at a time . A mutation at the 5' end of an intron would not affect polyadenylation.
24 Intron Polyadenylation Polymerase RNA Precursor mRNA Transcription (genetics) Adenosine Adenosine triphosphate Mutation Polynucleotide adenylyltransferase
Directionality (molecular biology) Molecule
25
E is no t co rrect. 35% cho se this.
26
Introns are non-protein-coding sections of an RNA transcript . Translation of RNA into protein would not be affected by mutation in an intron.
27 Intron Protein RNA Mutation Transcription (genetics)

28
29 Botto m Li ne:
30 Splicing is a process in which introns are excised, leaving exons joined in tandem . An adenine (A) base in the middle of an intron performs a nucleophilic
attack on the 5' end of an intron (usually at the sequence GU), forming a lariat -shaped structure. Then, the hydroxyl group on the 3' end of the exon
31
performs a nucleophilic attack on the last nucleotide of the 3' end of the intron, which completely excises the intervening intron and joins the two ends
32 of the exons. Mature m RNA is composed of a gene's exons, whereas pre-m RNA contains introns that must be excised and do not encode functional units
of protein.
33 Exon Gene Intron Adenine Nucleotide Protein Hydroxyl Messenqer RNA Precursor mRNA Directionality (molecular bioloqy) Nucleophile RNA splicinq

6
lock
s
Suspend
0
End Block
Item: 33 of 64
QIO: 1219
~
..L
1 • Ma r k -<:J
Pre v ious
I>
Next Lab
fJ
lues
£!1}>'

Notes
!!":-~
Calcula t o r
A A

13 FA17 p 39.1
14 Lariat
Splicing of pre-mRNA 0 Primary transcript combines with small ____A.____
15
16
nuclear ribonucleoproteins (sn R Ps) and
other proteins to form spliceosome.
(
snRNPs-
'
17 E) Lariat-shaped (looped) intermediate is 0 ~ <PrGU-A-A~
generated.
18
E) Lariat is released to precisely remo,·e intron
19
and join 2 exons.
20 Antibodies to spliccosomal snR1 Ps (anti-
21 Smith antibodies) are highly specific for
22 SLE. Anti-UI Ri P antibodies arc highly
23
associated with mixed connective tissue
disease ( ~ ICTD).
24
25
FA1 7 p 38.4
26

27 RNA processing In iliaI transcript is ca lied heterogeneous nuclear m RNA is transported out of the nucleus into the
28
(eukaryotes) R A (hnRNA). hnRNA is then modified and cytosol, where it is translated.
becomes mRNA. mR A quality control occurs at cytoplasm ic
29
The following processes occur in the nncleus: processing bodies (P-bodies), which contain

<bf/~o~di~ng~~
:Ca:p:]gc
~
30 Capping of 5' end (add it ion of exonucleases, decapping enzymes, and

31 Gppp 7-methylguanosine cap) microRNAs; mR lAs may be stored in P-bodies
32 3' I ',J • Polyadenylation of 3' end (= 200 A's) for future translation .
HO·AAAAA
....__._. Splicing out of introns Poi)-A polymerase does not require a template.
33 • Ta1l ...
a
Lock
s
Suspend
8
End Bl ock
Item: 33 of 64 ~ 1 • M k -<:J 1>- Jil ~· !:';-~
QIO: 1219 ..L ar Pre v ious Next Lab~lues Notes Calcula t o r
A A

13
FA17 p 39.2
14
lntrons vs exons Exons contain the actual genetic information lnlrons are intervening sequences and sta)
15
coding for protein. in the nucleus, whereas exons e'it and are
16
lntrons are inter•ening noncoding segments of e' pressed.
17 D. A. Abnormal splicing variants are implicated in
18 Different exons are frequent I) combined by oncogenesis and many genetic disorders (eg,
19 altemati,·e splicing to produce a larger number ~ -thalassemia).

20
of unique proteins.

21 Exonl Exon2 Exon3 Exon4 ExonS Exon6


5' 3'
DNA 3' 5'
22

23 1 Transcnption
1
24 hnRNA s·

l
3'
2 5 6
flte<M~~ "'<mg
1
25
Spllong
26
( ~
27 mRNA S' 3' 5' 3' s· 3'
1 2 4 5 6 1 3 5 6 1 3 4 5 6

j!""'""'"
28
29 1 1 1
30
Proteins
31
32
33 •
a
Lock
s
Suspend
8
End Bl ock
Item: 1 of 1 ~ 1 • M k -<:J 1>- Jil ~· !:';-~
QIO: 120 4 ..L ar Pre v ious Next Lab~lues Notes Calcula t o r

•1 •
A laboratory discovers a heat-sensitive strain of Escherichia coli. When Incubated above 37°C (98.6°F), the bacteria die. It is determined that the
heat-sensitive strain of E. coli is no longer able to form continuous strands of DNA from Okazaki fragments above 37°C (98.6°F).

These findings are most likely due to a mutation in which of the following proteins?

:
A. Hellcase

B. Ligase

c. Primase

o. RNase H

E. Topoisomerase

a
Lock
s
Suspend
8
End Bl ock
Item:34of64 ~. , . M k <:] t> al ~· ~
QIO: 1204 .l. ar Previous Next lab 'Vfl1 ues Notes Calculator

26
27 The co rrect a nswer is B. 69% cho se this.
DN A ligase is an enzyme that binds two polynucleotide chains together. It joins Okazaki fragments together to form a continuous section of DN A on the
28 lagging strand.
DNA ligase Enzyme Okazaki fragments DNA replication DNA ligase Polynucleotide Replication fork lagging strand
29
30 A is no t co rrect. 5 % cho se this.
Helicase is a eukaryotic enzyme that unwinds double -stranded DNA.
31 Enzyme Helicase DNA Eukaryote
32 c is no t co rrect. 16 % cho se this.
33 Primase is an RNA polymerase that makes RNA primers.
Primase RNA Primer (molecular biology) Polymerase
34
D is no t co rrect. 5 % cho se this.
o35
RNa se H is an enzyme that digests RNA primers .
• 36 Ribonuclease H Enzyme Ribonuclease Primer (molecular biology) RNA

• 37 E is no t co rrect. 5 % cho se this.

• 38 Topoisomerase is responsible for regulation of the unwinding process to prevent excessive supercoiling during DNA replication .
DNA replication Topoisomerase DNA supercoil DNA
o39
• 40
Botto m Li ne:
•41
Ligase is an enzyme that joins Okazaki fragments on the lagging strand to form a continuous strand .
• 42 Enzyme Okazaki fragments DNA replication lagging strand ligase Replication fork

• 43

• 44

• 45 I ill ;fi 1!1 I•J f o r yea r:[ 2017 ..


FI RST AID FA CTS
• 46

6
lock
s
Suspend
0
End Block
Item: 34 of 64 ~ 1 • M k -<:J 1>- Jil ~· !:';-~
QIO: 120 4 ..L ar Pre v ious Next Lab~lues Notes Calcula t o r
A A
26 FA17 p 35.1
27 DNA replication Eukaryotic DNA replication is more complex lh<1n the prokaryotic process but uses many
28 enZ)111es analogous to those listed below. In both prok<lryotes and eukaryotes, DNA replication is
29 semiconsen -ative, invok es both continuous and discontinuous (Okazaki fragment) S)11thesis, and
occurs in the )' - 3' direction.
30
Origin of Particular consensus sequence of base pair Kl ~richsequences (such as TATA box regions)
31
replication in genome where 0 1'\A replication begins. are found in promoters and origins of
32
Ylay be single (prokaryotcs) or multiple replication.
33 (eukarrotes).
34 Replication fork I] Y-shaped region along Dt A template \\'here
o35 leading and lagging strands are synthesized.
• 36 Helicase Un\\'inds D, template at replica! ion fork.
• 37 Single-stranded Pre,·ent strands from rcanncaling.
• 38 binding proteins l!l
o39 DNA Create a single- or double-stranded break in the lrinotecan/topotecan inhibit eukaryotic
0 40 topoisomerases hel ix to add or remove supercoils. topoisomerase I.
0
41 Etoposide/teniposide inhibit eukaryotie
topoisomerase II.
• 42
Fluoroquinolones in hi bit prokaryotic lopoisomerase
• 43
ll (Ot A gyrasc) and topoisomerase IV.
0 44
Primase !\lakes an R 'A primer on \\'hich Dt A
0 45 polymerase Ill can initiate replication .
• 46 DNA oolvmerase Ill ret ProkarYotes onk Elonl!ates leadine strand
• OJ oolvmerase III has ;' - 3' svnthesis and
a
Lock
s
Suspend
8
End Bl ock
Item: 34 of 64 ~ 1 • M k -<:J 1>- Jil ~· !:';-~
QIO: 120 4 ..L ar Pre v ious Next Lab~lues Notes Calcula t o r

26 •
27
Telomerase Eukaryotes only. An R A-dependent DNA Often dysregulated in cancer cells, allowing
polymerase that adds D 'A to 3' ends of unlimited replication.
28
chromosomes to avoid loss of genetic material
29 with e,·ery duplication.
30
31
32 rJ
Ong n of repi~Ubon
33
34

o35
• 36
ea
• 37
• 38

o39
[!I
0 40 DNA polymerase

0
41
• 42
• 43 FA17 p 38.3
0 44 RNA polymerases
0 45 Eukaryotes R lA polymerase I makes rR 'A (most I, II, and III are numbered in the same order
• 46 numerous Rl\A, rampant). that their products are used in protein

a
Lock
s
Suspend
8
End Bl ock
Item: 35 of 64 ~ 1 • M k -<:J 1>- Jil ~· !:';-~
QIO: 1192 ..L ar Pre v ious Next Lab~lues Notes Calcula t o r
A A
26 A researcher is investigating the DNA of a sample of bacteria that has been found living near thermal vents in the ocean. A sequence of DNA Is
27 denatured by heating. Upon examination, part of the sequence of DNA has not denatured and must be heated to a higher temperature before It
denatures.
28
29 Which of the following nucleotide sequences is likely concentrated in this area?
30
:
31 A. ATGAA CCAAT GGCTT

32 B. ATGAA CCAAT TTATT


33
C. ATTAA TAAAT TTATT
34
o35 D. CCGGA CCAGT GGCCC

. 36 E. CTGAA CCAGT GGCTC


. 37
. 38
o39
0 40
0 41
• 42
• 43
0 44
0 45
. 46

a
Lock
s
Suspend
8
End Bl ock
Item:35of64 ~. , . M k <:] t> al ~· ~
QIO: 1192 .l. ar Previous Next Lab 'Vfll ues Notes Calculator

26 The co rrect a nswer is D. 73% cho se this.


27 The guanine and cytosine bond is stronger than the adenine and thymine bond because guanine and cytosine form three hydrogen bonds, while adenine
and thymine form only two hydrogen bonds. As a result, it t akes more energy to break the bonds between guanine and cytosine. There are 12 GC bonds
28 in this sequence.
Guanine Thymine Cytosine Adenine Hydrogen bond Hydrogen
29
A is no t co rrect. 3 % cho se this.
30
There are only six GC bonds in this sequence.
31 General classification George Cross

32 B is no t co rrect. 4 % cho se this.


33 There are only three GC bonds in this sequence.
General classification George Cross
34
c is no t co rrect. 1 8 % cho se this.
35
There are no GC bonds in this sequence. This sequence would require the lea st amount of energy to be denatured .
• 36 Denaturation (biochemistry) Denatured alcohol

• 37 E is no t co rrect. 2 % cho se this.


• 38 There are only nine GC bonds in this sequence .
General classification George Cross
o39
• 40
Botto m Li ne:
•41
There are three hydrogen bonds between every guanine and cytosine, and two hydrogen bonds between every adenine and thymine. More energy
• 42 needs to be put into denaturing a series of guanine-cytosine pairs because more bonds need to be broken.
Thymine Cytosine Guanine Adenine Hydrogen bond Denaturation (biochemistry) GC-content Hydrogen
• 43

• 44

• 45
I ill ;fi 1!1 I•J f o r yea r :[ 2017 ..
• 46 FI RST AI D FA CTS

6
lock
s
Suspend
0
End Block
Item: 35 of 64 ~ 1 • M k -<:J 1>- Jil ~· !:';-~
QIO: 1192 ..L ar Pre v ious Next Lab~lues Notes Calcula t o r

A FIRST AID FAC T S A


26

27
FA17 p 33.1
28
Nucleotides 'ucleoSide = base + (deoxy)ribosc (Sugar).
29
NucleoT ide = base + (deoxy)ribosc + phosphaTe; 5' end of incoming nucleotide bears the
30 linked by 3'-5' phosphodiester bond. triphosphate (energy source for the bond).
31 Triphosphate bond is target of 3' hydro') I
32 allack.
33 P URines (A,G )-2 rings. PU Re \ s G old.
PYrimidines (C ,U,T )- 1 ring. CUT the PY (pie).
34
Th) mine has a methyl.
35
Deamination of cytosine makes uracil. C-C bond (3 H bonds) stronger than A-T bond
• 36 Oeamination of adenine makes guanine. (2 H bonds). f C-C content ... f melting
• 37 Uracil found in R lA; thymine in 0 A. temperature of 0 lA. "C -G bonds are like
• 38 Methylation of uracil makes thymine. C razy G lue."
-39 Purine (A, G) Pyrimidine (C. U, 'I)
GAG -Am ino acids necessa ry for purine
0 40 COz Carbamoyl { Aspartate synthesis:
0
41 Aspartate J .,......-Glycine phosphate \ G lycine
\ C N c
• 42 N C N c Aspartate
C- N10-Formyl-
• 43 G lutamine
C c tetrahydrofolate c c
0 44 I N N N
N10-fofmyt· "'--- \ .
0 45 tetrahydrololatr Glutamine
• 46

a
Lock
s
Suspend
8
End Bl ock
Item: 35 of 64
QIO: 1192
~
..L
1 • Ma r k -<:J
Pre v ious
I>
Next Lab
fJ lues
£!1}>'

Notes
!!":-~
Calcula t o r

26
- - . .
C<>z Carbamoyl synthesis:
{ Aspartate
27 Aspartate I ......--Giyone phosphate \ G lycine
\ C N c
28 N C N c Aspartate
C- N10-Fotmyt-
29 c Glutamine
C tetrahydtofoote c c
30 I N N N
NIO-formyt· ~ \ .
31 tetrahydrolo4ate Glutamine

32
33 FA17 p 48.2

34 Polymerase chain lolecular biology laborator} procedure used to amplify a desired fragment ofD ' . Useful as a
35 reaction diagnostic tool (cg, neonatal II IV, herpes encephalitis).
. 36 DNApr~mer
.,
. 37
. 38 s· ...
~
."
111111111111111111
3'

1111 111111111111
3' s·
1111111111111111 I

DNA primer _ /
w
3'


111111111111111111

3'
3'


/

Repeated
o39 111111111111111111 cycles
• 40
3' s· 5' 3'
Double-stranded DNA
• 41 ... ., ~
111111111111111111
3' s·
111111111111111111
3' s·
111111111111111111
3' s·
• 42 ,.~ ~ Steps: 0 Denaturation (T,. 95• C) E) Annealing (T,. 55" C) E) Elongation (T:: n• C)
• 43 Nucleotide DNA Is denatured by healing Dunng cooling. excess Heat-stable DNA polymerase
triphosphate to separate the strands DNApr~mers anneal to a replicates the DNAsequence
• 44 speofic sequence on each following each primer.
Wand to be amplified
• 45
. 46

a
Lock Suspend
s 8
End Bl ock
Item: 36 of 64 ~ 1 • M k -<:J 1>- Jil ~· !:';-~
QIO: 120 7 ..L ar Pre v ious Next Lab~lues Notes Calcula t o r

26
A

During the development of reticulocytes, the hormone erythropoietin stimulates hemocytoblasts to develop into erythroblast cells with large amounts
of euchromatin actively producing mRNA. As the cells mature into RBCs, they develop a biconcave shape to better hold oxygen and become
lA• A] A

27
anucleated.
28
29 Which of the following occurs in the final stage of RBC maturation leading to this phenomenon?
30
:
31 A. Gene activation

32 B. Gene amplification
33
c. Gene loss
34
35 D. Gene rearrangement

. 36 E. Gene recombination
. 37
. 38
-39
0 40
0 41
• 42
• 43
0 44
0 45
. 46

a
Lock
s
Suspend
8
End Bl ock
Item:36of64 ~. , . M k <:] t> al ~· ~
QIO: 1207 .l. ar Previous Next lab 'Vfl1 ues Notes Calculator

26
The co rrect a nswer is c. 55 % cho se this.
27 As reticulocytes mature into RBCs, the nuclei are extruded and there are no genes to produce mRN A. This is an example of the regulation of gene
expression through gene loss.
28 Gene Gene expression Messenger RNA Regulation of gene expression Reticulocyte Cell nucleus Nucleus (neuroanatomy)
29 A is no t co rrect. 1 8% cho se this.
30 Genes can be activated by inducers such as steroid hormones that bind to DNA sequences in the regulatory region of a gene. This is not an example of
gene activation.
31 Gene Regulation of gene expression Steroid DNA Steroid hormone Nucleic acid sequence Hormone
32 B is no t co rrect. 9 % cho se this.
33 In gene amplification, regions of the genome are copied ; this is typically due to misalignments, unequal crossing-over; nondisjunction, or other errors
during replication of somatic cells. As a result, the proteins encoded by the copied genes are amplified. One example of gene amplification can be seen in
34 tumor cells, where oncogenes such as MYC, KRAS or erbBl may be amplified.
35 Nondisjunction Gene KRAS Polymerase chain reaction Gene duplication Genome Myc Oncogene Chromosomal crossover Neoplasm Somatic cell Somatic (biology)
Protein Unequal crossing over
36
D is no t co rrect. 11% cho se this •
• 37
In gene rearrangement, DNA segments from one location are transferred to another location in the genome so that a different protein product is made .
• 38 Antibody production is a good example of this.
Gene Protein Antibody Mutation Genome V{D)J recombination DNA
o39
E is no t co rrect. 7 % cho se this •
• 40
DNA recombination can occur when homologous regions of genes cross over and hybridize. This is not an example of DNA recombination .
•41 Site-specific recombination Recombinant DNA DNA Genetic recombination Homology (biology) Hybrid (biology)

• 42
• 43
Botto m Li ne:
• 44 Gene loss is a phenomenon in which gene expression is turned off secondary to loss of genetic material. It is the mechanism by which reticulocytes
• 45 regulate gene expression .
Gene Gene expression Reticulocyte Deletion (genetics) Genome DNA
• 46

6
lock
s
Suspend
0
End Block
Item: 36 of 64 ~ 1 • M k -<:J 1>- Jil ~· !:';-~
QIO: 120 7 ..L ar Pre v ious Next Lab~lues Notes Calcula t o r
A A
26 FA17 p 38.2

27 Regulation of gene expression


28 Promoter Site\\ here R 'A pol) me rase II and mult iplc Promoter mutation commonly results in
29 other transcription fac tors bind to Dt t\ dramatic ! in b ·el of gene transcription.
upstream from gene locus (AT-rich upstream
30
sequence with TATA and CAJ\T bo,cs).
31
Enhancer tretch of Dl\ that alters gene expression b) Enhancers and silencers mav• be located close to,
32
binding transcription factors (cg, activator far from, or e\·en within (in an intron) the gene
33 proteins). \\hose expression it regulates.
34 Site where negati\·c regulators (repressors) bind.
Silencer
35
36
FA17 p 386.1
• 37
Erythrocyte Carries 0 2 to tissues and C02 to lungs. Erytlt = red; cyte = cell.
• 38
Anucleate and lacks organelles; biconcave rJ, Erythrocytosis= polycythemia= f hematocrit.
-39 with large surface area-to-volume ratio for Anisocytosis= \·arr ing sizes.
0 40 rapid gas exchange. Life span of 120 clays. Poikilocytosis= varying shapes.
0
41 Source of energy is glucose (90% used
in glycolysis, 10% used in li 11P shunt). Reticulocyte= immature RBC; reAects
• 42 erythroid proliferation.
Vlembrane contains CI-IHC03- antiporter,
• 43
wh ich allows RBCs to export IIC03- and Bluish color on Wright-Giemsa stain of
0 44 transport CO, from the peripher) to the lungs reticuloc)tes represents residual ribosomal
0 45 for elimination. R A.
• 46

a
Lock
s
Suspend
8
End Bl ock
Item: 36 of 64 ~ 1 • Ma rk -<:J 1>- Jil ~· !:';-~
QIO: 120 7 ..L Pre v ious Next Lab ~lues Notes Calcula t o r

26 •
Silencer Site where negative regulators (repressors} bind.
27
28
FA17 p386.1
29
Erythrocyte Carries 0 2 to tissues and C02 to lungs. Errth = red; cyte =cell.
30 Anucleate and lacks organelles; biconca,·e Erythrocytosis= polycythemia= t hematocrit.
31 with large surface area-to-,·olume ratio for Anisocytosis= varying sizes.
32 rapid gas exchange. Life span of 120 days. Poikilocytosis= varying shapes.
33 Source of energy is glucose (90% used
in glycolysis, 10% used in H ~ I P shunt). Reticuloc)ie = immature RBC; reflects
34 er} Lhroid proliferation.
\llembrane contains CJ-IHC01- antiporter,
35
which allows RBCs to export IIC03- and Bluish color on Wright-Giemsa stain of
36 transport C0 2 from the periphery to the lungs rcticuloc}tes represents residual ribosomal
• 37 for elimination . R 1A.
• 38
-39
FA1 7 p 38.1
0 40
Functional Transcription start
41 (mRNA synthesized 5' - 3')
0
organization of a
• 42 eukaryotic gene CAAT box TATA box 1 ATG rtart codon
Polyadenylation
signal
• 43
DNA coding strand 5' CAAT TATAAT Exon 1 GT AG Exon 2 GT AG Exon 3 AATAAA 3'
0 44

0 45 Promoter 5' UTR lntron 1 lntron 2 3' UTR

• 46

a
Lock
s
Suspend
8
End Bl ock
Item: 37 of 64 ~ 1 • M k -<:J 1>- Jil ~· !:';-~
QIO: 3699 ..L ar Pre v ious Next Lab~lues Notes Calcula t o r
A A
26
An 8-year-old boy has recently been diagnosed with cystic fibrosis (CF). After undergoing genetic testing, he has been found to be homozygous with
27 the C.FS08 mutation. Curiously, the biological father does not to have the C.FS08 mutation on either allele. Furthermore, neither allele contains any
other mutations known to cause CF. On the other hand, the biological mother Is shown to be heterozygous for the C.FS08 mutation.
28
29 Assuming parental fidelity, what is the likely method of genetic transmission for this child's presentation of cystic fibrosis?
30
:
31 A. Autosomal dominant

32 B. Autosomal recessive
33
c. Biparental disomy
34

35 D. Uniparental disomy

36 E. Uniparental trisomy
• 37
• 38

-39
0 40
0 41
• 42
• 43

0 44

0 45

• 46

a
Lock
s
Suspend
8
End Bl ock
Item: 37 of 64 ~ 1 • M k -<:J 1>- Jil ~· !:';-~
QIO: 3699 ..L ar Prev ious Next Lab~lues Notes Calculat o r

A A
26
Th e correct a nswer i s D. 690/o ch ose this.
27 Cystic fibrosis is an autosomal recessive disease. I n t his vignette the boy's

28
father does not have the mutation, so t he only way t he boy can inherit the
mutation Is through uniparental disomy. As shown in t he diagram,
Acquired Uniparental Disomy (UPD)
29 uniparental disomy occurs when t wo copies of a chromosomethere are
Inherited from the same parent . This arises f rom a nondisjunction event of
30 either meiOSIS I (resulting in uniparenta l heterodisomy) or II (resulting In
uniparental lsosomy). In particular, t he child must have inherited two
31 chromosome 7's from the mother, who had t he 6FS08 mutation. Other
genetic disorders that can arise from uniparental disomy are Angelman
32 syndrome and Prader-Willi syndrom e. • cofJVneurr•ILOW' UPO
33 Angeoonan .yndrome Prader-Willi syndrome Nondisjunction Meoosos Genory.,. AA
8io~gicatly f(IUiva ltnt
Uniparental disomy Cystic fibrosis Autosomal recessive Chromosome 7 (human) T- H•'
34 10

Meiosis I Autosome Mutation Chromosome Recessive Dominance (genetics)


35
Genetic disorder Fibrosis Aneuploidy
36
Fin t Hit Slit6C
37 Ge notype AI
•Point mutbtion ir) rSG
assoCidted with polymorphic
. 38 marker A.
-39 Nondysjunction Random elimination
0 40 I mage courtesy of Wikimedia Commons
0 41
A is no t co rrect. 80/o ch ose this .
• 42
Autosomal dominant is not an inherita nce pattern seen with t.FSOS mutation cystic fibrosis .
• 43 Cystic fibrosis Dominance (genetics) Autosome Mutation Fibrosis

0 44 B i s n ot correct. 18% ch ose this.


0 45 The "classic" inheritance pattern for 6FS0 8 mutation cystic fibrosis is autosomal recessive. Both parents would pass on t he recessive allele to the affected
offspring. However, this patient 's biologica l father is negative for the 6508 mutation. Therefore, there must be anot her explanation for the true
. 46 Inheritance pattern of this child's presentation .

a
Lock
s
Suspend
8
End Bl ock
Item:37of64 ~. , . M k <:] t> al ~· ~
QIO: 3699 .l. ar Previous Next lab 'Vfl1 ues Notes Calculator

26 I mage courtesy of Wikimedia Commons


27
A is no t co rrect. 8% cho se this.
28
Autosomal dominant is not an inheritance pattern seen with .C,FS08 mutation cystic fibrosis.
29 Cystic fibrosis Dominance (genetics) Autosome Mutation Fibrosis

30 B is no t co rrect. 18% cho se this.


The "classic" inheritance pattern for .C,FS08 mutation cystic fibrosis is autosomal recessive. Both parents would pass on the recessive allele to the affected
31
offspring. However; this patient's biological father is negative for the .C,S08 mutation . Therefore, there must be another explanation for the true
32 inheritance pattern of this child's presentation .
Cystic fibrosis Allele Autosomal recessive Recessive Autosome Mutation Dominance (genetics) Recessive allele Fibrosis
33
c is no t co rrect. 2 % cho se this.
34
Biparental disomy would result in a child with four copies of a particular chromosome; this is an unlikely presentation of .C,FS08 mutation cystic fibrosis.
35 Cystic fibrosis Chromosome Mutation Fibrosis Uniparental disomy Aneuploidy

36 E is no t co rrect. 3 % cho se this.

37 Uniparental trisomy is not a t erm commonly used in genetics. In theory, it would mean inheriting three copies of the same chromosome from the same
parent. In reality, nondisjunction events result in only two copies of the same chromosome (uniparental disomy) .
• 38 Nondisjunction Chromosome Genetics Trisomy Uniparental disomy Aneuploidy

o39
• 40 Bo tto m Line:
•41 Uniparental disomy is the inheritance of two copies of a chromosome from one parent through nondisjunction.
Nondisjunction Uniparental disomy Chromosome Aneuploidy
• 42
• 43

• 44
I iii I;fi 1!1 I•J f o r year:[ 20 17 "
• 45 FI RST AI D FA CTS

• 46

6
lock
s
Suspend
0
End Block
Item:37of64 ~. , . M k <:] t> al ~· ~
QIO: 3699 .l. ar Previous Next lab 'Vfl1 ues Notes Calculator

• •
26 FA17 p 52.2
27 Genetic terms

28 TERM DEFINITION EXAMPLE

29 Codominance Both alleles contribute to the phenotype of the Blood groups A, B, AB; a 1-antitrypsin
heterozygote. deficiency.
30
Variable expressivity Patients with the same genotype have varying 2 patients with neurofibromatosis type 1 ( 1Fl)
31
phenotypes. may have varying disease severity.
32
33
Incomplete 'ot all individuals with a mutant genotype BRCAJ gene mutations do not always result in
penetrance show the mutant phenotype. breast or ovarian cancer.
34
35
Pleiotropy One gene contributes to multiple phenotypic Untreated phenylketonuria (PKU) manifests with
effects. light skin, intellectual disability, and musty body
36
odor.
37
Anticipation Increased severity or earlier onset of disease in Trinucleotide repeat diseases (cg, Huntington
• 38
succeeding generations. disease).
o39
Loss of heterozygosity If a patient inherits or develops a mutation in Retinoblastoma and the "two-hit hypothesis,"
• 40
a tumor suppressor gene, the complementary Lynch syndrome (H PCC), Li-Fraumcni
• 41 allele must be deleted/mutated before cancer syndrome.
• 42 develops. This is not true of oncogenes.
• 43 Dominant negative Mutation of a transcription factor in its allosteric
Exerts a dominant effect. A heterozygote
• 44 mutation produces a nonfunctional altered protein that site. onfunction ing mutant can still bind
• 45 also prevents the normal gene product from DNA, preventing wild-type transcription factor
• 46 functioning. from binding. •

6
lock
s
Suspend
0
End Block
Item: 37 of 64 ~ 1 • M k -<:J 1>- Jil ~· !:';-~
QIO: 3699 ..L ar Pre v ious Next Lab~lues Notes Calcula t o r
A A
26 FA17 p 56.3

27 Cystic fibrosis
28 GENETICS Autosomal recessive; defect in CFTR gene on chromosome 7; commonly a deletion of Ph608.
29
!lost common lethal genetic disease in Caucasian population.

30 PATHOPHYSIOLOGY CFTR encodes an ATP-gated CJ- channel that secretes CJ- in lungs and GTtract, and reabsorbs
CJ- in s"eat glands. lost common mutation .... misfolded protein .... protein retained in RER
31
and not transported to cell membrane, causing l CJ- (and H10 ) secretion; t intracellular C)-
32 results in compensatory t 1 a+ reabsorption via epithelial Na+ channels .... t 1120 reabsorption
33 .... abnormally thick mucus secreted into lungs and Gl tract. t 1 a+ reabsorption also causes more
34 negati,·e transepithclial potential difference.
35 DIAGNOSIS t Cl- concentration (> 60 mEq/L) in $\\Cal is diagnostic. Can present with contraction alkalosis
36 and hypokalemia (ECF effects analogous to a patient taking a loop diuretic) because of ECF
37
H20f!'\a+ losses and concomitant renal K+f )J+ wasting. t immunoreacti,·e trypsinogen (newborn
screening).
• 38
COMPLICATIONS Recurrent pulmonary infections (eg, S aureus [early infancy), P aeruginosa [adolescence]), chronic
-39
bronchitis and bronchiectasis .... reticulonodular pattern on CXR, opacification of sinuses.
40
0
Pancreatic insufficiency, malabsorpt ion with steatorrhea, fat-soluble vitamin deficiencies (A, D, E,
0
41 K), bi liary cirrhosis, liver disease. Meconium ileus in newborns.
• 42 Infertil ity in men (absence of vas deferens, spermatogenesis may be unaffected) and subfertility in
• 43 women (amenorrhea, abnormally thick cen•ical mucus).
lasal polyps, clubbing of nails.
0 44
TREATMENT ~ Iultifactorial:chest physiotherapy, albuterol, aerosolized dornase alfa (DNAse), and hypertonic
0 45
saline facilitate mucus clearance. zithromycin used as anti-inflammatory agent. Ibuprofen slows
• 46 P..,n,....rP.,t-i,..
• r1ic-P'!JCQ n rnarPC"c-inn Pn?un--.Pc: fnr inc11(h,....i,nt'u

a
Lock
s
Suspend
8
End Bl ock
Item: 38 of 64 ~ 1 • M k -<:J 1>- Jil ~· !:';-~
QIO: 1376 ..L ar Pre v ious Next Lab~lues Notes Calcula t o r
A A
26 A female Infant is born to a 38-year-old Gl Pl female at 37 weeks' gestation. The birth and pregnancy were without complications, despite minimal
27 prenatal care . On physical examination, several dysmorphic features are noted, Including small, low-set ears, a protruding, furrowed tongue, single
transverse palmar creases, excessive skin on the posterior neck, and a flat nasal bridge. The infant has poor tone in both the upper and lower
28 extremities and a weak Moro reflex. In addition, a diffuse vesicular rash Is observed. Vital signs are within normal limits, and a complete blood count and
blood cultures are unremarkable. Peripheral smear shows > 5% megakaryoblasts (abnormally high). At the child's 3-month visit, the rash has resolved, and a
29 repeat peripheral blood smear is unremarkable.
30
31 The patient's transient condition in the newborn period directly places her at risk of what disease later in life?

32 :
A. Acute lymphocytic leukemia
33
34 B. Acute myeloid leukemia

35 c. Alzheimer disease
36
o. Obesity
37
. 38 E. Transient myeloproliferative disorder

-39
0 40
0 41
• 42
• 43
0 44
0 45
. 46

a
Lock
s
Suspend
8
End Bl ock
Item: 38 of 64 ~ 1 • M k -<:J 1>- Jil ~· !:';-~
QIO: 1376 ..L ar Prev ious Next Lab~lues Notes Calculat o r

A A
26
Th e correct a nsw er i s B. 400/o chose this.
27 Acute myeloid leukemia (AML) is a broad cat egory of neoplasms all characterized by a clonal proliferation
of myeloid precursor cells. A specific subtype of AML, called acute megakaryoblastlc leukemia, accounts for
28
<5% of all AMLs. However, the infant in the vignette has Down syndrome, or trisomy 21, as indicated by
29 the advanced maternal age (> 35 years) and the patient's physica l exam findings, which include classic
dysmorphlc features, like those in the image. In patients with Down syndrome, who suffer transient
30 myeloproliferative disorder (TMD) during infancy, the risk of acute megakaryoblastlc leukemia is increased
to 20%. It Is thought that mutations in the same cell population give rise to both disorders. This
31 association is most likely not required for the USMLE Step 1. However, the NBME can wnte cntical thinking
32 questions (including information on diseases with which you are not familiar) that can be answered
correctly by utilizing your current knowledge for critical basic science thinking.
33
It Is known that the patient expe ri enced a myelogenous disease in infancy, thus, it follows that the single
34 best answer would also involve the myeloid line.

35 Down syndrome Acute megakaryoblastic leukemia Myeloproliferative neoplasm Acute myeloid leukemia Myeloid
Leukemia Neoplasm Megakaryoblast Hematopoietic stem cell Myeloid leukemia
36
37
38
-39
I mage courtesy of CDC, National Center on
0 40 Birth Defects and Developmental Disabilities

0
41
A is no t co rrect . 3 10/o ch ose this.
0
42 Having trisomy 21 increases the risk for developing acute lymphocytic leukemia (ALL) 10- to 20-fold above the general population . However, the question
asks us to correlate the patient's transient proliferation of megakaryoblasts In the newborn period with another pathology. Specifically, what disease does
• 43
this proliferation of myeloid precursor cells place the patient at DIRECT risk of developing? Although it is true that patients with Down syndrome have an
0 44 Increased risk of ALL compared with the general population, this increased risk of clonal expansion of lymphoid precurser cells would not be related to a
transient proliferation of myeloid precurser cells in infancy. Recall that megakaryocytes are of the myleoid line. Thus, ALL is not the single best answer.
0 45 Down syndrome Acute lymphoblastic leukemia Leukemia Trisomy Megakaryocyte Myeloid Pathology Lymphatic system Hematopoietic stem cell

•46 C i s not corr ect. 160/o chose this•



a
Lock
s
Suspend
8
End Block
Item:38of64 ~. , . M k <:] t> al ~· ~
QIO: 1376 .l. ar Previous Next lab 'Vfllues Notes Calculator

26 c is no t co rrect. 16% cho se this.


Down syndrome places patients at extremely high risk of developing dementia lat er in life. However; dementia is most likely not relat ed to a transient
27
proliferation of myeloid precursor cells (megakaryoblasts) in infancy.
28 Down syndrome Dementia Myeloid Hematopoietic stem cell

29 D is no t co rrect. 3 % cho se this.


Grea t er proportions of patients with Down syndrome are obese than in the general population . This is thought to be due to their lower resting met abolic
30
rat e and not to a hematologic abnormality in infancy. This is not the single best answer.
31 Down syndrome Basal metabolic rate Metabolism Obesity Kleiber' s law Resting metabolic rate

32 E is no t co rrect. 10% cho se this.


Transient myeloproliferative disorder (TMD) is seen in approximat ely 20% of newborns with Down syndrome. it is mostly asymptomatic; however ; it can
33
be fat al. It usually resolves within 2 - 3 months. It is characterized by a proliferation of megakaryoblasts in the peripheral blood smear and bone marrow.
34 Often hemoglobin, neutrophil, and plat elet counts are unaffected. Most patients, even those who are asymptomatic, experience a vesiculopustular skin
rash, which if biopsied, would contain the same blasts seen in the peripheral blood and bone marrow . The disorder can be diagnosed with peripheral
35 smea r; bone marrow aspirat e, or flow cytometry, which would indicat e an increa sed number of megakaryoblasts. Patients who suffer from TMD have a
20% increa sed risk of acute megakaryoblastic leukemia, which generally develops before the age of 4 . As this patient had TMD in infancy and the
36 question asks what condition she is at increa sed risk o developing, this is not the single best answer.
37 Down syndrome Neutrophil Myeloproliferative neoplasm Flow cytometry Hemoglobin Blood film leukemia Platelet Bone marrow Acute megakaryoblastic leukemia
Bone marrow examination Asymptomatic Biopsy Bone Megakaryoblast Rash
38
o39
0 40 Botto m Li ne:

0 41 Although the incidence of ALL is increa sed significantly in Down syndrome, the most common hematologic abnormality is acute megakaryoblastic
leukemia.
0
42 Down syndrome Acute megakaryoblastic leukemia leukemia Megakaryoblast Hematology

0 43

0 44

0 45 Iiii I;fi IJ I•J f o r yea r :[ 2017 ..


FI RST AID FAC T S
0
46

6
lock
s
Suspend
0
End Block
Item: 38 of 64 ~ 1 • M k -<:J 1>- Jil ~· !:';-~
QIO: 1376 ..L ar Pre v ious Next Lab~lues Notes Calcula t o r
A A
26 FA17 p 59. 1
27 Autosomal trisomies
28 Down syndrome Findings: intellectual disabilit}, flat facies, Incidence 1:700.
29 (trisomy 21 ) prominent epicanthal folds, single palmar Orin king age (21).
30
crease, gap between lsi 2 toes, duodenal lost· common ,·iable chromosomal disorder and
atresia, Hirschsprung disease, congenital heart most common cause of genetic intellectual
31
disease (eg, atriO\·entricular septal defect), disability.
32 Brushfield spots. ssociated with early-onset First-trimester ultrasound commonly shows
33 Alzheimer disease (chromosome 21 codes for t nuchal translucency and h~ poplastic nasal
34 amyloid precursor protein) and t risk of ALL bone; l serum PAPP-A, t free ~-hCC.
35
and A IL. econd-trimester quad screen shows
95% of cases due to meiotic nondisjunction l a-fctoprotein, t f3-hCC, l estriol,
36
(t with advanced maternal age; from 1:1500 in t inh ibin
37 women< ZO to 1:25 in women> 45 years old).
38 4% of cases clue to unbalanced Robertson ian
-39 translocation, most typically between
0 40 chromosomes 14 and Zl. 1% of cases due
to mosaicism (no association with materna l
0
41
nondisjunction; postfertilization m itoI ic error).
0
42
Edwards syndrome Find ings: PRIJ\CF. F.clward- Promincnt Incidence 1:8000 .
• 43
(trisomy 18) occiput, Rocker-bottom feel, Intellect ua I Election age (18).
0 44 disability, 1\ondisjunction, Clenched fists 2nd most common autosomal trisomy resulting
0 45 (with o,·erlapping fingers), low-set Ears, in live birth (most common is Down syndrome).
• 46 micrognathia (small jaw), congenital hea rt P PP-A and free f3-hCC are l in first trimester.

a
Lock
s
Suspend
8
End Bl ock
Item:38of64 ~. , . M k <:] t> al ~· ~
QIO: 1376 .l. ar Previous Next Lab 'Vfll ues Notes Calculator

• •
26 FA17 p410.1
27 Leukemias Unregulated growth and differentiation of WBCs in bone marrow - marrow fa ilure - anemia
28 (l RBCs), infections (I mature WBCs}, and hemorrhage (I platelets). Usually presents with
29 f circulating WBCs (mal ignant leukocytes in blood); rare cases present with normal/! WBCs.
Leukemic cell infiltration of liver, spleen, lymph nodes, and skin (leukemia cutis) possible.
30
TYPE NOTES
31
Lymphoid neoplasms
32
Acute lymphoblastic Most frequently occurs in children; less common in adults (worse prognosis). T-cell ALL can
33
leukemia/ lymphoma present as mediastinal mass (presenting as SVC-like syndrome). Associated with Down syndrome.
34 Peripheral blood and bone marrow have f f f lymphoblasts fl.
35 TdT+ (marker of pre-1' and pre-B cells}, CDIO+ (marker of pre-B cells).
36 Most responsive to therapy.
37 May spread to C 1$ and testes.
t(l2;21) - better prognosis.
38
Chronic lymphocytic Age: > 60 years. Most common adult leukemia. C D20+, CD23+, CD5+ B-cell neoplasm. Often
o39
0 40
leukemia/small asymptomatic, progresses slowly; smudge cells in peripheral blood smear; autoimmune rn
lymphocytic hemolytic anemia. CLL = Crushed Little Lymphocytes (smudge cells).
• 41 lymphoma Richter t ransformation-CLLISLL transformation into an aggressive lymphoma, most commonly
• 42 diffuse large B-cell lymphoma (DLBCL).
• 43 Hairy cell leukemia Age: /\dull males. Mature B-cell tumor. Cells have fi lamentous, hair-like projections
0 44 {fuzzy appearing on LM (!!). Peripheral lymphadenopathy is uncommon.
0 45 Causes marrow fibrosis - dry tap on aspiration. Patients usually present with massi,·e splenomegaly
and pancytopenia.
• 46 <::.._,;..,"TRAP / h.,rf.r.-.f-.o, r .oC":c-h •l 1"ll- ..,,..;,..I "''""".,h . .,t-.-.r£>' r.I:\ TRAP r f.., ;n lt'lrrul.lu ro n,l •·u•or11u:4--l, l=l n,u •

6
lock
s
Suspend
0
End Block
Item: 39 of 64 ~ 1 • M k -<:J 1>- Jil ~· !:';-~
QIO: 1394 ..L ar Pre v ious Next Lab~lues Notes Calcula t o r

26
A

An 8-year-old boy is evaluated by his pediatrician for abnormalities of the skin. On physical examination, t he patient is noted to have multiple
hyperpigmented macules on the trunk, freckles in t he axillary region, and tan-colored nodules of the iris. Genetic analysis of multiple members of the

lA A] A

27
patient's family shows that they all have t he sa me muta nt gene. However, not all family members have t he sa me symptoms.
28
29 Which of the following terms describes t he presentation of t his patient's genetic disease?
30
:
31 A. AntiCipation

32 B. Codominance
33
C. Dominant negative mutation
34

35 D. Incomplete penetrance

36 E. Variable expressivity

37
38
0 39

0 40
0 41
0 42
• 43

0 44

0 45

• 46

a
Lock
s
Suspend
8
End Bl ock
Item: 39 of 64 ~ 1 • M k -<:J 1>- Jil ~· !:';-~
QIO: 1394 ..L ar Pre v ious Next Lab~lues Notes Calcula t o r
A A
26 Th e co rrect an sw er i s E. 7 40/o chose this.
27 variable expressivity indicates that the phenotype varies among individuals with the same genotype. The mutant gene is expressed in all individuals that
carry It but in different ways. Neurofibromatosis type 1 is an example. Cafe-au-lait spots, axillary freckling, Lisch nodules, optic pathway gliomas, and
28 headaches are characteristic features that may appear by 6-10 years of age. Subcutaneous and cutaneous neurofibromas, hypertension, and malignant
peripheral nerve sheath tumors may begin to appear in adolescence and adulthood. The images illustrate some of the variation in expresslvlty In
29 neurofibromatosis type 1.
30 Neu ofobromatosis type I Neurofibromatosis Gene Phenotype Hypertension Gl oma F• eckle Neurofibroma Usch nodule Optic nerve Genotype
Expressivity (genetics) cancer Malignancy Neoplasm
31
32
33
34
35
36
37 Cafe-au-lait spots Axi llary freckling Cutaneous neurofibromas
38 Images copyright ©2013 by Anais Brasileiros de Oermatologia
39 A is no t co rrect . 20/o chose this.
0 40 Anticipation Is the term used when a disea se worsens or the age of onset of the disease is earlier in successive generations. Huntington disease and
fragile X syndrome (both of which involve trinucleotide repeats) show anticipation.
0 41 Fragile X syndrome Huntington's disease Trinucleotide repeat disorder
0 42 B Is no t co rrect . 1 o;o ch ose this .
• 43 Codomlnance Is a term used when both alleles contribute to the phenotype of the heterozygote. Blood groups and o 1-antitrypsin deficiency Illustrate
codomlance. In a person with type AB blood group, both A and B alleles are equally expressed and are, therefore, codominant .
0 44 Codominance Heterozygote Dominance (genetics) Zygosity Phenotype Blood type Human blood group systems Allele Codominant
0 45 C i s not cor r ect. 2 0/o ch ose t his•
. 46 Dominant negative mutation is a mutation in which the mutant product Interferes with the function of the normal gene product, leading to pathology.

a
Lock
s
Suspend
8
End Bl ock
Item:39of64 ~. , . M k <:] t> al ~· ~
QIO: 1394 .l. ar Previous Next lab 'Vfl1 ues Notes Calculator

26 A is no t co rrect. 2% cho se this.


27 Anticipation is the t erm used when a disea se worsens or the age of onset of the disea se is earlier in successive generations. Huntington disea se and
fragile X syndrome (both of which involve trinucleotide repea ts) show anticipation .
28 Fragile X syndrome Huntington' s disease Trinucleotide repeat disorder

29 B is no t co rrect. 1 Ofo cho se this.

30 Codominance is a t erm used when both alleles contribute to the phenotype of the het erozygote. Blood groups and a 1 -antitrypsin deficiency illustrat e
codomiance. In a person with type AB blood group, both A and B alleles are equally expressed and are, therefore, codominant.
31 Codominance Heterozygote Dominance (genetics) Zygosity Phenotype Blood type Human blood group systems Allele Codominant

32 c is no t co rrect. 2% cho se this.


33 Dominant negative mutation is a mutation in which the mutant product interferes with the function of the normal gene product, lea ding to pathology.
Marfan syndrome provides an example of a dominant negative mutation . The defective product of the mutat ed fibrillin gene ( FBN l ) antagonizes the
34 product of the normal allele.
Marfan syndrome Allele Gene FBNl Mutation Dominance (genetics) Fibrillin Dominant negative mutation Mutant Gene product Pathology
35
Dis no t co rrect. 21% cho se this.
36
Incomplet e penetrance indicat es that not all individuals with a mutant genotype exhibit the mutant phenotype at all. For example, BRCAl gene
37 mutations do not always result in brea st or ova rian cancer. In this question stem, the family members have differences in symptoms but all of them have
some symptoms as a result of their mutant genotype.
38 BRCAl Penetrance Gene Ovarian cancer Phenotype Mutant Genotype Cancer
39
0 40
Bo tto m Line:
0 41
Variable expressivity means that all individuals with the mutant genotype express it, but may do so differently. Incomplet e penetrance means that of
0
42 the individuals with the mutant genotype, some may not express it and will appear phenotypically normal.
Penetrance Genotype Mutant Phenotype Expressivity (genetics)
0 43

0 44

45
0
Iiii I;fi IJ I•J f o r year:[ 20 17 ..
0
46 FI RST AID FAC T S

6
lock
s
Suspend
0
End Block
Item: 39 of 64 ~ 1 • M k -<:J 1>- Jil ~· !:';-~
QIO: 1394 ..L ar Pre v ious Next Lab~lues Notes Calcula t o r
A A
26 FA17 p 52.2

27
Genetic terms
TERM DEFINITION EXAMPLE
28
29
Codominance Both alleles contribute to the phenotype of the Blood groups A, B, AB; a 1-antitrypsin
heterozygote. deficiency.
30
Variable expressivity Patients with the same genotype ha\e var} ing 2 patients with neurofibromatosis t~ pe I (I\ Fl)
31
phenotypes. mar have ,·arying disease se,·erity.
32
Incomplete l\ot all individuals'' ith a mutant genot) pe BRCA I gene mutations do not always result in
33
penetrance show the mutant phenotype. breast or ovarian cancer.
34

35
Pleiotropy One gene contributes to multiple phenotypic Untreated phenylketonuria (PKU) manifests with
effects. light skin, intellectual disability, and musty bod}
36
odor.
37
Anticipation Increased se,·erity or earlier onset of disease in Trinucleotide repeat diseases (eg, l luntington
38
succeeding generations. disease).
39
Loss of heterozygosity If a patient inherits or develops a mutation in Reti noblastoma and the "two-hit hypothesis,"
0 40
a tumor suppressor gene, the complementary Lynch syndrome (HNPCC), Li-Fraumcni
0
41 allele must be deleted/mutated before c:1nccr syndrome.
0
42 develops. This is not true of oncogenes.
• 43 Dominant negative Exerts a dominant effect. A heterozygote 1utation of a transcription factor in its allosteric
0 44 mutation produces a nonfunctional altered protein that site. on functioning mutant can still bind
0 45 also prevents the normal gene product from DNA, preventing wild-type transcription factor
• 46
functioning. from binding.

a
Lock
s
Suspend
8
End Bl ock
Item: 39 of 64 ~ 1 • M k -<:J 1>- Jil ~· !:';-~
QIO: 1394 ..L ar Pre v ious Next Lab~lues Notes Calcula t o r

mosa1c1sm. •
26

27 locus heterogeneity Mutations at different loci can produce a simi lar Albinism.
phenotype.
28
Allelic heterogeneity Different mutations in the same locus ~-th alassemia.
29
produce the same phenol) pe.
30
Heteroplasmy Presence of both normal and mutated
31
mtD. A, resulting in ,·ariable e>..pression in
32
mitochondrially inherited disease.
33
Uniparental disomy Offspring receives 2 copies of a chromosome from Uniparental is euploid (correct number of
34 1 parent and no copies from the other parent. chromosomes), not aneuploid. ~ lost occurrences
35 Heterodlsomy (heterozygous) indicates a meiosis of uniparental disomr (UPD) - normal
36 I error. Isodlsomy (homozygous) indicates a phenotype. Consider UPD in an individual
37
meiosis II error or postzygotie chromosomal manifesting a recessive disorder when only one
duplication of one of a pair of chromosomes, parent is a carrier.
38
and loss of the other of the original pair.
39
0 40
FA17 p 56.1
0
41
Autosomal dominant Achondroplasia, autosomal dominant polycystic kidney disease, fam ilial adenomatous polyposis,
42
famil ial hypercholesterolemia, hereditary hemorrhagic telangiectasia, hereditary spherocrtosis,
0

diseases
• 43 Huntington disease, Li-F'raumeni S) ndrome, ,\ 1arfa n syndrome, multiple endocrine neoplasias,
• 44 neurofibromatosis type I ('on Recklinghausen disease), neurofibromatosis type 2, tuberous
• 45 sclerosis, von Hippel-Lindau disease.
• 46

a
Lock
s
Suspend
8
End Bl ock
Item: 40 of 64
QIO: 3372 ~
~ , .1 • Mar k <1
Previous
t>
Next Lab
d\ ues
•.

Notes
~
calculator

• A 24- year-old man has sensorineural hearing loss, heart block, cerebellar ataxia, easy fatigability, and proximal muscle weakness , He has a known myopathy
26 diagnosed in childhood, His mother and maternal aunt were known to have a sim ilar syndrome , No one on his father's side is known to be affected , Results of a
27 muscle biopsy are shown in the image ,

28
29
30
31
32
33
34
35
36
37
.~

38 •'
39
Image courtesy of Wikipedia
• 40
• 41
What is the most likely mode of inheritance for this man's disorder?
• 42
:
• 43 A. Mitochondr ial
• 44
B. Autosomal dom inant
• 45
• 46 c. Autosomal recessive

• 47 D. X-linked dom inant


• 48
.. • E. X- linked recessive •

a
Lock
s
Suspend
o
End Block
Item: 40 of 64 ~ 1 • M k -<:J 1>- Jil ~· !:';-~
QIO: 3372 ..L ar Prev ious Next Lab~lues Notes Calculat o r

A A

26 Th e correct a nsw er i s A. 540/o ch ose this.


27 This patient's symptoms and biopsy findings are compatible with ragged red fiber (RRF) myopathy, caused
by a deletion In mitochondrial DNA. The yellow boxes in the image show RRFs In a Gomori trichrome stain.
28 Typical symptoms of RRF myopathy are ophthalmoplegia, dysphagia, proximal muscle weakness,
sensorineural hearing loss, cerebellar ataxia, and cardiac conduction defects. These syndromes are passed
29
along through mitochondrial inheritance. Mitochondrial inheritance Is limited to mother-to-child •
30 transmission because only the ovum contributes mitochondria to the progeny; the sperm does not.
Therefore, it IS Important to note that a father wit h one of these syndromes cannot pass 1t to his children.
31 The mother does not necessarily need t o express dinical symptoms, as mitochondrial genetics has variable
expression due to heteroplasmy.
32 Heteropla,,my Dysphagia Mitochondrion Sensorineural hearing loss Mitochondrial DNA Ophthalmoparesis
33 Cerebellar ataxia Ataxia Biopsy Myopathy Hearing loss DNA Egg cell Genetics Cerebellum •
34 Anatomical terms of location Muscle weakness Muscle Expressivity (genetics) Vertically transmitted infection
Image courtesy of Wikipedia
35
36 B is n ot co rrect. 1 30/o chose this.
The mode of inheritance of ragged red fiber (RRF) myopathies is mitochondrial, not autosomal dominant. The question stem hints at mitochondrial
37 Inheritance with affected family members only on the maternal side, and no affected members on the paternal side. Although this does not preclude
38 autosomal dominant inheritance, the diagonosis of RRF (symptoms and muscle biopsy) makes the correct answer choice mitochondrial inheritance.
Myopathy Dominance (genetics) Biopsy Autosom e Mitochondrion Muscle biopsy Mitochondrial DNA Muscle
39
C is no t co rrect. 70fo chose this.
40 Although many autosomal recessive syndromes are severe and present In early childhood, the ragged red fibers (RRFs) seen on this patient's biopsy
• 41 should alert you to the fact that this is RR F myopathy.
Autosomal recessive Myopathy Biopsy Dominance (genetics) Recessive Autosome Mitochondrial disease
• 42
D is n o t co rrect. 140/o ch ose this .
• 43 Only a few known diseases are transmitted in an X-linked dominant fashion . In these disorders, either male or female offspring may be affected, and all
• 44 female offspring of an affected father will be affected. One such syndrome Is hypophosphatemic rickets, which results in increased phosphate wasting In
the proximal tubule, leading to a rickets-like presentation .
• 45 R1ckets Proximal convoluted tubule Sex linkage Phosphate X-linked dominant onherltance Anatomical terms of location

• 46 • E i s n o t correct. 1 20/o chose t his•

a
Lock
s
Suspend
8
End Block
Item: 40 of 64 ~. I • M k <:] t> al ~· ~
QIO: 3372 .l. ar Previous Next Lab 'Vfll ues Notes Calculator

26 E is not correct. 12% chose this.


Although there is evidence of disease on the m aternal side of the family and one could trace the defective gene to the m aternal lineage, the m ajor hint
27 that this disease cannot be X-linked recessive is that the proband has a m aternal aunt who has the disease. Although it is possible for a female to be
28 affected by an X-linked recessive disorder due to lyonization, the pattern seen in this vignette is more suggestive of a mitochondrial inheritance .
Proband X-inactivation Gene X-linked recessive inheritance Recessive Dominance (genetics) Sex linkage Mitochondrion Mitochondrial DNA
29
30
Bottom Line:
31
The ragged red fiber myopathies are inherited through the mother; because all of the embryo's mitochondria are provided via the ovum.
32 Mitochondrion Myopathy Egg cell

33
34

35 Iiii I;fi IJ I•J for year:[ 2017 ..


FI RST AID FAC T S
36
37 FA17 p 55.1
38 Modes of inheritance
39 Autosomal dominant Often due to defects in structural genes. Many Often pleiotropic (multiple apparently unrelated
40 generations, both males and females are effects) and variably expressive (different
• 41 affected . between individuals). Family history crucial
to diagnosis. With one affected (heterozygous)
• 42
parent, on average, Y2 of children affected .
• 43

• 44

• 45

• 46 Autosomal recessive Often due to enzyme deficiencies. Usual ly seen Common ly more severe than dominant disorders;

6
lock
s
Suspend
0
End Block
Item: 41 of 64 ~ 1 • M k -<:J 1>- Jil ~· !:';-~
QIO: 1397 ..L ar Pre v ious Next Lab~lues Notes Calcula t o r

26
A

A 30-year-old patient presents to an ophthalmologist with complaints of decreased vision . Multiple angiomatous lesions are visible in the retina .
History Is significant for cerebellar and spinal hemangioblastomas, bilateral renal cysts, and pancreatic microcystic adenomas. DNA analysis revealed a
IA•A] A

27
deleted tumor suppressor gene on chromosome 3.
28
29 Which of the following is the inheritance pattern of this patient's disease?
30 :
31 A. Autosomal dominant

32 B. Autosomal recessive
33
c. Mitochondrial
34
35
o. Spontaneous

36 E. X-llnked recessive

37
38
39
40
• 41
• 42
• 43
• 44
• 45
• 46 •
a
Lock
s
Suspend
8
End Bl ock
Item:41of64 ~. , . M k <:] t> al ~· ~
QIO: 1397 .l. ar Previous Next Lab 'Vfl1ues Notes Calculator

26 The co rrect a nswer is A. 67% cho se this.


27 This patient has von Hippei- Lindau disea se, an autosomal dominant disorder characterized by abnormal blood vessel growth. The overgrowth of blood
vessels lea ds to angiomas and hemangioblastomas in the retina, brain, and spinal cord, as well as in other regions of the body. Patients also show cystic
28 growths in the kidneys and pancrea s, pheochromocytomas, islet cell tumors, and clear -cell renal carcinoma. The disea se is due to loss of function
mutation of the VHL tumor suppressor gene on the short arm of chromosome 3. Autosomal dominant inheritance involves multiple generations of a
29 family, with multiple individuals affected. According to the "two-hit hypothesis," tumor suppressive genes require the loss of function of both alleles to
30 cause expression of the disea se.
Von Hippel-lindau disease Blood vessel Gene Retina Dominance (genetics) Tumor suppressor gene Pancreas Mutation Neoplasm Knudson hypothesis
31 Renal cell carcinoma Spinal cord Allele Autosome Chromosome Pheochromocytoma Kidney cancer Pancreatic islets Kidney Carcinoma Brain
32
B is no t co rrect. 22% cho se this.
33 In autosomal recessive (AR ) inheritance, a defective gene from ea ch carrier parent is transmitted to the offspring. Disea se often is seen in only one
generation . Males and females are equally likely to be affected. Cystic fibrosis and the thalassemias are examples of AR disea ses.
34
Cystic fibrosis Gene Autosomal recessive Recessive Autosome Dominance (genetics) Fibrosis
35
c is no t co rrect. 2% cho se this.
36 In mitochondrial inheritance, children (male and female) of an affected mother may exhibit the disea se. The disea se is not transmitted from fathers to
any of their children (only mat ernal transmission). Leber hereditary optic neuropathy is an example of a mitochondrial disorder.
37
Mitochondrial disease Peripheral neuropathy Leber' s hereditary optic neuropathy Mitochondrial DNA Mitochondrion Optic neuropathy
38
D is no t co rrect. 4 % cho se this.
39 Spontaneous mutations generally affect only one member of a family and are not seen in multiple generations.
Mutation
40
41 E is no t co rrect. 5 % cho se this.
In X-linked recessive inheritance, affected males inherit a defective copy of the X chromosome from het erozygous (asymptomatic) mothers. There is no
• 42 male-to-male transmission. Hemophilia A and glucose-6 -phosphat e dehydrogenase deficiency are examples of X-linked recessive disea ses. An important
concept to remember is that although males are most commonly affected by X-linked recessive disea ses, females may also show mild-moderat e
• 43
symptoms due to the effect of skewed X-inactivation .
• 44 Sex linkage Haemophilia X chromosome X-inactivation Heterozygous Haemophilia A Glucose-6-phosphate dehydrogenase
Glucose-6-phosphate dehydrogenase deficiency X-linked recessive inheritance Recessive Dominance (genetics) Asymptomatic Chromosome Glucose 6-phosphate
• 45

• 46

6
lock
s
Suspend
0
End Block
Item:41of64 ~. , . M k <:] t> al ~· ~
QIO: 1397 .l. ar Previous Next lab 'Vfl1ues Notes Calculator

26 Mutation

27 E is no t co rrect. 5 % c ho se this .
In X-linked recessive inheritance, affected m ales inherit a defective copy of the X chromosome from heterozygous (asymptomatic) mothers. There is no
28 m ale -to -m ale transmission . Hemophilia A and glucose -6 -phosphate dehydrogenase deficiency are exa mples of X-linked recessive diseases. An important
29 concept to rem ember is that although m ales are most commonly affected by X-linked recessive diseases, fem ales m ay also show mild-moderate
symptoms due to the effect of skewed X-inactivation.
30 Sex linkage Haemophilia X chromosome X-inactivation Heterozygous Haemophilia A Glucose-6-phosphate dehydrogenase

31 Glucose-6-phosphate dehydrogenase deficiency X-linked recessive inheritance Recessive Dominance (genetics) Asymptomatic Chromosome Glucose 6-phosphate

32
33 Bo tto m Line :

34 Von Hippei- Lindau disease is an autosomal dominant syndrome caused by deletion of the VHL tumor suppressor gene on chromosome 3p. The most
common m anifestations are hem angioblastom as and renal cell carcinoma .
35 Von Hippel-lindau disease Renal cell carcinoma Tumor suppressor gene Gene Dominance (genetics) Chromosome Carcinoma Neoplasm Autosome Kidney VHl

36
37
38 I ill ;fi 1!1 I•J f o r year:[ 20 17 ..
FI RST AI D FA CTS
39
40 FA11 p 56.1
41
Autosomal dominant Achondroplasia, autosomal dominant polycystic kidney disease, fam ilial adenomatous polyposis,
• 42 diseases familial hypercholesterolemia, hereditary hemorrhagic telangiectasia, hereditary spherocytosis,
• 43 Huntington disease, Li-Fraumeni syndrome, Marfan syndrome, multiple endocrine neoplasias,
0 44 neurofibromatosis type l (von Recklinghausen disease), neurofibromatosis type 2, tuberous
0 45
sclerosis, von H ippei-Lindau disease.
• 46

6
lock
s
Suspend
0
End Block
Item: 42 of 64 ~ 1 • M k -<:J 1>- Jil ~· !:';-~
QIO: 1220 ..L ar Pre v ious Next Lab~lues Notes Calcula t o r

26
A

Researchers in a molecular biology laboratory are interested in studying gene expression in mice. They are surprised to find that in mice, although the
same gene codes for apolipoprotein B in the liver and the intestine, the version made in the liver is longer by 2411 amino acids than the protein
IA•A] A

27
translated in the Intestines.
28
29 What Is the most likely mechanism for this observation?

30 :
31 A. Dimerization

32 B. Framesh1ft mutation
33
c. Gene recombination
34
35
o. Proteasomal degradation

36 E. RNA Editing

37
38
39
40
41
• 42
. 43
• 44
• 45
. 46 •
a
Lock
s
Suspend
8
End Bl ock
Item: 42 of 64 ~ 1 • M k -<:J 1>- Jil ~· !:';-~
QIO: 1220 ..L ar Prev ious Next Lab~lues Notes Calculat o r

A A

26 Th e co rrect an sw er i s E. 5 60/o chose this.

27 RNA editing is a molecular process that allows cells to make modifications to specific
Apo BGene
nucleotide sequences. As illustrated in the diagram, a c to u editing of the mRNA for
28 apollpoprotein B takes place in the intestines, which changes a CAA codon to the stop codon
DNA strond
-UAA. Therefore Apo 648 is expressed in the intestines.
29 Apol poprotein B RNA editing Nucleotide Messenger RNA Apolipoprotein Genetic code RNA
30
31
Gastrointestinal tract
l 1'1-1 mRHA II rand

32
33
34 RtlA •di:ing changes a CPA to rlo RHAedllng
UM oauslng a STOP lnllead of
a Gin
35
Tratl!latlon
36
Apolipoprotein 6 48 ApolQ>oproteln B 100
37
38
39
Image courtesy of Wikimedia Commons
40
41 A is no t co rrect . 70fo ch o se this.
42 Dlmerlzatlon of a protein does not change the length of the protein, only the tertiary structure.
Protein tertiary structure Tertiary structure Dimer {chemistry) Protein dimer Protein
. 43
B i s n o t correct. 1 1 Ofo ch o se this .
• 44
Frameshlft mutations typically shorten the length of a gene instead of lengthen it. By shifting the entire sequence by 1-2 base pairs, the chance of a stop
• 45 codon occurring increases. Also, frameshift mutations would not exist stably within a genome; most are spontaneous.
Gene Stop codon Translational frameshift Frameshift mutation Genome Genetic code Base pair Mutat'on
. 46 •
a
Lock
s
Suspend
8
End Block
Item:42of64 ~. , . M k <:] t> al ~· ~
QIO: 1220 .l. ar Previous Next lab 'Vfl1 ues Notes Calculator

26 c is not correct. 16% chose this.


Different proteins can be produced by gene recombination, wherein homologous segments of DNA exchange base pairs during meiosis. In the case of
27 recombination, however; the length of the gene (and thus the size of the protein) remains the same.
Meiosis Gene Genetic recombination Protein Homology (biology) DNA Base pair
28
29 D is not correct. 10 % chose this.
Proteasomal degradation is a posttranslational modification in which ubiquitin marks defective proteins for breakdown. Proteasomal degradation of
30 apolipoprotein B would make that protein undetectable rather than just shortened.
Post-translational modification Apolipoprotein B Protein Ubiquitin Apolipoprotein Proteasome
31
32
33 Bottom Line:

34 Many changes occur before a gene becomes a protein. Know all the posttranscriptional modifications that can occur; including gene splicing and the
addition of the 5 ' cap and poly(A) tail.
35 Polyadenylation Gene Protein Recombinant DNA RNA splicing

36
37
38 I iii I;fi 1!1 I•J for year:[ 2017 "
FI RST AID FA CTS
39
40 FA17 p 38.4
41 RNA processing Initial tra nscript is called heterogeneous nuclear mRNA is transported out of the nucleus into the
42 (eukaryotes) R 'A (hnR A). hnR A is then modified and cytosol, where it is translated.
• 43 becomes mR lA. mRNA quality control occurs at cytoplasm ic
• 44
The following processes occur in the nucleus: processing bodies (P-bodies), which contain
Cap Coding
s· Capping of 5' end (add ition of cxonucleases, decapping enzymes, and

~
• 45
Gppp ( 6:. 7-methylguanosine cap) microR As; mR As may be stored in P-bodies
!
• 46 • ~· • Pnlv<~ rl Pnvbtinn nF ~~ P nrl (,. )()() A'o\

6
lock
s
Suspend
0
End Block
Item: 42 of 64 ~ 1 • M k -<:J 1>- Jil ~· !:';-~
QIO: 1220 ..L ar Pre v ious Next Lab~lues Notes Calcula t o r
A A

26 FA17 p 39.1
Lariat
27 Splicing of pre-mRNA 0 Primary transcript combines with small .--~A.____
28 nuclear ribonucleoproteins (sn RN Ps) and
29 other proteins to form spl iceosomc.
r
snRNPs-
'
30 E) Lariat-shaped (looped) intermediate is 0 ~ <!rGU -A-AG~
generated.
31
E) Lariat is released to precisely remo\e intron
32 and join 2 exons.
33 Antibodies to spliceosomal snR Ps (anti-
34 Smith antibodies) are highly specific for
35 SLE. Anti-Ul Rl P antibodies are highly
associated with mixed connective tissue
36
disease (r-. ICTD).
37
38
FA17 p 39.2
39
40 lntrons vs exons Exons contain the actual genetic informal'ion lntrons are intervening sequences and stay
cod ing for protein. in the nucleus, whereas cxons c:~. it and are
41
Jntrons are intervening noncoding segments of expressed.
42 DNA. Abnormal splicing variants are implicated in
• 43 Different exons are frequently combined by oncogenesis and many genetic disorders (eg,
• 44 alternative splicing to produce a larger number ~-thalassemia).
• 45 of unique proteins.
• 46 •
Exonl Exon2
-
Exon3 Exon4 ExonS Exon6

a
Lock
s
Suspend
8
End Bl ock
Item: 42 of 64 ~ 1 • M k -<:J 1>- Jil ~· !:';-~
QIO: 1220 ..L ar Pre v ious Next Lab~lues Notes Calcula t o r
A A

26
FA17 p 39.2
27
lntrons vs exons Exons conta in the actual genetic information Introns are intervening sequences and stay
28
coding for protein. in the nucleus, " hereas exons e'\it and are
29
lntrons are intervening noncoding segments of e'\pressed.
30 D A. Abnormal splicing ,·ariants are implicated in
31 Different exons are frequent I} combined by oncogenesis and many genetic disorders (eg,
32 alternati,·e splicing to produce a larger number ~-thaJ assemia).
of unique proteins.
33
34 Exon l Exon2 Exon3 Exon 4 Exon S Exon6
S' 3'
DNA 3' S'
35
36 1 Tran!.enptlon
1
I
37 hnRNA S' 3'
1 2 4 5 6

f"'
38
ato~e .phc1ng
Splicmg
39
( )
40 mRNA 5' 3' S' 3' 5' 3'
1 2 4 5 6 1 3 5 6 1 3 4 5 6
41
42
• 43
j'"""'"'" 1 1 1
Proteins
• 44

• 45
• 46 •
a
Lock
s
Suspend
8
End Bl ock
Item: 43 of 64
QIO: 2853 ~
~ , .1 • Mar k <1
Previous
t>
Next Lab
d\ ues
•.

Notes
~
calculator

26 A 23 -year-old woman presents to the emergency department because of vaginal bleeding , She says that she is in her ninth week of pregnancy according to her last
menstrual period , Laboratory studies show a 13- human chorionic gonadotropin level of 103,000 m!U/ L The sample shown in the image is retrieved from the patient's
27 uterus, There are no recognizable fetal parts ,
28
29
30
31
32
33
34
35
36
37
38
39
Which of the following descr ibes the most likely genotype and parental source of DNA in this mass?
40
41 :
A. 46,XX; maternal
42
• 43 B. 46,XX; paternal

• 44
c. 46,XX; maternal and paternal
• 45
D. 69,XXX; maternal and paternal
• 46
• 47 E. 69,XXY; maternal and paternal

• 48

a
Lock
s
Suspend
o
End Block
Item: 43 of 64 ~. I • M k <:] t> al ~· ~
QIO: 2853 .l. ar Previous Next lab 'Vfl1 ues Notes Calculator

26 The co rrect a nswer is B. 73% cho se this.


A hydatidiform mole is a noninva sive tumor caused by aberrant fertilization, lea ding to cystic swelling of chorionic villi and proliferation of the trophoblast.
27 Hydatidiform moles can be complet e or partial. The genotype of a complet e mole is usually 46,XX, complet ely consisting of pat ernal DNA. It results when
a haploid sperm fertilizes an empty ovum and then duplicat es. It can also, more rarely, occur when an empty ovum is fertilized by two sperm, lea ding to a
28
46,XX or 46,XY genotype. It results in a mass that can look like a "cluster of grapes," as seen in the image. Ultrasound may show a "snowstorm"
29 appearance of the uterus. There is no associat ed fetus despite the eleva t ed levels of 13-human chorionic gonadotropin (13 -hCG). With a complet e mole the
patient may present with symptoms such as pre -eclampsia, hyperemesis, enlarged uterus (grea t er in size than estimat ed for gestational age), first-
30 trimester bleeding and hyperthyroidism . Theca lutein cysts may also more commonly be present with a complet e mole. Malignant potential for a complet e
mole is higher than for a partial mole. Trea tment includes a dilation and curettage to eva cuat e all uterine contents. Serial 13-hCG levels must also be
31 followed for about 6 months. Levels should return to normal, plat eau, or rise in 13-hCG levels may indicat ed persistent or invasive disea se.
32 Pre-eclampsia Trophoblast Hyperthyroidism Dilation and curettage Haploid Molar pregnancy Chorionic villi Egg cell Uterus Medical ultrasound Gestational age
Ploidy Ultrasound Fertilisation Fetus Neoplasm Sperm Genotype Spermatozoon Pregnancy DNA Cancer Karyotype
33
A is no t co rrect. 8% cho se this.
34
Mat ernally derived 46,XX would not cause a hydatidiform mole. The question describes a complet e mole which is derived from " empty" ovum that is then
35 fertilized by sperm .
Molar pregnancy Sperm Egg cell Spermatozoon Karyotype
36
37
c is no t co rrect. 5 % cho se this.
46,XX describes the genotype of a normal fetus, receiving one set of chromosomes from ea ch parent.
38 Genotype Chromosome Karyotype Fetus

39 D is no t co rrect. 6 % cho se this.


40 Pa t ernally derived 69,XXX describes another possible DNA make-up of a partial mole. A partial mole contains more than two sets of chromosomes that
usually consist of two pat ernal and one mat ernal set of chromosomes, resulting in triploidy. Tetraploidy can also occur; typically with three pat ernal and
41 one mat ernal set of chromosomes. Partial moles may present with a similar grapelike mass but are also associat ed with fet al parts. Malignant potential
for partial moles is lower than for complet e moles.
42 Chromosome Polyploid DNA Mole (animal)
43
E is no t co rrect. 8% cho se this.
0 44 Mat ernally and pat ernally derived 69,XXY describes one possible DNA make-up of a partial mole. A partial mole contains more than two sets of
chromosomes that usually consist of two pat ernal and one mat ernal set of chromosomes, resulting in triploidy. Tetraploidy can also occur; typically with
0 45 three pat ernal and one mat ernal set of chromosomes. Partial moles may present with a similar grapelike mass and are associat ed with fet al parts.
0
46 Polyploid Chromosome Mole (animal) DNA

6
lock
s
Suspend
0
End Block
Item: 43 of 64 ~. I • M k <:] t> al ~· ~
QIO: 2853 .l. ar Previous Next lab 'Vfl1 ues Notes Calculator

26 Bottom Line:
27 Complete hydatidiform moles most commonly have a 46,XX genotype, and all DNA is paternally derived. They look like a "cluster of grapes" on
histologic examination and are not associated with the presence of fetal parts.
28
Genotype Mole (unit) Karyotype Histology DNA
29
30
31 lijj ;fi IJ l•l for year:l 2017 ..
FI RST AID FAC TS
32
33
FA11 p605.1
34
Hydatidiform mole Cystic swelling of chorionic villi and prol iferation of chorion ic epithelium (only trophoblast).
35
Presents with vaginal bleeding, uterine enlargement more than expected, pelvic pressure/pain.
36 Associated with hCG-mediated sequelae: early preeclampsia (before 20 weeks), theca-lutei n cysts,
37 hyperemesis gravida rum, hyperthyroidism.
38 Treatment: dilation and curettage and methotrexate. lonitor ~-hCG.
39 Complete mole Partial mole
40 KARYOTYPE 46,XX; 46,XY 69,XXX; 69,X>..'Y; 69,>..'YY
41
COMPONEN TS Most commonly enucleated 2 sperm + l egg
42 egg + single sperm
43 (subsequently duplicates
0 44 paternal D A)
0 45 FETAL PARTS No Yes (partial =fetal parts)
• 46 UTERINESIZE t
6
lock
s
Suspend
0
End Block
Item: 43 of 64 ~. I • M k <:] t> al ~· ~
QIO: 2853 .l. ar Previous Next lab 'Vfl1 ues Notes Calculator

• FA17 p 220.2 •
26
Serum tumor markers Tumor markers should not be used as the I0 tool for cancer diagnosis or screen in g. They may be
27
used to monitor tumor recurrence and response to therapy, but definitive diagnosis is made via
28
biopsy.
29 MARKER ASSOCIATED CANCER NOTES
30 Alkaline phosphatase Metastases to bone or liver, Paget disease of Must exclude hepatic origin by checking LFTs
31 bone, sem inoma (placental ALP). and CCT levels.
32 a -fetoprotein Hepatocellular carcinoma, hepatoblastoma, yolk Normally made by fetus. Transiently elevated in
33 sac (endodermal sinus) tumor, mixed germ pregnancy. High levels associated with neural
34 cell tumor. tube and abdominal wa ll defects, low levels
associated with Down syndrome.
35
~-hCG Hyda tidiform moles and C horiocarcinomas Produced by syncytiotrophoblas ts of the
36
(G estational trophoblastic disease), testicular placenta.
37
cancer, mixed germ cell tumor.
38
CA 15-3/CA27-29 Breast cancer.
39
CA 19-9 Pancreatic adenocarcinoma.
40
CA 125 Ovarian cancer.
41
Calcitonin Medullary thyroid carcinoma (alone and in
42
ME 2A, MEN2B).
43
CEA Ivlajor associations: colorectal and pancreatic C arcinoembryonic antigen. Very nonspecific.
0 44
cancers.
0 45 M inor associations: gastric, breast, and
0 46 • medullarv thvroid carcinomas. •

6
lock
s
Suspend
0
End Block
Item: 44 of 64 ~ 1 • M k -<:J 1>- Jil ~· !:';-~
QIO: 3378 ..L ar Pre v ious Next Lab~lues Notes Calcula t o r
A A

26 A 7-year-old boy has suffered from progressive muscle weakness since he was 3 years old. When rising from a seated or lying position, the boy
requires assistance from his upper extremities. The boy's uncle suffered from the same symptoms, and died at the age of 23 years.
27
28
What type of mutation is most likely responsible for this patient's condition?
29
:
30 A. Deletion
31
B. Duplication
32
33
c. Inversion

34 D. Point mutation

35 E. Robertsonian translocation
36
37
38
39
40
41
42
43
• 44
• 45
. 46 •
a
Lock
s
Suspend
8
End Bl ock
Item: 44 of 64 ~. I • M k <:] t> al ~· ~
QIO: 3378 .l. ar Previous Next Lab 'Vfll ues Notes Calculator

The co rrect a nswer is A. 72% cho se this.


26
The patient suffers from Duchenne's muscular dystrophy, a disea se that causes progressive muscle weakness and early dea th due to an accelerat ed rat e
27 of muscle breakdown . Patients begin suffering from weakness before the age of 5 years; weakness begins in pelvic girdle muscles and then proceeds
superiorly. Gowers' maneuver is described in the stem; patients use their upper extremities when rising from a sea t ed or supine position. Duchenne's
28 muscular dystrophy is an X-linked disorder that is caused most often by a frame -shift mutation that causes the failure to produce viable gene product.
29 Remember that frame -shift mutations can be caused by a deletion or an insertion of a number of nucleotides not divisible by 3. These mutations result in
misrea ding of all nucleotides downstream lea ding to the production of a shorter or a longer protein with an impaired functionality.
30 Frameshift mutation Duchenne muscular dystrophy Gowers' sign Gene Pelvis Muscular dystrophy Protein Mutation Sex linkage Nucleotide Muscle Rhabdomyolysis

31 B is no t co rrect. 6 % cho se this.


32 Gene duplications are rarer causes of Duchenne's muscular dystrophy (DMD ) than deletions. Duplications are seen in 5 -10 % of patients with DMD.
Gene Duchenne muscular dystrophy Muscular dystrophy Deletion (genetics)
33
c is no t co rrect. 3 % cho se this.
34 Gene inversions, in which a segment of a chromosome rotat es 18 0 degrees before rejoining with its respective fragments, are not usually a cause of
35 Duchenne's muscular dystrophy. Unlike deletions and duplications, inversions do not change the overall amount of genetic mat erial so inversions are
generally viable without a phenotypic effect. However; inversions can cause breaks in essential genes, introducing a lethal mutation, or lea d to
36 recombinant products with decrea sed viability or adverse phenotypic effects.
Gene Duchenne muscular dystrophy Chromosome Mutation Muscular dystrophy Phenotype Chromosomal inversion Recombinant DNA Genome Deletion (genetics)
37
DNA
38
D is no t co rrect. 1 7% cho se this.
39 Point mutations are single base changes in a single codon and can lea d to silent, missense, or nonsense mutations. Nonsense mutations in the dystrophin
40 gene can lea d to Duchenne's muscular dystrophy but they are not as common as frameshift mutations (caused by deletion/insertion ). Silent and missense
mutations are not common causes of Duchenne's muscular dystrophy.
41 Dystrophin Duchenne muscular dystrophy Gene Muscular dystrophy Nonsense mutation Frameshift mutation Translational frameshift Point mutation

42 Missense mutation Genetic code Mutation

43 E is no t co rrect. 2% cho se this.


Robertsonian translocations but are not usually the cause of Duchenne's muscular dystrophy. The robertsonian translocation is a rearrangement involving
44 the five acrocentric chromosome pairs ( 13, 14, 15, 21, and 22, i.e.), and occurs when the two long arms of the acrocentric chromosmoes fuse and the
• 45 short arms are lost. Unbalanced forms can lea d to inherited disea ses such as Down's syndrome and balanced carriers may experience recurrent fet al loss.
Down syndrome Duchenne muscular dystrophy Centromere Muscular dystrophy Acrocentric Chromosomal translocation Chromosome Genetic disorder
• 46

6
lock
s
Suspend
0
End Block
Item: 44 of 64 ~. I • M k <:] t> al ~· ~
QIO: 3378 .l. ar Previous Next Lab 'Vfll ues Notes Calculator

• •
26 Bottom Line:
27 Duchenne's muscular dystrophy, which produces weakness starting before age 5, is usually due to a frame -shift mutation .
Frameshift mutation Ouchenne muscular dystrophy Muscular dystrophy Mutation
28
29
30
Iiii I;fi IJ I•J for year:[ 2017 ..
31 FI RST AID FAC T S

32
FA17p57.1
33
Muscular dystrophies
34
Duchenne X-linked disorder typically due to framcshift Duchenne = deleted dystrophin.
35
or nonsense mutations ..... truncated or Dystrophin gene (DMD) is the largest
36 . · •.:.~
;:~· f,~cle fiber absent dystrophin protein ..... progressi,·e protein-coding human gene ..... t chance of
~ • I'
...... .. myofiber damage. Weakness begins in pelvic spontaneous mutation. Dystrophin helps
37
~.J"
38 ~ ,,....../( .
I: .-~ ,...
, . •'
. . . . · ·~ ~
,' ~ girdle muscles and progresses superiorly.
Pseudohypertrophy of calf muscles due to
anchor muscle fibers, primarily in skeletal and
cardiac muscle. It connects the intracellular
~~ . !f.\.w...(~·•. ...
39
..... . t fibrofatty replacement of muscle f.J. Wadd ling cytoskeleton (actin) to the transmembrane
40
41 ......
••
,
.
. !
a
gait. Onset before 5 years of age. Dilated proteins a- and P-clystroglycan, which are
cardiomyopathy is common cause of death. connected to the extracel lular matrix (ECM).
42 Loss of dystrophin results in myonecrosis.
43 t CK and aldolase are seen; genetic testing
44 confirms diagnosis.
• 45 Becker X-linkecl disorder typically due to non- Deletions can cause both Duchenne and
frameshift deletions in dystrophin gene Becker muscular dystrophies. Y, of cases ha,·e •
• 46 •

6
lock
s
Suspend
0
End Block
Item: 44 of 64 ~ 1 • M k -<:J 1>- Jil ~· !:';-~
QIO: 3378 ..L ar Pre v ious Next Lab~lues Notes Calcula t o r
I • '' I t I ;,

26
Gower sign-patient uses upper extremities lo
27 help stand up.
28 Classically seen in Duchen ne muscular
29 dystrophy, but also seen in other muscular
30
dystrophies and innammator) 111)0pathies (eg,
polymyositis). lofdo51s............,.
31
32
33
34 Q

35
36 FA17 p 56.4

37 X-linked recessive Ornithine transcarbamylase de ~ c ie n C}', Fabry Oblivious Female \\'ill O ften G ive ller Boys
38 disorders disease, \Viskotl- ldrich syndrome, Ocular ller x-Linked Disorders
39
albinism, G6PD defi ciency, l lunter syndrome,
Bruton agammaglobul in em ia, l lemoph ilia
40
A and B, Lcsch-Nyhan syndrome, Duehcnne
41 (and Becker) muscular dystrophy.
42 Lyonization - female carriers variably affected
43 depending on the pattern of inacl ivaiion of the
44
X chromosome carrying the mutant vs normal
gene.
• 45
. 46 •
a
Lock
s
Suspend
8
End Bl ock
Item: 45 of 64 ~ 1 • M k -<:J 1>- Jil ~· !:';-~
QIO: 4986 ..L ar Pre v ious Next Lab~lues Notes Calcula t o r
A A

26 A 28-year-old G1PO woman presents at 6 weeks' gestation because of vaginal bleeding. On physical examination the height of her uterine fundus Is
found to be consistent with a 12-week-sized uterus. Laboratory studies show a 13-human chorionic gonadotropin level of 120,000 miU/mL (normal
27 range at this gestational age is 1,080- 56,500 miU/mL) . Karyotype analysis of cells In the amniotic fluid revea ls a genotype of 46,XX.
28
29 What Is the origin of this patient's condition?

30 :
A. Maternal DNA only
31
32 B. Paternal DNA only

33 C. Union of maternal and paternal DNA


34
D. Vi ral DNA
35
36 E. Viral RNA

37
38
39
40
41
42
43
44
• 45
. 46 •
a
Lock
s
Suspend
8
End Bl ock
Item: 45 of 64 ~ 1 • M k -<:J 1>- Jil ~· !:';-~
QIO: 4986 ..L ar Prev ious Next Lab~lues Notes Calculat o r

A A

26 The correct a nsw er i s B. 750/o chose this.


27 This patient is presenting with a molar pregnancy. This condition typically presents with vaginal bleeding, a m arkedly elevated 13-hCG level, and a larger
than expected uterus. (At 6-weeks gestational age, the uterine fundus should not be palpable outside the pelvis.) The key to answering this question is
28 that the mole is 46,XX, making it a complet e mole, which is exclusively the product of paternal DNA. The m ajority of complete moles arise when an egg
has no chromosomes as a result of m eiotic error, the empty egg is fertilized by a normal X-bearing sperm, and the 23 paternally-derived chromosomes
29 duplicate to produce a 46,XX karyotype. In 2% of cases, complete moles progress to frank choriocarcinoma. In som e cases, the markedly elevated 13-hCG
30 levels give rise to symptoms of hyperthyroidism because of a shared subunit. The red arrows in the histologic sample on t he left below highlight
edematous molar villi characteristic of a hydat idiform molar pregnancy. Compare these edematous villi to t he well -developed vascular villi highlighted by
31 the green arrows in the figure on the right, which are assodated with normal pregnancy.
.aryotype C 10 ocarcinoma Hyperthyroidism Molar pregnancy Uterus Sperm Ch01101 IC v• Pe s Vaginal bleeding Meiosis Spermatozoon Histology
32
Gestational age Chromosome Edema Intestinal villus DNA Pregnancy Molar (tooth)
33
34
35
36
37
38
39
40
41
42
43 Left-hand image courtesy of Bousfiha N, et at. Right-hand image courtesy of Wikimedia Commons

44 A i s n o t correct. 12% chose this.


There Is no molar pregnancy generat ed exclusively from maternal DNA.
45
Molar pregnancy Molar (tooth) Pregnancy DNA
. 46 •
a
Lock
s
Suspend
8
End Block
Item: 45 of 64 ~. I • M k <:] t> al ~· ~
QIO: 4986 .l. ar Previous Next lab 'Vfl1ues Notes Calculator

c is no t co rrect. 11% cho se this.


26
Partial moles contain both m aternal and paternal DNA. Partial moles most often arise from the fertilization of a normal egg by two normal sperm, thus
27 containing both m aternal and paternal DNA. As such, karyotype analysis of a partial mole would revea l either 69,XXX, 69,XXY, or 69,XYY. Of note, the 13-
hCG level is far less elevated in partial mole, and the risk of progression to choriocarcinoma is much lower.
28 Karyotype Choriocarcinoma Sperm DNA Melanocytic nevus Fertilisation Egg
29
Dis no t co rrect. 1 % cho se this.
30 Although viruses can contribute to some cancers (eg, African Burkitt lymphoma), there is no evidence supporting a role in hydatidiform mole formation .
Burkitt' s lymphoma Molar pregnancy Virus lymphoma
31
E is no t co rrect. 1 Ofo cho se this.
32
There is no known role of viral RNA in complete mole formation .
33 RNA Mole (animal) Mole (unit)

34
35 Bo tto m Line:
36 Molar pregnancies most commonly m anifest with vaginal bleeding, a uterus that is larger than expected for gestational age, and abnormally high 13-hCG
37 levels. Complete moles have a karyotype of 46,XX or 46,XY, and they are exclusively the product of paternal DNA.
Karyotype Uterus Molar (tooth) Gestational age Molar pregnancy DNA Vaginal bleeding
38
39
40 lijj ;fi IJ l•l f o r year:l 2 0 1 7 ..
FI RST AID FAC T S
41
42
FA17 p605.1
43
Hydatidiform mole Cystic swell ing of chorionic villi and prol iferation of chorion ic epithelium (only trophoblast).
44
ru .•' ,·:;
.r. ,.. . .... .' Presents with vaginal bleeding, uterine enlargement more than expected, pelvic pressure/pa in.
- ~!. ....
~~ .
45 ' ,:
.... "' ..
' ...
' .' . :• • J. t - ,. Associated with hCG-mediated sequelae: early preeclampsia (before 20 weeks), theca-lutei n cysts,
• • .,. • J •
• 46 •
-.'\.. ~-~... ~'\. "-.._, M •
hvnPrPmP~i ~ or~virbn1m hvnPrt h vrn i{li~m

6
lock
s
Suspend
0
End Block
Item: 46 of 64 ~ 1 • M k -<:J 1>- Jil ~· !:';-~
QIO: 5 0 17 ..L ar Pre v ious Next Lab~lues Notes Calcula t o r

26
A

A 27-year-old man is brought to the emergency department after he was found shuffling unsteadily around a busy intersection for several hours. The
patient Is unreliable in providing his medical history. His vital signs are normal. On physical examination his liver edge is palpable 6 em below the
IA•A] A

27
costal margin, and truncal spider angiomata are noted. Ophthalmologic examination reveals corneal deposits at the limbus. His abdomen is soft and
28 nontender.

29
What Is the pathophysiology associated wit h t his disease?
30
31 :
A. Decreased secretion into the bile of a metal that forms free radicals
32
B. Decreased solubilization of a steroid
33
34 c. Gallbladder stasis and ischemia

35 D. Increased gastrointestinal absorption of a metal that forms free radicals


36
E. Obstruction at the gallbladder neck or cystic duct
37
38
39
40
41
42
43
44
45
. 46 •
a
Lock
s
Suspend
8
End Bl ock
Item: 46 of 64 ~ 1 • M k -<:J 1>- Jil ~· !:';-~
QIO: 5 0 17 ..L ar Prev ious Next Lab~lues Notes Calculat o r

A A

26 Th e co rrect a nswer i s A. 55% ch ose this.


The patient presents with several classic features of Wilson disease. Wilson disease Is a genetic condition
27 caused by an autosomal recessive mutation in the ATP7B gene found In the liver. This mutation results in
28 the Inability to secrete copper into the biliary system, leading to an accumulation of copper in several
organs, such as the brain, liver, and cornea. Subsequently, the patient presents with low serum copper
29 levels, and increase of "free" copper, leading to a build -up of copper In other parts of the body over time.
Copper Is a pro-oxidant that forms free radicals and damages tissues (eg, liver, eye, and basal ganglia). So
30 Its build-up manifests as Kaiser-Fieischer rings (corneal copper deposits at the limbus-as shown in the
Image), mlcronodular cirrhosis of the liver, basal ganglia degeneration (producing parkinsonian-like
31
symptomatology), and neuropsychiatric symptoms. Low serum ceruloplasmin concentrations are not
32 associated w1th the clinical symptoms of Wilson disease.
Ceruloplasmin Basal gang a Wilson disease protein Cornea Gene Cirrhosis Autosomao recessive Wilsons disease
33
Bile duct Radical (chemistry) Uver Blood plasma Autosome Dominance (genetics) Genetic disorder Mutation
34 Pro-oxidant Bile Copper Brain Neuropsychiatry Serum (blood)

35 Image copyright© 2012, Iranian J Psych


Behav Sci
36
37 B is not co rrect. 80/o chose t his.
38 Cholelithiasis, or gallstone formation, results from dysfunction of cholesterol ?a-hydroxylase activity, which leads to reduced solubilization of cholesterol
Into bile acids and an increase in cholesterol secretion into bile. The excess cholesterol then precipitates into insoluble crystals and forms gallstones.
39 Patients experience biliary colic and other complications (eg, pancreatitis and gallbladder infection). However, they do not generally present with central
nervous system and ophthalmic symptoms.
40 Gallstone Biliary colic Central nervous system Pancreatitis Gallbladder Cholesterol Bile Bile acid Colic Nervous system Horse colic
41 C is not co rrect. 60fo ch ose this.
42 Acalculous cholecystitis is caused by gallbladder stasis and ischemia that Induces inflammation and injury to the gallbladder wall, which may result In
fever, severe right upper quadrant pain, and Murphy sign. However, it Is not associated with central nervous system and ophthalmic symptoms.
43 Cholecystitis Gallbladder Central nervous system Quadrant (abdomen) Ischemia Murphy's sign Inflammation Nervous system Fever
44 0 i s n ot correct. 250/o ch ose this.
45 Hemochromatosis originates from a mutation of the HFE gene that results In Increased gastrointestinal absorption of iron, a metal that induces formation
of hydroxyl free radicals and damages tissues. The common triad of symptoms Is micronodular cirrhosis, diabetes, and skin pigmentation (bronze skin).
46 • However. it Is not associated with the centra l nervous svstem and the oohthalmlc svmotoms seen in this oatient .

a
Lock
s
Suspend
8
End Block
Item: 46 of 64 ~. I • M k <:] t> al ~· ~
QIO: 5017 .l. ar Previous Next lab 'Vfl1 ues Notes Calculator

26 D is no t co rrect. 25% cho se this.


Hemochromatosis originat es from a mutation of the HFE gene that results in increa sed gastrointestinal absorption of iron, a met al that induces formation
27
of hydroxyl free radicals and damages tissues. The common triad of symptoms is micronodular cirrhosis, diabet es, and skin pigmentation (bronze skin).
28 However; it is not associat ed with the central nervous system and the ophthalmic symptoms seen in this patient.
Iron overload Central nervous system Gene Cirrhosis Diabetes mellitus Mutation Hydroxyl Radical (chemistry) Nervous system Iron Pigment Gastrointestinal tract
29
Human gastrointestinal tract Human skin color
30
E is no t co rrect. 6 % cho se this.
31 Calculous cholecystitis is caused predominantly by obstruction of the gallbladder neck or cystic duct. This obstruction results in chemical irritation and
inflammation by stones in the gallbladder and may manifest as severe colicky pain and Murphy sign. However; this condition does not present with central
32 nervous system and ophthalmic symptoms.
33 Cholecystitis Cystic duct Central nervous system Gallbladder Renal colic Kidney stone Calculus (medicine) Murphy' s sign Nervous system Inflammation
Biliary colic
34
35
36 Bo tto m Line:

37 Wilson disea se is caused by a mutation in the ATP78 gene needed for copper to bind to ceruloplasm for copper secretion into the biliary system . The
disea se is characterized by basal ganglia degeneration (producing parkinsonian -like symptoms), elevat ed plasma copper levels, Kayser -Fleischer rings
38 (corneal copper deposits), micronodular cirrhosis of the liver; and neuropsychiatric symptoms.
Basal ganglia Wilson disease protein Wilson' s disease Gene Cirrhosis Blood plasma Bile duct liver Mutation Secretion Copper Bile Ganglion
39
40
41
I iii I;fi 1!1 I•J f o r yea r :[ 20 17 "
42 FI RST AI D FA CTS

43
FA17 p 378.1
44
45 Wilson disease Autosomal recessive mu tations in hepatocyte copper-transporti ng ATPasc (ATP7B gene;
(hepatolenticular chromosome 13) -+ ~ copper excretion into bile and incorporation into apoceruloplasmi n
46
' . o I I o lo
' .'
6
lock
s
Suspend
0
End Block
Item: 46 of 64 ~ 1 • M k -<:J 1>- Jil ~· !:';-~
QIO: 5 0 17 ..L ar Pre v ious Next Lab~lues Notes Calcula t o r
A A

26 FA17p210.1

27 Free radical injury Free radicals damage cells via membrane lipid peroxidation, protein modification, and 0 l
28 breakage.
29 Initiated via radiation exposure (eg, cancer therapy), metabolism of drugs (phase 1), redox reactions,
nitric oxide (eg, inAammation), transition metals, \VBC (eg, neutrophils, macrophages) oxidati,·e
30
burst.
31
Free radicals can be eliminated by scavenging enqmes (eg, catalase, superoxide dismutase,
32 glutathione peroxidase), spontaneous decay, antioxidants (eg. vitamins A, C, E), and certain metal
33 carrier proteins (eg, transferrin , ceruloplasmin).
34 Examples:
Oxygen toxicity: retinopathy of prematurity (abnormal \'aSeularization), bronchopulmonary
35
drsplasia, reperfusion injury after thrombolr tic therapy
36
• Drug/chemical toxicity: carbon tetrachloride and acetaminophen 0\·erdose {hepatotoxicity)
37 Metal storage diseases: hemochromatosis (iron) and Wilson disease (copper)
38
39 FA17 p60.1

40 Genetic disorders by CHROMOSOME SELECTED EXAMPLES

41 chromosome 3 von l lippci-Lindau disease, renal cell carcinoma


42 4 ADPKD (PKD2), achondroplasia, Huntington disease
43 5 Cri-du-chat syndrome, familial adenomatous polyposis
44 6 Hemochromatosis (JJFE)
45 7 Williams syndrome, cystic fibrosis
46 • 9 Friedreich ataxia
a
Lock Suspend
s 8
End Bl ock
Item: 47 of 64 ~ 1 • M k -<:J 1>- Jil ~· !:';-~
QIO: 3945 ..L ar Pre v ious Next Lab~lues Notes Calcula t o r
A A

27 A 22-year-old woman is brought to the doctor by her husband because of her Increasingly aberrant behavior. She has become progressively forgetful
and has exhibited worsening jerky movements. Her mother and grandfather were also afflicted by similar symptoms at the ages of 33 and 49,
28
respectively.
29
30 Which of the following disorders exhibits the same genetic mechanism of symptom onset as the disease affecting this patient?
31 :
32 A. Alzheimer disease

33 B. Multiple sclerosis
34
c. Myoclonic epilepsy with ragged red fibers
35
D. Myotonic dystrophy
36
37 E. Parkinson disease

38
39
40
41
42
43
44
45
46
. 47 •
a
Lock
s
Suspend
8
End Bl ock
Item:47of64 ~. , . M k <:] t> al ~· ~
QIO: 3945 .l. ar Previous Next lab 'Vfl1 ues Notes Calculator

27
The co rrect a nswer is D. 70% cho se this.
28
The patient has Huntington disea se, an autosomal dominant disea se caused by abnormal expansion of CAG trinucleotide repea ts in the HTT gene on
29 chromosome 4, which lea ds to atrophy of the caudat e (indicat ed by the red circles in the image), putamen, and globus pallidus. Physical symptoms
include chorea (jerky movements), muscle rigidity, and dementia. The increa singly early age of onset with ea ch generation is known as a ntici pa tio n,
30 which is due to the expansion of the pathologic trinucleotide repea ts with ea ch successive generation . Other genetically acquired disea ses that are caused
31 by trinucleotide repea t expansion include myotonic dystrophy, Friedreich at axia, and fragile X syndrome.
Fragile X syndrome Friedreich's ataxia Trinucleotide repeat disorder Huntington's disease Putamen Globus pallidus Myotonic dystrophy Gene Dominance (genetics)
32 Trinucleotide repeat expansion Chorea Ataxia Huntingtin Chromosome 4 (human) Dementia Autosome Chromosome Hypertonia Caudate nucleus Atrophy
33
A is no t co rrect. 7% cho se this.
34 Alzheimer disea se is the most common cause of dementia in the elderly and results from a diffuse loss of neurons in the cerebral cortex, especially in the
t emporal, pariet al, and frontal lobes. On imaging, there is often gross cerebra l atrophy. The underlying pathophysiology is relat ed to abnormal j3 -amyloid
35 deposition and neurofibrillary t angles. In Down syndrome patients (trisomy 21 ), the disea se appears at an earlier age than in the general population .
36 However; it does not demonstrat e anticipation .
Down syndrome Cerebral cortex Cerebral atrophy Neurofibrillary tangle Dementia Alzheimer's disease Frontal lobe Neuron Atrophy Parietal lobe Pathophysiology
37
Trisomy
38
B is no t co rrect. 7% cho se this.
39 Multiple sclerosis is the most common acquired demyelinating disea se. The pathogenesis is relat ed to autoantibodies and CDS T lymphocytes directed
against myelin shea ths and oligodendrocytes. It does not exhibit anticipation .
40 Multiple sclerosis Demyelinating disease Myelin Autoantibody Oligodendrocyte lymphocyte T cell Pathogenesis CDS
41
c is no t co rrect. 9 % cho se this.
42 Myoclonic epilepsy with ragged red fibers (MERRF) shows no anticipation and exhibits mat ernal inheritance as a result of transmission via mitochondrial
DNA. Because egg cells contribute mitochondria to the developing embryo, only women can pass mitochondrial conditions to their children.
43
Mitochondrion Mitochondrial DNA Epilepsy Mitochondrial disease Embryo Myoclonus Non-Mendelian inheritance MERRF syndrome DNA Egg cell Egg
44
E is no t co rrect. 7% cho se this.
45 Parkinson disea se is a degenerative disea se caused by loss of dopaminergic neurons in the striat al system . This disea se does not exhibit anticipation .
Striatum Parkinson's disease Dopaminergic Neuron Degenerative disease
46
47

6
lock
s
Suspend
0
End Block
Item:47of64 ~. , . M k <:] t> al ~· ~
QIO: 3945 .l. ar Previous Next lab 'Vfl1 ues Notes Calculator

27
Bottom Line:
28 Anticipation is a phenomenon in which the age of disease onset decreases with each generation. Anticipation is seen in trinucleotide repeat diseases
29 such as Huntington disease, myotonic dystrophy, and Fried reich ataxia.
Friedreich' s ataxia Huntington' s disease Myotonic dystrophy Ataxia Trinucleotide repeat disorder
30
31
32 I iii I;fi 1!1 I•J for year:[ 2017 "
FIRST AID FAC T S
33
34
FA17 p 52.2
35 Genetic terms
36 TERM DEFINITION EXAMPLE
37 Codominance Both alleles contribute to the phenotype of the Blood groups A, B, AB; a 1-antitrypsin
38 heterozygote. deficiency.
39 Variable expressivity Patients with the same genotype have varying 2 patients with neurofibromatosis type 1 ( JFl)
40 phenotypes. may have va rying disease severity.
41 Incomplete ot all individuals with a mutant genotype BRCA .l gene mutations do not always result in
42 penetrance show the mutant phenotype. breast or ovarian cancer.
43 Pleiotropy One gene contributes to multiple phenotypic Untreated phenylketonuria (PKU) manifests with
44 effects. light skin, intellectual disability, and musty body
45 odor.
46 Anticipation Increased severity or earlier onset of disease in Trinucleotide repeat diseases (cg, Huntington
47 succeeding generations. disease).

6
lock
s
Suspend
0
End Block
Item: 47 of 64 ~ 1 • M k -<:J 1>- Jil ~· !:';-~
QIO: 3945 ..L ar Pre v ious Next Lab~lues Notes Calcula t o r
A A

27 FA17 p55.1

28 Modes of inheritance
29 Autosomal dominant Often due to defects in structural genes. tvlany Often pleiotropic (multiple apparent!) unrelated
30
generations, both males and females are effects) and variably expressive (different
affected. between individuals). Family history crucial
31
to diagnosis. With one affected (hetero.l) gous)
32 parent. on average, V2 of children affected.
33
34
35
Autosomal recessive Often due to enzrme deficiencies. Usuall) seen Commonly more se,·ere than dominant disorders;
36
in only I generation. patients often present in ch ildhood.
37 f risk in consanguineous families.
38 With 2 carrier (heterozygous) parents, on average:
39 V.. of children will be affected (homozygous),
40
Yz of children will be carriers, and \4 of children
will be neither affected nor carriers.
41
42 X-linked recessive Sons of heterozygous mothers have a 50% Common ly more severe in males. Females
43 chance of being affected. o ma le-to-male u~ua l l y must be homozygous to be affected.

44 transmission. Skips generations.


45
46
47 •
a
Lock
s
Suspend
8
End Bl ock
Item: 47 of 64 ~ 1 • M k -<:J 1>- Jil ~· !:';-~
QIO: 3945 ..L ar Pre v ious Next Lab~lues Notes Calcula t o r
A A

27 Mitochondrial Transmitted only through the mother. Al l ariable expression in a population or even
28
inheritance offspring of affected females may shO\\ signs of within a family due to heteroplasmy.
disease.
29 Mitochondrial myopathies- rare disorder~;
30 often present with myopathy, lactic acidosis,
31
and C1 disease, eg, \I ELA syndrome
(mitochondrial encephalopathy, lactic acidosis,
32
and stroke-like episodes). 2° to failure in
33 oxidati\·e phosphorylation. ~luscle biopsy often
34 shows '·ragged red fibers" (due to accumulation
35 of diseased mitochondria).
36 0 = unaffected male; • =affected male; 0 = unaffected female; e =affected female.
37
38 FA17 p 58.2

39 Trinucleotide repeat Huntington disease, myotonic dystrophy, Tr) (trinucleotide) hunting for my fragile cage-
40 expansion diseases fragile X syndrome, and Fried reich ataxia. free eggs (X).
41
May show genetic anticipation (disease severity
f and age of onset ! in successive generations).
42
Huntington disease = (CAG), Caudate has ! ACh and GABA
43 lyotonic dystrophy = (CTC)., C ataracts, Toupee (early balding in men),
44 Gonada l atrophy
45 Fragile X syndrome = (CGC)11 C hin (protruding}, Giant Gonads
46 F'riedreich ataxia = (CAA)., Ataxic C \\it
47 •
a
Lock
s
Suspend
8
End Bl ock
Item: 48 of 64 ~ 1 • M k -<:J 1>- Jil ~· !:';-~
QIO: 4865 ..L ar Pre v ious Next Lab~lues Notes Calcula t o r
A A

28 A couple brings their 2-year-old daughter to the clinic for the first time for a well-child check-up . Physical examination revea ls short stature, a flat
29 mldface with a prominent forehead, and shortening of the limbs. The child's father exhibits similar physical features.

30
What Is the most likely cause of this child's condition?
31
:
32
A. Autosomal dominant cell -signaling defect of fibroblast growth factor receptor 3
33
B. Autosomal dominant defect in fibrillin 1
34
35 c. Autosomal recessive cell -signaling defect of fibroblast growth factor receptor 3

36 D. Autosomal recessive defect in fibrillin 1


37
E. X-llnked recessive defect in fibroblast growth factor receptor 3
38
39
40
41
42
43
44
45
46
47
. 48

a
Lock
s
Suspend
8
End Bl ock
Item: 48 of 64 ~ 1 • M k -<:J 1>- Jil ~· !:';-~
QIO: 4865 ..L ar Prev ious Next Lab~lues Notes Calculat o r

A A

28 Th e co rrect a nswer i s A. 780/o ch ose this.


Achondroplasia is a defect of cartilage-derived bone caused by a mutation In the fibroblast growth factor receptor 3 ( FGFR3) gene. FGFR3 Is expressed In
29
early human development in the cartilage growth plates of long bones during endochondral ossification. Eighty percent of cases result from spontaneous
30 mutations; remaining cases are inherited in an autosomal dominant fashion, as in this patient. Achondroplasia leads to numerous phenotypic
abnormalities including short stature, macrocephaly, midface hypoplasia with prominent forehead, and shortening of t he limbs (rhizomelia, preferentially
31 affecting the proximal segments). Achondroplasia occurs in approximately 1 In 12,000 births and is the most common bone dysplasia in humans.
Rhizome! a Achondroplasia Endochondral ossification Fibroblast Fibroblast growth facto eceptor 3 Gene Macrocephaly Dominance (genetics) Ossification
32
Fibroblast growth factor receptor cartilage Growth factor Fibroblast growth factor Autosome Epiphyseal plate Bone Mutation Dysplasia
33
Human development biology) Long bone Phenotype Hypoplasia Receptor (biochemistry) Short stature
34
B is not correct. 50/o chose this.
35 An autosomal dominant mutation in fibrillin 1 is believed to cause Marfan syndrome, not achondroplasia. Rbrillin 1 is an essential element of the
myoflbrillar array of connective tissue. Characteristic clinical features include marfanoid body habitus: tall stature, arachnodactyly, pectus deformity,
36
disproportionately long extremities in comparison to the length of the trunk (le, dolichostenomelia), and joint hypertaxity. Spinal abnormalities (scoliosis
37 and kyphosis) may be present, and the majority of patients experience upward ectopia lentis. Mitral valve prolapse and aortic root dilation are common
cardiac conditions.
38 Marfan syndrome FBNl Achondroplasia Kyphosis Ectopia lentis Mitral valve prolapse Dominance (genetics) Fibrillin Oolichostenomelia Hypermobility (joints)

39 Connective tissue Pectus excavatum Arachnodactyly Mutation Autosome Aorta Marfanoid Mitral valve Prolapse

40 C is not co rrect. 100/o chose t his.


Achondroplasia Is an autosomal dominant disorder. Autosomal recessive mutations have been associated with an extremely rare syndrome known as the
41
camptodactyly, tall stature, scoliosis, and hearing loss syndrome (CATSHL), which Is an interesting but relatively low-yield topic for the USMLE.
42 Achondroplasia Scoliosis Dominance (genetics) Camptodactyly Autosomal recessive Recessive Autosome Mutation

43 D is not co rrect. 20/o ch ose this.


An autosomal dominant mutation in fibrillin 1 is believed to cause Marfan syndrome. Currently, no known syndromes are associated with an autosomal
44 recessive mutation in the gene that produces fibrillin 1 (FBNl ).
45 Marfan syndrome Fibrillin Gene Dominance (genetics) FBNl Mutation Autosome Recessive Autosomal recessive

46 E i s n ot correct. 50/o ch ose this.


Achondroplasia is an autosomal dominant disorder. No known syndromes are associated with X-linked recessive defects in fibroblast growth factor
47
receptor 3 (FGFR3).
48 Achondrop asia Fibroblast Fobroblast growth factor receptor 3 Domonance (genetocs) Fibroblast growth factor receptor Fibroblast growth factor Autosome Recessive

a
Lock
s
Suspend
8
End Bl ock
Item: 48 of 64 ~. I • M k <:] t> al ~· ~
QIO: 4865 .l. ar Previous Next lab 'Vfl1ues Notes Calculator

• An autosomal dominant mutation in fibrillin 1 is believed to cause Marfan syndrome. Currently, no known syndromes are associated with an autosomal
28
recessive mutation in the gene that produces fibrillin 1 ( FBNl ).
29 Marfan syndrome Fibrillin Gene Dominance (genetics) FBNl Mutation Autosome Recessive Autosomal recessive
30 E is not correct. 5 % chose this.
31 Achondroplasia is an autosomal dominant disorder. No known syndromes are associated with X-linked recessive defects in fibroblast growth factor
receptor 3 ( FGFR3).
32 Achondroplasia Fibroblast Fibroblast growth factor receptor 3 Dominance (genetics) Fibroblast growth factor receptor Fibroblast growth factor Autosome Recessive
33 Growth factor Sex linkage X-linked recessive inheritance Receptor (biochemistry)

34
35 Bottom Line:
36 Achondroplasia is caused by an autosomal dominant mutation in the fibroblast growth factor receptor 3 gene.
Achondroplasia Fibroblast Fibroblast growth factor receptor 3 Gene Dominance (genetics) Fibroblast growth factor receptor Mutation Fibroblast growth factor
37 Growth factor Autosome Receptor (biochemistry)
38
39
40 I ill ;fi 1!1 I•J for year:[ 2017 ..
FI RST AI D FA CTS
41
42
FA11 p435.1
43
Achondroplasia Failure of longitudinal bone growth (endochondral ossification) -+ short limbs. ~embranous
44 ossification is affected -+ large head relative to limbs. Constitutive acti,·ation of fibroblast growth
45 fac tor receptor (FGFR3) actually inhibits chondrocyte prol iferation. > 85% of mutations occur
46 sporadically; autosomal dominant with full pe netrance (homozygosity is lethal). Most common
47 cause of dwa rfism.
48 •

6
lock
s
Suspend
0
End Block
Item: 49 of 64 ~ 1 • M k -<:J 1>- Jil ~· !:';-~
QIO: 3341 ..L ar Pre v ious Next Lab~lues Notes Calcula t o r

29
A

Acute promyelocytic leukemia (APL)- also referred to as type 3 acute myelogenous leukemia (AML)- is characterized by a translocation event
resulting In the fusion of the PML gene on chromosome 15 with another gene on chromosome 17. Treatment for this condition results in maturation of
IA•A] A

30
leukemic cells and clinical remission .
31
32 What Is the normal product of the gene on chromosome 17?

33 :
34 A. BCR (breakpoint duster reg ion)

35 B. C-myc
36
c. Estrogen
37
38
o. Immunoglobulin heavy chain (IgH)

39 E. Ras
40 F. Retlnoic acid receptor
41
G. V-Src
42
43
44
45
46
47
48
• 49 •
a
Lock
s
Suspend
8
End Bl ock
Item: 49 of 64 ~ 1 • M k -<:J 1>- Jil ~· !:';-~
QIO: 3341 ..L ar Previous Next Lab~lues Notes Calculat or
A A

29 Th e co rrect an sw er i s F. 560/o chose this.


30 APLis associated with the t( l S; 17 ) translocat ion involving PM L and the retlnolc acid receptor. Treatment wit h retinoic acid frequently leads to clinical
remission.
31 Retinoic ac d Retinoic ac•d receptor Chromosomal translocation Receptor (biochem•stry)

32 A i s not correct. 90/o ch o se t his.


33 The BCR gene forms a portion of t he BCR-ABL f usion protein that results from the Philadelphia chromosom e t ranslocation [t 9; 22)]. This genetic
abnormality IS found in nearly all ca ses of chronic myelog enous leukemia, but Is not 1nvolved in APL
34 Chronic myeloge oous leu :emoa Chromosomal translocation Philadelphia chromosome Geo e Leul,emia Fusion protein Protein BCR (gene) Chromosome

35 Protein targeting

36 B i s n ot correct. 10 0/o chose thi s.


C-myc Is a protooncogene that promotes tumor formation when its activit y is unregu lated via translocation. Fo r example, translocation between c-myc
37 and the Immunoglobulin heavy chain locus t(8; 14) results in Burkitt 's lymphoma.
38 Burkitt's lymphoma Myc Protooncogene Antibody Locus (genetics) Lymphoma Neoplasm Chromosomal translocation Immunoglobulin heavy chain Heavy chain

39 C is n ot co rrect. 30/o chose t h is.


Estrogen Is requ ired for the growth of certain kinds of breast cancer. It Is not associated with APL
40 Estrogen Breast cancer Cancer
41
0 is not co rrect . 90/o chose this.
42 The lgH locus is a common site for t ranslocat ions leading to ca ncers, Including Burkitt's lymphoma [t(8; 14) ], m ant le cell lymphom a [t ( 11 ; 14) ], and
follicular lymphoma [t ( 14; 18) ]. It is not, however~ involved in APL.
43
Mantle cell lymphoma Burkitt's lymphoma Follicular lymphoma Lymphoma Chromosomal translocation Locus (genetics)
44
E i s n o t co rrect . 100/o ch o se this.
45 Ras Is located at the plasma m embra ne and has GTP binding and GTPase abilities. Mitogenic signaling via Ras is increased in m any cancers, but Ras Is not
associated with translocations or APL
46
Cell membrane GTPase Guanosine triphosphate Mitogen Blood plasma Chromosomal translocation Ras subfamily
47
G i s n ot correct. 30/o ch ose this.
48 The v-src oncogene is found in t he Rous sa rcoma virus of chickens. I t Is not responsible for human disea se.
Rous sarcoma virus Oncogene Sarcoma Virus
49 •
a
Lock
s
Suspend
8
End Block
Item:49of64 ~- , . M k <:] t> al ~· ~
QIO: 3341 .l. ar Previous Next lab 'Vfl1ues Notes Calculator
• •
29
Bo tto m Line:
30
The retinoic acid receptor is produced by the gene on chromosome 17 that is involved in APL, which can be treated with retinoic acid .
31 Retinoic acid Gene Retinoic acid receptor Chromosome 17 (human) Chromosome Receptor (biochemistry)

32

Iiii I;fi IJ I•J for year:[ 2017 ..


33
34
FI RST AI D FAC TS
35

36 FA17 p412.1
37 Chromosomal translocations
TRANSLOCATION ASSOCIATED DISORDER
38

39
t(8;14) Burkitt lymphoma (c-myc activation)

40
t(9;22) (Philadelphia CML (BCR-A BL hybrid), ALL (less common, Philadelphia C real\IL cheese.
chromosome) poor prognostic factor) The Ig heavy chain genes on chromosome 14
41
are constitutively expressed. When other
42
genes (eg, c-myc and BCL-2) are translocated
43 next to this heavy chain gene region, they are
44 overexpressecL
45 t(ll;l4) Iantlc cell lymphoma (cyclin Dl activation)
46 t(l4;18) Follicular lymphoma (BCL-2 activation)
47 t(l5;17) APL (M3 type of AML) Responds to all-trans retinoic acid.
48
49 • •

6
lock
s
Suspend
0
End Block
Item:49of64 ~. , . M k <:] t> al ~· ~
QIO: 3341 .l. ar Previous Next lab 'Vfl1ues Notes Calculator
• •
29 FA17 p410.1

30 Leukemias Unregulated growth and differentiation of WBCs in bone marrow - marrow failure - anemia
31 (l RBCs), infections (I mature WBCs}, and hemorrhage (I platelets). Usually presents with
32
f circulating WBCs (malignant leukocytes in blood); rare cases present with normal/! WBCs.
Leukemic cell infiltration of liver, spleen, lymph nodes, and skin (leukemia cutis) possible.
33
TYPE NOTES
34
Lymphoid neoplasms
35
Acute lymphoblast ic Most frequently occurs in children; less common in adults (worse prognosis). T-cell ALL can
36 le uke mia/ lymphoma present as mediastinal mass (presenting as SVC-like syndrome). Associated with Down syndrome.
37 Peripheral blood and bone marrow have f f f lymphoblasts fl.
38 TdT+ (marker of pre-1' and pre-B cells}, CDIO+ (marker of pre-B cells).
39 Most responsive to therapy.
May spread to C 1$ and testes.
40
t(l2;21) - better prognosis.
41
Chronic lymphocyt ic Age: > 60 years. Most common adult leukemia. C D20+, CD23+, CD5+ B-cell neoplasm. Often
42
leuke mia/small asymptomatic, progresses slowly; smudge cells in peripheral blood smear; autoimmune rn
43 lymphocyt ic hemolytic anemia. CLL = Crushed Little Lymphocytes (smudge cells).
44 lymphoma Richter transformation-CLLISLL transformation into an aggressive lymphoma, most commonly
45 diffuse large B-celllymphoma (DLBCL).
46 Hairy cell le ukemia Age: /\dull males. Mature B-cell tumor. Cells have fi lamentous, hair-like projections
47 {fuzzy appearing on LM (!!). Peripheral lymphadenopathy is uncommon.
Causes marrow fibrosis - dry tap on aspiration. Patients usually present with massi,·e splenomegaly
48
and pancytopenia.
49 • <::.._,;..,"TRAP / h.,rf.r.-.f-.o, r .oC":c-h •l 1"ll- ..,,..;,..I "''""".,h . .,t-.-.r£>' r.I:\ T R AP r f.., ;n lt'lrrul.lu ro n,l •·u •or11u :4--l, l=l n,u •

6
lock
s
Suspend
0
End Block
Item: 49 of 64 ~ 1 • M k -<:J 1>- Jil ~· !:';-~
QIO: 3341 ..L ar Pre v ious Next Lab~lues Notes Calcula t o r

29
A
Treatment: cladribine, pentostat in. A

30 Myeloid neoplasms
31 Acute myelogenous ledian onset 65 years. Auer rods[!]; 111} eloperoxidase EE> cytoplasmic inclusions seen mostly in
leukemia PL (formerly 13 \1 L); t t t circulating myeloblasts on peripheral smear; adults.
32
Risk factors: prior exposure to alkylating chemotherapy, radiation, myeloproliferati\'e disorders,
33
Down syndrome. APL: t(l5;17), responds to all-trans rctinoic acid (\'itamin A), inducing
34
differentiation of promyelocytes; DIC is a common presentation.
35
Chronic myelogenous Occurs across the age spectrum \\'ilh peak incidence 45-85 years, median age at diagnosis 6-f years.
36 leukemia Defined by the Philadelphia chromosome (t[9;22], BCR-ABL) and myeloid stem cell proliferation.
37 Presents with dysregulated production of mature and maturing granulocytes (eg, neutrophils,
38 metamyelocytes, myelocytes, basophils ) and splenomegal}. May accelerate and transform to
39
VIL or ALL (" blast crisis").
Very low LAP as a result of low activity in malignant ncutrophils (\'S benign neulroph ilia
40
[leukemoid reaction], in wh ich LAP is t).
41 Responds to bcr-abl tyrosine kinase inhibitors (eg, imatinib).
42 m--lr- .. ,.,
43 .o .·...:o
/..,_,
44
45
46

47
48
49 •

Lock
a s
Suspend
8
End Bl ock
Item: 50 of 64 ~ 1 • M k -<:J 1>- Jil ~· !:';-~
QIO: 3844 ..L ar Pre v ious Next Lab~lues Notes Calcula t o r

30
A

A 18-month-old child is brought to the office for fever, irritability and lethargy for the past month. Over the past 3 month, he had abdominal
discomfort and has gained little weight . Physical exam shows a firm nodular mass in the left flank. He is being evaluated by a pediatric oncologist for
lA• A] A

31
a recently diagnosed malignancy. Genetic testing revea ls amplification of the N-myc gene.
32
33 Assuming there has been no metastasis, where will the malignancy most likely be found?
34 :
35 A. Adrenal medulla

36 B. Colon
37
c. Kidney
38
39
o. Spleen

40 E. Stomach

41
42
43
44
45
46
47
48
49
• so •
a
Lock
s
Suspend
8
End Bl ock
Item: 50 of 64 ~ 1 • M k -<:J 1>- Jil ~· !:';-~
QIO: 3844 ..L ar Prev ious Next Lab~lues Notes Calculat o r

A A

30 The co rrect a nsw er i s A. 52% ch ose this.


N-myc Is an oncogene, the product of which functions as a transcription factor. A mutation causing amplification of t his gene can result in neuroblastoma .
31 Neuroblastoma, a malignancy derived from neural crest cells, is the second most common childhood malignancy and is found most commonly In the
adrenal medulla .
32
Adrenal medulla Neuroblastoma Transcription factor Gene Oncogene Neural crest Malignancy Mutation Transcription (genetics) cancer Medulla oblongata
33 N-terminus Myc
34 B i s not correct. 8% chose this.
35 Colon cancer has a myriad of assodations: fa milial a denomatous polyposis (FAP, a mutation on the APC g ene), KRAS, RAF, p 53 mutations, and hereditary
nonpolyposls colorectal cancer (Lynch syndrom e; a d efect in DNA methylmlsmatch repair genes) are among t he most common.
36 Famoha adenomatous polyposis Hereditary nonpolyposis colorectao cancer PSJ .RAS Gene Colorectal cancer Colon (anatomy) Mutation Adenomatous polyposis co,
37 Cance DNA

38 C i s no t correct. 27% ch ose this.


39 Wilms tumor, the most common primary renal t umor in children, often Is associated with a deletion or mutation of the WTl tumor-suppressor gene at
llpl3. Deletions or mutations of genes at t he l lp lS.S locus also are associated with Wilms tumor. N-myc amplification is not found in Wilms tumors.
40 WTl Wilms' tumor Gene Mutation Neoplasm Tumor suppressor gene Locus (genetics) Kidney N·terminus Myc

41 D is no t co rrect . 9 0/o chose this.


42 Burkitt lymphoma often is associated wit h infection wit h Epstein -Barr virus. It Is associated most commonly wit h a t (8; 14) t ranslocation and production of
the c-MYC oncogene.
43 Burkitt's lymphoma Epstein-Barr virus Oncogene Myc Lymphoma Chromosomal translocation Virus Infection

44 E is no t co rrect. 40/o ch ose this.


45 Gastrointestinal stomal tumors (GIST) are commonly driven by mutations In the c-KIT gene, not dealing wit h any translocat ions.
Gene Human gastrointestinal tract CDl l7 Gastrointestinal stromal tumor Chromosomal translocation Gastrointestinal tract Neoplasm Mutation
46
47
Bottom Line:
48
Mutations resulting in amplification of N-myc, an oncogene, can result In neuroblastoma, a malignancy derived from neural crest cells that Is found
49 commonly In the adrenal medulla.
Adrenal medul a Neuroblastoma Neural crest Oncogene Malignancy Cancer Mutation Medul a oblongata N-terminus Adrenal gland Myc
so •
a
Lock
s
Suspend
8
End Block
Item: 50 of 64 ~ 1 • M k -<:J 1>- Jil ~· !:';-~
QIO: 3844 ..L ar Pre v ious Next Lab~lues Notes Calcula t o r
-· • • •• · - · ••••• - 10- - w •• - - • • • • • • • • •• • •• ·-

30
31
32 ljl;fil!1j•J for year: 2017 ...
FIRST AID FACTS
33
34
FA17 p 325.2
35
Neuroblastoma i\ lost common tumor of the adrenal medulla in children, usually < 4 years old. Originates from
36
neural crest cells. Occurs an)" here along the S} mpathetic chain.
37 lost common presentation is abdominal distension and a firm, irregular mass that can cross the
38 midline (,·s Wilms tumor, which is smooth and unilateral). Less likely to de,·elop hypertension
39 than with pheochromocytoma. Can also prcscut with opsoclonus-myoclonus syndrome ("dancing
40 eyes-dancing feet").
t HVA and VNIA (catecholamine metabolites) in urine. Homer-Wright rosettes · characteristic of
41
neuroblastoma and medulloblastoma. B01nbcsin and SE ®. Associated with O\'Crcxprcssion of
42 N-myc oncogene. Classified as an PUD tumor.
43
44
45
46
47
48
49
so •
a
Lock
s
Suspend
8
End Bl ock
32
33 FA17 p218.1
34 Oncogenes Cain of function .... t cancer risk. Need damage to only one allele of an oncogene.
35 GENE GENE PRODUCT ASSOCIATED NEOPLASM
36 ALK Receptor tyrosine kinase Lung adenoca rcinoma
37 BCR-ABL Tyrosine kinase CML, ALL
38 BCL-2 Antiapoptotic molecule (inhibits apoptosis) Follicular and diffuse large B cell lymphomas
39 BRAF Serine/threonine kinase Melanoma, non-l lodgkin lymphoma, papillary
40 thyroid carcinoma
41 c-K/T Cytokine receptor Gastrointestinal stromal tumor (CIST)
42 c-MYC Transcription factor Burkitt lymphoma
43
HER2/ neu (c-erb82) Receptor tyrosine kinase Breast and gastric carcinomas
44
JAK2 Tyrosine kinase Chronic myeloproliferati,·c d isorders
45
KRAS CTPase Colon cancer, lung cancer, pancreatic cancer
46
MYCL1 Transcription factor Lung tumor
47
MYCN Transcription factor N euroblastoma
48
RET Receptor tyrosine kinase ME 2A and 2B, medullary thyroid cancer
49
50

6
lock
s
Suspend
0
End Block
Item: 51 of 64 ~ 1 • M k -<:J 1>- Jil ~· !:';-~
QIO: 120 9 ..L ar Pre v ious Next Lab~lues Notes Calcula t o r
A A

31 Escherichia coli is able to grow on a medium containing lactose alone. No glucose or other sugars are present in the growth medium. In these
situations, the bacteria must rely solely on lactose for energy. A special gene pathway, called the lac operon, allows E. coli to accomplish this task.
32
33
Which of the following occurs when E. coli is plat ed on medium containing only lactose?
34
:
35 A. Allolactose binds to the repressor
36
B. The levels of cAMP are decreased and the repressor is inactive
37
38 c. The levels of cAMP are increased and the repressor is active

39 D. The repressor displaces cAM P receptor p rotein from the DNA


40
E. The repressor is detached from RNA polymerase
41
42
43
44
45
46
47
48
49
50
. 51 •
a
Lock
s
Suspend
8
End Bl ock
Item: 51 of 64 ~ 1 • M k -<:J 1>- Jil ~· !:';-~
QIO: 120 9 ..L ar Prev ious Next Lab~lues Notes Calculat o r

A A

31 Th e co rrect an swer i s A. 3 4 0/o ch ose this.


32 The lac operon allows bacteria to conserve resources. Through a regulatory mechanism, E. coli produces lactose-m etabolizing enzymes only when It must
use lactose as an energy source. Two steps need to happen to accomplish this. First, allolactose, an isomer of lactose, binds to the repressor, detaching It
33 from the DNA just upstream of the lac genes. This allows the cell's transcription machinery access to the lac genes. Second, when glucose levels are low,
cAMP levels increase. Cyclic-AMP binds to cAMP receptor protein, a regulatory molecule that binds to DNA and facilitates the binding of RNA polymerase to
34 the lac operon.
35 A1 olactose Escherochia col CAMP receptor protein Lactose Cyclic adenosine monophusphate Isomer Operon Repressor RNA polymerase Transcnption ,genetJCSJ
Protein Glucose Bacteria Molecule Enzyme RNA DNA Receptor (biochemistry)
36
B i s not correct. 27% chose this.
37
Although It Is true that the repressor is inactivated, cAMP levels increase (do not decrease) in low-glucose environments.
38 Cyclic adenosme monophosphate Repressor

39 C i s not correct. 1 5% chose this.


40 cAMP Is Increased in low-glucose states, and the repressor is inactivated In the presence of lactose.
Lactose Cyclic adenosine monophosphate Repressor
41
0 is not co rrect. 90/o chose t his.
42 The repressor and cAMP receptor protein bind to the DNA at two distinct, separate sites. The two do not interact with each other.
43 CAMP receptor protein Cyclic adenosine monophosphate Protein Repressor DNA Receptor (biochemistry) Protein -protein interaction

44 E is not co rrect. 150/o chose this.


The repressor prevents gene transcription of the lac genes by attaching to the DNA strand, not by interacting directly with RNA polymerase.
45
Gene ltanscrlption (genetics) Repressor RNA polymerase DNA RNA Polymerase
46
47
Bottom Lin e:
48
Lactose stimulates the lac operon. Allolactose acts as the inducer by binding to and inactivating the repressor. When no glucose is present, cAMP levels
49 rise. cAMP subsequently binds to cAMP receptor protein. The cAMP/cAMP receptor protein complex binds to the promoter of the lac operon and
facilitates transcription initiation.
50 Allolactose Operon CAMP receptor protein Lactose Repressor Cyclic adenosine monophosphate Transcription (genetics) Glucose Protein Inducer
Receptor (biochemistry)
51 •
a
Lock
s
Suspend
8
End Block
Item: 51 of 64 ~ 1 • M k -<:J 1>- Jil ~· !:';-~
QIO: 120 9 ..L ar Pre v ious Next Lab~lues Notes Calcula t o r
A A

31 FA17 p 36.2
32
Lac operon Classic example of a genetic response to an environmental change. Clucose is the preferred
33 metabolic substrate in E coli, but ''hen glucose is absent and lactose is available, the lac operon is
34 activated to switch to lactose metabolism. Vlechanism of shift:
35 • Low glucose - t adenylatc crclase acti,·itr - t generation of cA \IP from TP - acti,·ation of
catabolite activator protein (C P) - t transcription.
36
l ligh lactose - unbinds repressor protein from repressor/operator site - t transcription.
37
CAP
38
39
CAP i
~
~ cAMP\
~<1er ~te
• .e
L...C\._

....-o- Glucose
~
DNA

Messenger RNA
5'
WdSite P O lxl lxY lxA
- 3'

40 Binds CAP s1te, ATP


1nduces transcription
41
r-+~----- Lac operon - -- - - - - , STATE
42
43
Low glucose
l.Ktose '""table Lac genes strongly expressed
-
L Lacl Repressor pre>~
44
45
l High glucose
Lactose unavailable Li1c genes not expressed -
46 Low glucose
Lactose unavail.lble Lac genes not expressed
47 CAP
48
49 Allolactose
(lllducer)
Inactivated
repressor
High glucose
l.Ktose 1va~ble
site P 0
Very low (basaU expression -
50
51 •
a
Lock
s
Suspend
8
End Bl ock
Item: 51 of 64 ~ 1 • M k -<:J 1>- Jil ~· !:';-~
QIO: 120 9 ..L ar Pre v ious Next Lab~lues Notes Calcula t o r

31 Very low (basaU expre~SIOn


Allolactose Inactivated
{mducer) repressor
32
33
34 FA17 p 38.2

35
Regulation of gene expression

36
Promoter Site where R 'A polymerase II and multiple Promoter mutation commonly results in
other transcription factors bind to [) dramatic l in b ·el of gene transcription.
37
upstream from gene locus (Al~rich upstream
38 sequence with T T and C T bo,es).
39
Enhancer Stretch of D 'A that alters gene expression b) Enhancers and silencers mav be located close to,
'
40 binding transcription factors (eg, activator far from, or e,·en within (in an intron) the gene
41 proteins). whose expression it regulates.
42 Silencer Site where negative regulators (repressors) bind.
43
44 FA1 7 p 38.1
45 Functional Transcription start
(mRNA synthesized 5' - 3')
46 organization of a
47 eukaryotic gene CAAT box TATA box 1 ATG rtart codon
Polyadenylation
signal
48
DNA coding strand 5' CAAT TATAAT Exon 1 GT AG Exon2 GT AG Exon 3 AATAAA 3'
49
Promoter 5' UTR lntron 1 lntron 2 3' UTR
50
51 •
a
Lock
s
Suspend
8
End Bl ock
Item: 52 of 64 ~ 1 • M k -<:J 1>- Jil ~· !:';-~
QIO: 120 6 ..L ar Pre v ious Next Lab~lues Notes Calcula t o r
A A

32 While working in a microbiology laboratory, a researcher comes across an unlabeled cryotube in the freezer. She deduces that it contains a strain of
Escherichia coli and decides to test whether this strain has an intact lactose operon. After growing the cells in media containing both glucose and
33
lactose, she observes that a considerable amount of galactosidase, encoded by the lac operon, is expressed compared with controls. Galactosidase
34 production is also observed in the presence of lactose only. No protein products are produced when the E. coli is grown only with glucose.

35
Based on this observation, where is the mut ation most likely located?
36
37 :
A. Catabolite activator protein
38
B. Inducer-binding site
39
40 c. Promoter

41 o. Repressor
42
E. RNA polymerase
43
44
45
46
47
48
49
so
51
. 52 •
a
Lock
s
Suspend
8
End Bl ock
Item: 52 of 64 ~ 1 • M k -<:J 1>- Jil ~· !:';-~
QIO: 120 6 ..L ar Prev ious Next Lab~lues Notes Calculat o r

A A

32 Th e co rrect a nsw er i s A. 320/o ch ose this.


33 The lactose (lac) operon is an example of an inducible operon (as shown In the image below).
34 The lac operon is a polycistronic gene segment that encodes genes important for lactose metabolism including those for galactosidase, which cleaves
lactose Into glucose and galactose. Expression of the lac operon requires cAMP and lactose. As a surrogate marker for low glucose (ie, low ATP), cAMP
35 binds to the cAMP receptor protein (CRP). The CRP-cAMP complex then forms the catabolite activator protein (CAP), which facilitates binding of RNA
polymerase to the promoter. Lactose induces lac operon expression by binding to the repressor and removing it from the operator. Together, lactose and
36
cAMP are necessary for lac operon expression. In the absence of lactose, the repressor protein remains bound to the operator and the lac operon Is not
37 expressed. As the preferred energy source for the E. coli bacterium, glucose Is associated with low cAMP levels and therefore minimal expression of the
lac operon. Alternatively, when lactose is present and glucose is low, the lac operon ts expressed and galactosidase is produced.
38
In this vignette, no galactosidase is produced in the presence of glucose only, since lactose is not available to bind the repressor. In the presence of
39 lactose only, galactosidase is appropriately produced. However, in the presence of both lactose and glucose, galactosidase is still produced at a high level,
suggesting that glucose fails to suppress expression of the lac operon. Therefore, the mutation is most likely located in the CAP. Since galactosidase Is
40 produced at all, the repressor, the inducer-b inding site (of the repressor), RNA polymerase, and the promoter must all be functioning normally.
41 Catabolite activator protein Operon Messenger RNA Escherichia coli CAMP receptor protein Lactose Gene Cistron Lac operon Galactose RNA polymerase Glucose
42 RNA Cyclic adenosine monophosphate Protein Repressor Metabolism Mutation Bacteria Polymerase Receptor (biochemistry) Promoter (genetics) Catabolism

43 Operator
Promoter!
cAMP
44 activator /acZ (encodes JoeY (encodes lacA (encodes
protein t- IJ·galactosldase) p.galactoslde p.galactoside
45 lac/ . (CAP) permease) etylase)
tranS<Jc.,
s· --~"'~--'-t".__-------:·~---"',....--I Jllll-_;l-3' DNA
46 ,. binding ' ' '
site
47 AUG AUG AUG
Messenger RNA
48 s s s
49
High glucose
so )I - - - • • • Lactose available
Very low (basal) level of gene expression
51
52 • -CAP RNA polymerase
.-
a
Lock
s
Suspend
8
End Bl ock
Item: 52 of 64 ~ 1 • M k -<:J 1>- Jil ~· !:';-~
QIO: 120 6 ..L ar Pre v ious Next Lab~lues Notes Calcula t o r
A A

32 RNA Cyclic adenosine monophosphate Protein Repressor Metabolism Mutation Bacteria Polymerase Receptor (biochemistry) Promoter (genetics) Catabolism

33 Promote]
Operator

34 cAMP
activat or lacZ (encodes lacY (encodes lacA (encodes
protein 1:- !}-galactosidase) l}galaC1oside ~alactoside
35 lac/. (CAP) permease) transacetylase)
s· --'fli':-:"'~--'-t".__-------i"~---tJo-"--"'I Jlll---11-3 · DNA
36 ' binding ' ' '
site
37 AUG AUG AUG
Messer19er RNA
38 s s s
39
40 High glucose
Lactose available
Very low (basal) level of gene expression
41
CAP RNA polymerase
42
Low glucose
43 Lactose available
lac genes strongly expressed
44
Repressor protein
45
46 • - _o lac genes not expressed
High glucose
Lactose unavailable

47
48 Low glucose
Lactose unavailable
lac genes not expressed
49
so B i s not correct. 17% chose this.
Lactose Is the inducer of the lac operon. If t he inducer-binding site were mutated, the lac operon would not be expressed in t he presence of lactose
51 because the repressor protein would remain bound to t he operator region.
Operon Lactose Protein Repressor Inducer Enzyme inducer
52 •
a
Lock
s
Suspend
8
End Bl ock
Item:52of64 ~. , . M k <:] t> al ~· ~
QIO: 1206 .l. ar Previous Next lab 'Vfl1 ues Notes Calculator

lac genes not expressed



32

33 B is no t co rrect. 1 7% cho se this.


La ctose is the inducer of the lac operon . If the inducer -binding site were mutat ed, the lac operon would not be expressed in the presence of lactose
34 because the repressor protein would remain bound to the operator region.
Operon lactose Protein Repressor Inducer Enzyme inducer
35

36 c is no t co rrect. 20% cho se this.


A mutation in the promoter of the lactose operon is not consistent with the observation in the vignette. If the promoter were mutat ed, lac operon
37 transcription would va ry based on the mutation : a significant mutation could prevent RNA polymerase from binding altogether; while a more subtle
mutation could potentially decrea se or increase the affinity with which RNA polymerase binds to the promoter.
38
Operon lactose Mutation RNA polymerase RNA Transcription (genetics) Polymerase Promoter (genetics)
39
D is no t co rrect. 25% cho se this.
40 Lactose normally binds to the repressor to prevent binding to the lac operon . The repressor is functioning normally because protein production is inhibited
in the glucose -only scenario and permitted in the lactose -only scenario . If the repressor were mutat ed and nonfunctional, we would observe protein
41
production in any condition of low ATP availability, regardless of whether lactose were present.
42 Operon lactose Protein Repressor Adenosine triphosphate Protein production

43 E is no t co rrect. 6 % cho se this.


RNA polymerase is reponsible for the process of transcription, generating RNA by using DNA as a t emplat e. If RNA polymerase were mutat ed, all gene
44
expression would be affected.
45 Gene RNA polymerase RNA Transcription (genetics) Gene expression DNA Polymerase

46
47 Botto m Line:
48 The lac operon is an inducible set of genes that allows bacteria to use lactose as an energy source when glucose is not available. When glucose is
present, the lac operon should not be active because cA MP levels are low. When cA MP is low, it does not bind to CA P, and transcription of genes involved
49 in lactose met abolism is not activat ed.
50 Operon lactose Glucose Transcription (genetics) Cyclic adenosine monophosphate Metabolism Bacteria

51

52 •

6
lock
s
Suspend
0
End Block
Item: 52 of 64 ~ 1 • M k -<:J 1>- Jil ~· !:';-~
QIO: 120 6 ..L ar Pre v ious Next Lab~lues Notes Calcula t o r
A A

32 FA17 p 36.2

33 Lac operon C lassic example of a genetic response to an environmental change. Clucose is the preferred
34 metabolic substrate in E coli, but ''hen glucose is absent and lactose is available, the lac operon is
35 activated to switch to lactose metabolism. \llechanism of shift:
• Low glucose - t adenylatc crclase acti,·ity - t generation of cA VIP from TP - acti,·ation of
36
catabolite activator protein (C P) - t tra nscription.
37 lligh lactose - unbinds repressor protein from repressor/operator site - t transcription.
38 CAP
39
40
CAP i
~
~cAMP\
~<1er ~te
• .e
L...C\._

....-o- Gluc05e
~
DNA 5'
I.Jd s.te P o lxl

AUG'
lxY
--3'

41 Binds CAP stte ATP


tnduces transcripbon
42 ,---;.~----- Lac operon - -- - - - - , STAT£
Low glucose
43 l.Kiose available
44 L Lacl
45
1 High glucose
Lactose unavailable
46
47 Low glucose
Lactose unavail.lble
48
Hogh glucose
49 l.Ktose ,.all.lble
Allolactose Inactivated
so {lllducerl repressor

51
52 • C:A17..,. '1D ')

a
Lock
s
Suspend
8
End Bl ock
Item: 52 of 64 ~ 1 • Ma rk -<:J 1>- Jil ~· !:';-~
QIO: 120 6 ..L Pre v ious Next Lab ~lues Notes Calcula t o r


32 FA17 p 126.1
33 Bacterial ge net ics
34 Transformation Competent bacteria are able to bind and import Degraded

--."- a
uncombined \ _; (Recipient DNA
35 short pieces of environmental naked bacterial DNA

36
chromosomal D 'A (from bacterial cell ~"'-
""
0 Oonot'DNA

lysis). T he transfer and expression of ne" ly


37 Naked DNA Recipient cell TransfOfrned cell
transferred genes is called transformation.
38 A feature of many bacteria, especiallr S
39 pneumo11iae, H i11{luen::ae type B, and
40 :-..'eisseria (SHi:'\ ). Any Dr\ A can be used.
41
dding deoxyribonuclease to environment
will degrade naked 0 A in medium - no
42
transformation seen.
43
Conjugation
44
F+ x F· ~plasmid contains genes required for sex pilus Single strand
45 Sex pilus transferred
and con jugation. Bacteria withont this plasmid
46
47
48
are termed F-. Sex pilus on F"' bacterium
contacts F- bacterium. A single strand
of plasm id DNA is transferred across the
conjugal bridge ("mating bridge"). 'o transfer
Plasmid ol ~
f•cett F· cett
- ol~
~
f' cell f· cell
- f' cell f' cell
D
49
of chromosomal DNA.
so
Hfr x F F"' plasmid can become incorporated into
51
52
bacterial chromosomal D lA, termed high- ,
• frpnnpnr~· rPrnn,hin"tinn f H fr) rPII Tr.,.ncfpr

a
Lock
s
Suspend
8
End Bl ock
Item: 52 of 64
QIO: 120 6
~
..L
1 • Ma r k -<:J
Pre v ious
I>
Next Lab
fJ
lues
£!1}>'

Notes
!!":-~
Calcula t o r
A A

32 Transduction
33 Generalized A "packaging" event. Lytic phage infects Oeavage of Bacterial DNA package
34 bacterium, leading to clea\'age of bacl"erial
lytic
phage
I Bacteria bacterial DNA in phage capsid

35 o, A. Parts of bacterial chromosomal Dt A •


may become packaged in phage capsid. Phage
36
infects another bacterium, transferring these

-
37
genes.
38
39 0
40 Release of new phage Infects other Phage's genes
from lysed cell bacteria transferred IOl
41
Specialized An '·excision" e,·ent. Lrsogenic phage infects . Viral DNA Viral DNA .
42 lysogen1c ~ IOCOfpotates 1n Phage partiCles
bacterium; ,·iral D 'A incorporates into phage I
Bacteria bacterial DNA carry bactenal DNA

-
43
44
45
bacterial chromosome. When phage Dt
is excised, flanking bacterial genes may be
excised with it. D A is packaged into phage
c
46 capsid and can infect another bacterium.
47
Genes for the following 5 bacterial toxins are
encoded in a lysogenic phage (ABCD'S}: C roup
48
A strep erythrogenic toxin, Botulinum toxin,
49 Release of new phage Infects other Genes different from
Cholera toxin, Diphtheria toxin, Shiga toxin. from lysed cell bacteria donor and rec1ptent ~
so
Transposition Segment of Dl"\A (cg, transposon) that can
51 "jump" (excision and reintegration) from
52 • one location to another. can transfer eenes
a
Lock Suspend
s 8
End Bl ock
Item: 53 of 64 ~ 1 • M k -<:J 1>- Jil ~· !:';-~
QIO: 1683 ..L ar Pre v ious Next Lab~lues Notes Calcula t o r

33
A

A mother brings her 1-week-old infant to the physician because she has been having difficulty breast-feeding . On examination, the infant's head
circumference is found to be less than expected, given its height and weight, and cardiac auscultation reveals a ventricular septal defect. In addition,
IA•A] A

34
the mother reports that the infant's cry is strange.
35
36 Which of the following would most likely be found on cytogenetic analysis of the Infant?
37 :
38 A. 47,XYY Karyotype

39 B. Deletion of the short arm of chromosome s


40
c. Hybrid chromosome made up of a translocation between chromosomes 9 and 22
41
D. Robertsonian translocation between the long arms of chromosomes 14 and 21
42
43 E. Trisomy of chromosome 18

44
45
46
47
48
49
so
51
52
. 53

a
Lock
s
Suspend
8
End Bl ock
Item: 53 of 64 ~ 1 • M k -<:J 1>- Jil ~· !:';-~
QIO: 1683 ..L ar Prev ious Next Lab~lues Notes Calculat o r

A A

33 Th e co rrect an sw er i s B. 7 40/o ch ose this.


34 The Infant suffers from cri-du-chat syndrome, which is caused by a chromosome 5 deletion. In addition to microcephaly, hypotonicity, feeding difficulties,
congenital heart defects, and the characteristic "cry of the cat," affected Infants may also present with a moonlike face, wide-set eyes, epicanthal folds,
35 and Intellectual disability.
Microcephaly Cr du chat Intellectual disability Chromosome 5 (human) Epicanthic fold Chromosome Congenital heart defect Congenital disorder
36
A i s not correct. 2% chose t his.
37
A double-Y karyotype is not assodated with any gross abnormalities in Infants. In adult life, 47,XYY patients can suffer from severe acne. An association
38 with antlsodal behavior has been reported in these patients.
.aryotype syndrome Acne vulgaris
39
C i s not correct. 4 % chose this.
40
A characteristic chromosome translocation involVing chromosome 9 and chromosome 22 is seen in chronic myeloid leukemia and is called a Philadelphia
41 chromosome.
Chromosomal translocation Philadelphia chromosome Chromosome 22 (human) Leukemia Chronic myelogenous leukemia Chromosome Chromosome 9 (human)
42
Myeloid
43
0 is not co rrect. 8 0/o chose t his.
44 A Robertsonian translocation between the long arms of chromosomes 14 and 21 Is a cause of some forms of Down syndrome.
45 Robertsonian translocation Down syndrome Chromosomal translocation Chromosome Protein targeting

46 E is not co rrect. 120/o chose this.


Trisomy 18 Is Edwards syndrome, which is associat ed with clenched fists, rocker-bottom feet, intellectual disability, and micrognathia . These Infants usually
47 die within 1 year of birth.
48 Rocker bottom foot Micrognathism Edwards syndr ome Intellectual disability Trisomy

49
so Bottom Line:

51 Crl-du-chat syndrome, notable for the mew-like cry of affected infants, Is caused by a terminal deletion of 5p. Affected individuals have a partial
monosomy (one copy) for the genes in this region, since they have one normal copy of chromosome 5 in addition to the chromosome with the terminal
52 deletion.
Cr' du chat Chromosome 5 (human) Chromosome Monosomy Aneuploidy Delet1on (genetiCS)
53 •
a
Lock
s
Suspend
8
End Block
Item: 53 of 64 ~ 1 • M k -<:J 1>- Jil ~· !:';-~
QIO: 1683 ..L ar Pre v ious Next Lab~lues Notes Calcula t o r
A A

33 FA17 p60.3

34 Cri-du-chat syndrome Congenital microdeletion of short arm of Cri du chat = cry of the cat.
35 chromosome 5 (46,XX or XY, 5p-).
Findings: microcephaly, moderate to severe
36
intellectual disability, high-pitched crying/
37
meowing, epieanthal folds, cardiac
38 abnormalities (VSD).
39
40 FA17 p 59.1
41 Autosomal trisomies
42 Down syndrome Findings: intellectual disabilit}, Aat facies, Incidence 1:700.
43 (trisomy 21) prominent epicanthal folds, single palmar Drinking age (21).
crease, gap between 1st 2 toes, duodenal ~ lost common ,·iable chromosomal disorder and
44
atresia, Hirschsprung disease, congenital heart most common cause of genetic intellectual
45 disease (eg, atrioventricu lar septa I defect), dis abiIity.
46 Brush field spots. Associated with early-onset First-trimester 11ltrasound commonly shows
47 Alzheimer disease (chromosome 21 codes for t n11chal translucency and hypoplastic nasal
48 amyloid precmsor protein) and t risk of ALL bone; l serum PAPP-A, t free ~-hCC.
and AML. Second-trimester quad screen shows
49
95% of cases due to meiotic nondisjunction l a -fetoprotein, t ~-hCC, l estriol,
so (t with advanced maternal age; from 1:1500 in t inhibin A.
51 women< 20 to 1:25 in \\Omen> 45 )Cars old).
52 4% of cases due to unbalanced Robertson ian
53 translocation, most trpically between

a
Lock
s
Suspend
8
End Bl ock
Item: 53 of 64 ~ 1 • M k -<:J 1>- Jil ~· !:';-~
QIO: 1683 ..L ar Pre v ious Next Lab~lues Notes Calcula t o r
A A

33 FA17 p603.1

34 Sex chromosome Aneuploidy most common ly due to meiotic nondisjunction.


35 disorders
36 Klinefelter syndrome Testicular atrophy, eunuchoid bod) shape, Dysgenesis of seminiferous tubules
37
[male] (47,XXY) tall, long extremities, g) necomasti<l, female - l inhibin B - t FSII.
hair distribution . ~ lay present with Abnormal Leydig cell function - l testosterone
38
de,·elopmental delay. Presence of inacti,·ated - t Lll - t estrogen.
39
X chromosome (Barr body). Common cause of
40 hrpogonadism seen in infertilitr \\Ork-up.
41
42
43
44
45
46
47
Tu rner syndrome Short stature (if untreated; preventable wit h lenopause before men<uche.
48 [female] (45,XO) l estrogen leads to t Lll, FSII.
growth hormone therapy), ov:1rian dysgenesis
49
[] (streak ovary), shield chest : , bicuspid aortic Sometimes due to mitotic error ..... mosaicism (eg,
so valve, coarctation (femoral < brachial pulse), 45,X0/46,X:X).
51 lymphatic defect s (result in " ebbed neck or Pregnancy is possible in some cases (IVF,
cystic hygroma; lymphedema in feet. hands), exogenous estradiol-17~ and progesterone).
52
horseshoe kidney : .
53 • A l l"t.tof. ,...,.,,...,..,..,n,..., ,..,..,, coo nf 1° ""' " "'onnrrJ, ,_,. ' Tn l:t.,,.r

a
Lock
s
Suspend
8
End Bl ock
Item: 54 of 64 ~ 1 • M k -<:J 1>- Jil ~· !:';-~
QIO: 1387 ..L ar Pre v ious Next Lab~lues Notes Calcula t o r
A A

34 A 10-year-old boy is brought to the pediatrician after a school-sponsored hearing test reveals diminished hea ring in both ea rs. The boy complains of
ringing In both ears. The physician confirms the patient's hearing loss and orders an MRI, shown in the image.
35
36
37
38
39
40
41
42
43
44
45
46
47
48
49
so
51
52
53
• 54 •
a
Lock
s
Suspend
8
End Bl ock
34
35
36
37
38
39
40
41
42
43
44
45
46 Given the current findings, this patient is at risk for which of the following?

47 :
48 A. Hypopigmented spots

49 B. Macroorchidism
so
c. Meningioma
51
o. Pigmented iris hamartomas
52
53 E. Retinal hemangioblastomas
• 54 •
a
Lock
s
Suspend
8
End Block
Item: 54 of 64 ~ 1 • M k -<:J 1>- Jil ~· !:';-~
QIO: 1387 ..L ar Prev ious Next Lab~lues Notes Calculat o r

A A

34
35 Th e co rrect an swer i s c . 390/o chose this.
36 Patients with schwannomas, also called acoustic neuromas, may present with complaints of diminished
hearing, ringing in the ears (tinnitus), and impaired ba lance. These symptoms are suggestive of a problem
37 In the Inner ear, which should prompt imaging to rule out intracranial problems. In this case, bilateral
acoustic schwannomas are visible on MRI, as shown by the red circles In Image. The finding of bilateral
38 schwannomas IS one of the manifestations of t he autosomal dominant disorder neuro fibromatosis type 2
39 (NF2), caused by mutations in the NF2 gene on chromosome 22. This Is a comm on clinical presentation for
a patient with NF2. In addition to schwannomas, meningiomas develop In about SO% of patients with NF2.
40 NF2 Is also assoc1ated with gliomas, neurofibromas, and schwannomas.
Neu ofibromatosis type II Gene Dominance (genetics) Chromosome 22 human. NeL:·ofibromatosis
41
Vestibular schwannoma Glioma Meningioma Neurofibroma Inner ear Tinnitus Schwannoma
42 Magnetic resonance imaging Chromosome Merlin (protein) Neuroma Mutation
43
44
45
46
47
48
A is not co rrect . 150/o chose this.
49
Hypoplgmented "ash leaf" spots are seen in patients with tuberous sclerosis, an autosomal dominant disorder characterized by intellectual disability,
so seizures, facial angiofibromas, and astrocytomas.
Tuberous sclerosis Dominance (genetics) Intellectual disability Epileptic seizure Autosome Hypopigmentation Astrocytoma
51
B i s n ot correct. 6% ch ose this.
52
Macroorchidism is seen in patients with fragile X syndrome. Other findings Include marked developmental delay and delay increasing as the patient ages.
53 The affected person also presents with a large, long face with a prominent forehead and jaw.
Fragi e syndrome Macroorchidism Specific developmental disorder
54 •
a
Lock
s
Suspend
8
End Bl ock
Item:54of64 ~. , . M k <:] t> al ~· ~
QIO: 1387 .l. ar Previous Next lab 'Vfl1ues Notes Calculator

34 D is no t co rrect. 22 % c ho se this.
35 Pigmented iris ham artomas, also known as Lisch nodules, are seen in neurofibromatosis type 1. Other findings include cafe-au-la it spots, neural tumors,
and m arked skeletal disorders.
36 Neurofibromatosis type I Neurofibromatosis Iris (anatomy) Lisch nodule Hamartoma Neoplasm

37 E is no t co rrect. 1 8% c ho se this.

38 Von Hippei- Lindau syndrome is an autosomal dominant disorder characterized by abnormal growth of blood vessels. The overgrowth of vessels leads to
the development of angiomas and hem angioblastom as in the retina, brain, and spinal cord, as well as other areas of the body.
39 Dominance (genetics) Retina Von Hippel-lindau syndrome Spinal cord Autosome Von Hippel-lindau disease Human brain Blood vessel

40
41 Bo tto m Line:
42 Bilateral vestibular schwannomas, also known as acoustic neuromas, are a hallmark of neurofibromatosis type 2 ( NF2). NF2 is also associated with other
nervous system tumors, including ependymomas, gliomas, neurofibromas, and m eningiomas.
43
Neurofibromatosis type II Neurofibromatosis Meningioma Vestibular schwannoma Glioma Neurofibroma Neuroma Merlin (protein) Nervous system Schwannoma
44 Vestibular system

45
46
47 I ill ;fi 1!1 I•J f o r yea r :[ 20 17 ..
FI RST AID FA CTS

48
49 FA11 p 56.1

50 Autosomal dominant Achondroplasia, autosomal dominant polycystic kidney disease, fam ilial adenomatous polyposis,
51 diseases familial hypercholesterolemia, hereditary hemorrhagic telangiectasia, hereditary spherocytosis,
52 Huntington disease, Li-Fraumeni syndrome, Marfan syndrome, multiple endocrine neoplasias,
neurofibromatosis type l (von Recklinghausen disease), neurofibromatosis type 2, tuberous
53
sclerosis, von H ippei-Lindau disease.
54

6
lock
s
Suspend
0
End Block
Item: 54 of 64 ~ 1 • M k -<:J 1>- Jil ~· !:';-~
QIO: 1387 ..L ar Pre v ious Next Lab~lues Notes Calcula t o r
A A

34
35 FA17 p56.1

36 Autosomal dominant Achondroplasia, autosomal dominant pol) cystic kidne) disease, fam ilial adenomatous polyposis,
37 diseases familial hypercholesterolemia, hereditary hemorrhagic telangiectasia, hereditary spheroc)tosis,
Huntington disease, Li-Fraumeni S) ndrome, \ larfan S) ndrome, multiple endocrine neoplasias,
38
neurofibromatosis t} pe I ('on Reeklinghausen disease), neurofibromatosis type 2, tuberous
39
sclerosis, ,·on Hippei-Lindau disease.
40
41 FA17 p 56.2
42
Autosomal recessive Albinism, autosomal recessive pol) cystic kidne) disease (ARPKD), cystic fibrosis, glycogen
43 diseases storage diseases, hemochromatosis, Kartagener syndrome, mucopolysaccharidoses (except
44 Hunter S)1ldrome), phenylketonuria, sickle cell anemia, sphingolipidoses (except Fabry disease),
45 thalassemias, Wilson disease.
46
FA17 p 464.2
47
48 Schwann cells Each Schwann cell myclinates only J PJ Saxon. Ia)' be injured in Guilla in-Barre syndrome.
Also promote axonal regeneration. Derived
49 Nucleus { Schwann cell
so 1 from neural crest.

51
52 Myelin sheath} Node of tnvier
~
53
54 •
a
Lock
s
Suspend
8
End Bl ock
Item: 55 of 64 ~ 1 • M k -<:J 1>- Jil ~· !:';-~
QIO: 1191 ..L ar Pre v ious Next Lab~lues Notes Calcula t o r
A A

35 A nucleosome is composed of DNA wrapped around a core of eight histone molecules. The positively charged histones bind tightly to the negatively
charged phosphate group DNA.
36
37
Which of the following amino acids contributes to t he net positive charge of a histone?
38
:
39 A. Arginine
40
B. Aspartate
41
42 c. Cysteine

43 D. Glutamate
44
E. Seri ne
45
46
47
48
49
so
51
52
53
54
• 55 •
a
Lock
s
Suspend
8
End Bl ock
Item:55of64 ~. , . M k <:] t> al ~· ~
QIO: 1191 .l. ar Previous Next lab 'Vfllues Notes Calculator

35
The co rrect a nswer is A. 62% cho se this.
36 Arginine, lysine, and histidine are all positively charged, basic amino acids. Both arginine and lysine are found in histones and bind negatively charged
DNA.
37 Histidine lysine Arginine Amino acid Histone DNA
38
B is no t co rrect. 10% cho se this.
39 Aspartat e and glutamat e are negatively charged amino acids.
Glutamic acid Amino acid Aspartic acid
40
c is no t co rrect. 11% cho se this.
41
Cysteine is a polar amino acid with uncharged side chains.
42 Cysteine

43 D is no t co rrect. 10% cho se this.

44 Glutamat e and aspartat e are negatively charged amino acids.


Glutamic acid Amino acid Aspartic acid
45
E is no t co rrect. 7 % cho se this.
46 Serine is a polar amino acid with uncharged side chains.
47 Serine

48
49 Botto m Li ne:
50 Lysine and arginine are basic acid amino acids and carry a positive charge at a physiologic pH. Aspartat e and glutamat e are acidic amino acids and carry
a negative charge at a physiologic pH. Histones are basic proteins and contain a high amount of arginine and lysine residues.
51 lysine Arginine Glutamic acid Amino acid Aspartic acid Histone Acid Physiology Protein

52
53
54 I iii I;fi 1!1 I•J f o r yea r:[ 20 17 "
FI RST AI D FA CTS
55

6
lock
s
Suspend
0
End Block
Item: 55 of 64 ~ 1 • M k -<:J 1>- Jil ~· !:';-~
QIO: 1191 ..L ar Pre v ious Next Lab~lues Notes Calcula t o r
A A

35 FA17 p 77.3
36 Amino acids Only L-amino acids arc found in proteins.
37
Essential Glucogenic: methionine (\1ct}, histidine ( II i ~). I met h i~ ,·alentine, she is so sweet (glucogenic).
38 valine (Val). All essential amino acids need to be supplied in
39 Glucogenic/ketogenic: isoleucine (lie}, the diet.
40 phenylalanine (Phe}, threonine (Tin),
tr~ ptophan (Trp).
41
Ketogenic: leucine (Leu), lysine (L)S}.
42
Acidic Aspartic acid (Asp}and glutamic acid (Glu).
43
'egatively charged at body pi I.
44
Basic Histidine (Ilis), lysine (L) \), arginine (Arg). IIi\ I)~ (Iies) are basic.
45 Arg is most basic. Arg and His are required during periods of
46 His has no charge at body pH. growth. Arg and Lys are t in histones, which
47 bind negatively charged 0 1 A.
48
49 FA17 p 32.1

so Chromatin structure Dt exists in the condensed, chromatin form


51
in order to fi t into the nucleus. 1 egati,cJy
charged DNA loops twice around positively
52
charged histone octamer to form nucleosome
53 " bead~ on a ~tring." llistoncs arc rich in the
54 amino acids lysine and arginine. H 1 binds to
55 •
the nucleosome and to "linker DNA," thereby

a
Lock
s
Suspend
8
End Bl ock
Item: 55 of 64 ~ 1 • M k -<:J 1>- Jil ~· !:';-~
QIO: 1191 ..L ar Pre v ious Next Lab~lues Notes Calcula t o r
A
. .. ' -: . A

35 chromosome ~

36 Heterochromatin Condensed, appears darker on E\11 (labeled II l leteroC hromatin = Highly Condensed.
37 in rJ). Transcriptionally inacti\'e, sterically Barr bodies (inacti,·e X chromosomes) are
38 inaccessible. t methylation, l acetylation. heterochromatin.
39
40
41
42
43 Euchromatin Less condensed, appears lighter on E\ r (labeled Eu = true, utmly transcribed."
E in rJ). Transcriptionally acti,e, sterically Euchromatin is Expressed.
44
accessible.
45
DNA methylation Template strand cytosine and adenine arc
46
methylated in DNA replication, which allows
47 mismatch repair enzymes to distinguish
48 between old and new strands in probryotcs.
49 Dt A methylation at CpC islands represses CpC .Methylation Makes DNA Mute.
so transcription.

51 Histone methylation Usually reversibly represses D A transcription, llistone .Methylation Mostly i\takes D /\ .M ute.
but can acti,·ate it in some cases depending on
52
methylation location.
53
Histone acetylation Relaxes D 1A coiling, allowing for transcription. Histone \ eeh·lation
, makes D 'A \ eti\'e.
54
55 •
a
Lock Suspend
s 8
End Bl ock
Item: 56 of 64 ~ 1 • M k -<:J 1>- Jil ~· !:';-~
QIO: 4866 ..L ar Pre v ious Next Lab~lues Notes Calcula t o r
A A

36 An 11-year-old boy with intellectual disability presents for his annual check-up . The physician notes a number of abnormalities on physical
examination, including an unusually long face, large j aw, large everted ears, and macroorchidism. A family history reveals that the child's uncle has
37
the same condition, but the child's case is more severe than his uncle's.
38
39 This Is an example of which specific genetic phenomenon?
40 :
41 A. Antrcrpation

42 B. Heteroplasmy
43
c. I mprinting
44
D. Incomplete penetrance
45
46 E. Locus heterogeneity

47 F. Mosaicism
48
G. Uniparental disomy
49
50
51
52
53
54
55
. 56 •
a
Lock
s
Suspend
8
End Bl ock
Item: 56 of 64 ~ 1 • M k -<:J 1>- Jil ~· !:';-~
QIO: 4866 ..L ar Prev ious Next Lab~lues Notes Calculat o r

A A

36
Th e correct an swer i s A. 640/o ch ose this.
37 This patient has fragile X syndrome, the second most common genetic cause of intellectual disability.
38 Physical abnormalities associated with it include macroorchidism, a long face with a large jaw, and large
everted ears, as depicted to varying degrees in these images. Fragile X syndrome Is caused by an X-linked defect affecting the methylation and
39 expression of the FMRl gene. The mutation is a triplet repeat (CGG). like other triplet repeat disorders, fragile X may show genetic anticipation, the
phenomenon in which the severity of a disease worsens in succeeding generations, in this case due to the expanding repeat. Other disorders arising from
40 a similar mechanism include Huntington disease and myotonic dystrophy.
Frag le ~ •d ome Huntington's disease Myotonic dystrophy FMRl Gene Mac oo chidism Intellectual disability Antici pation (genetics) Methylation Mutation
41
Se n age
42
B i s not correct. 60/o chose this.
43
Heteroplasmy describes the presence of both normal and mutated mitochondrial DNA. This phenomenon is responsible for the variable expression of
44 mitochondrial inherited diseases.
Heteroplasmy Mitochondrial DNA Mitochondrion Expressivity (genetics) DNA Genetic disorder
45
C is not co rrect. 50/o chose this.
46
Imprinting occurs when the phenotype differs depending on whether the mutation is of paternal or maternal origin. This happens when only one allele Is
47 active at a single locus, resulting in disease if the active allele is deleted. The classic example is Prader-Willi syndrome and Angelman syndrome. Deletions
In Prader-Willi syndrome occur on the paternal chromosome 15, whereas deletions at the same site of chromosome 15 on the maternal chromosome
48 result In Angelman syndrome, a phenotypically distinct disorder.
Angelman syndrome Prader-Willi syndr om e Allele Phenotype Chromosome 15 (human) Chromosome Locus ( genetics) Genomic imprinting Mutation
49
Imprinting (psychology) Deletion ( genetics)
50
D is not co rrect . 100/o ch ose this.
51
Incomplete penetrance is the phenomenon by which not all individuals with a certain mutant genotype exhibit the mutant phenotype. In this case, both
52 the child and his uncle exhibit the fragile X phenotype; the child's case Is simply more severe.
Penetrance Phenotype Genotype Mutant
53
E i s n ot correct. 60/o ch ose this.
54
Locus heterogeneity describes the phenomenon by which mutations at different loci can result in the same phenotype. An example of this Is albinism,
55 which can be caused by a number of different mutations.
Albinism Phenotype Locus heterogeneity Locus (genetics) Mutation Heterogeneity
56 •
a
Lock
s
Suspend
8
End Bl ock
Item:56of64 ~. , . M k <:] t> al ~· ~
QIO: 4866 .l. ar Previous Next lab 'Vfl1 ues Notes Calculator

E is not correct. 6% chose this.


36
Locus heterogeneity describes the phenomenon by which mutations at different loci can result in the same phenotype. An example of this is albinism,
37 which can be caused by a number of different mutations.
Albinism Phenotype locus heterogeneity locus (genetics) Mutation Heterogeneity
38
F is not correct. 6% chose this.
39
Mosaicism occurs when cells in the body have different genetic make-up. This sometimes occurs, for example, in Turner syndrome. This does not occur in
40 fragile X syndrome.
Fragile X syndrome Turner syndrome Mosaic (genetics)
41
G is not correct. 3% chose this.
42
Uniparental disomy occurs when an individual receives two copies of a chromosome from one parent and no copies from the other parent.
43 Uniparental disomy Chromosome Aneuploidy
44
45
Bottom Line:
46 Anticipation is defined as worsening severity of the disease or earlier age at onset of the disease in succeeding generations.
47 Anticipation (song) Anticipation (early Simon album)

48
49
50
Iiii I;fi IJ I•J for year:[ 2017 ..
FI RST AID FAC T S

51
52 FA17 p 58.1

53 Fragile X syndrome X-linked dominant inheritance. Trinucleotide Trinucleotide repeat disorder {CCC)11•
54 repeat in FMR l gene - hypermethylation C hin (protruding), G iant G onads
55
- ! expression. tfost common cause of
inherited intellectual disability and autism
56 __ ..) .., __ ..) ---·· - - - - - - ---- -- .... c ____ ,.; __ 11. .

6
lock
s
Suspend
0
End Block
Item:56of64 ~. , . M k <:] t> al ~· ~
QIO: 4866 .l. ar Previous Next lab 'Vfl1 ues Notes Calculator

• •
36 FA17 p 52.2
37 Genetic terms

38 TERM DEFINITION EXAMPLE

39
Codominance Both alleles contribute to the phenotype of the Blood groups A, B, AB; a 1-antitrypsin
heterozygote. deficiency.
40
Variable expressivity Patients with the same genotype have varying 2 patients with neurofibromatosis type 1 ( 1Fl)
41
phenotypes. may have varying disease severity.
42
43
Incomplete ot all individuals with a mutant genotype BRCA.l gene mutations do not always result in
penetrance show the mutant phenotype. breast or ovarian cancer.
44
Pleiotropy One gene contributes to multiple phenotypic Untreated phenylketonuria (PKU) manifests with
45
effects. light skin, intellectual disability, and musty body
46
odor.
47
Anticipation Increased severity or earlier onset of disease in Trinucleotide repeat diseases (cg, Huntington
48 succeeding generations. disease).
49
Loss of heterozygosity If a patient inherits or develops a mutation in Retinoblastoma and the "two-hit hypothesis,"
50
a tumor suppressor gene, the complementary Lynch syndrome (Hl PCC), Li-Fraumcni
51 allele must be deleted/mutated before cancer syndrome.
52 develops. This is not true of oncogenes.
53 Dominant negative Exerts a dominant effect. A heterozygote Mutation of a transcription factor in its allosteric
54 mutation produces a nonfunctional altered protein that site. onfunctioning muta nt can still bind
55 also prevents the normal gene product from DNA, preventing wild-type transcription factor
functioning. from binding.
56 • •

6
lock
s
Suspend
0
End Block
Item: 57 of 64 ~ 1 • M k -<:J 1>- Jil ~· !:';-~
QIO: 5115 ..L ar Pre v ious Next Lab~lues Notes Calcula t o r

37
A

A 3-year-old child is brought to the pediatrician by his parents after he becomes jaundiced following a recent vira l illness. He also seems more tired
than usual. Inquiry into his family history reveals t hat both his mother and 9-year-old brother had similar symptom s at young ages and both have had

lA A] A

38
splenectomies. A peripheral blood sm ea r and osm otic fragility t est are diagnostic.
39
40 Deficiency or mutation of wha t protein is likely t he ca use of t his child's disorder?

41 :
42 A. Dystrophin

43 B. Flbrillln
44
c. Hemoglobin o chain
45
D. Hemoglobin 13 chain
46
47 E. Spectrin

48
49
so
51
52
53
54
55
56
. 57 •
a
Lock
s
Suspend
8
End Bl ock
Item: 57 of 64 ~ 1 • M k -<:J 1>- Jil ~· !:';-~
QIO: 5115 ..L ar Prev ious Next Lab~lues Notes Calculat o r

A A

37 Th e correct a nswer i s E. 770/o chose this.


38 This child has hereditary spherocytosis, a disorder in which the RBC proteins spectrin or ankyrin are lacking or nonfunctional. These proteins are
responsible for the biconcave shape of the erythrocyte, and without them, the cell becomes a round spherocyte. These spherocytes are more fragile and
39 patients are prone to episodes of hemolysis with stress, as seen with this patient. The spherical shape makes it more difficult for them to be filtered
through the spleen, contributing to the hemolysis and resulting in splenomegaly. Splenectomy ameliorates the condition, but would probably be delayed
40
until this child Is older because of an increased risk of infection. A peripheral smear would show spherocytes.
41 Spherocyto~.s Hereditary spherocytosis Spectrin Splenectomy Splenomegaly Red b ood ce Hemolysis Spleen Ankyrin Infection

42 A i s not correct. 3% chose this.

43 Dystrophin Is a protein that is mutated in Becker's and Duchenne's muscular dystrophies. It would not result in a hemolytic anemia, and there Is no role
for splenectomy in treatment. It would not be diagnosed on peripheral blood smear or by osmotic fragility test.
44 Splenectomy Hemolytic anemia Protein Blood film Anemia Erythrocyte frag'lity Hemolysos

45 B is not correct. 3% chose this.


Fibrlllln Is a protein that is defective in Marfan's syndrome. It is required for the formation of elastic fibers and in the disorder results in weaker connective
46
tissue. Patients with Marfan's syndrome are tall with long arms and legs (arachnodactyly), but are also prone to problems with the lens of the eye and
47 cardiovascular system. Aortic aneurysms along with mitral valve prolapse can be major complications for these patients.
Marfan syndrome Mitral valve prolapse Fibrillin Mitral valve Connective tissue Protein Circulatory system Elastic fiber Cardiovascular system Lens (anatomy)
48
Prolapse Aneurysm
49
C is not co rrect. 60/o chose t his.
so a-Thalassemia results from underproduction of the a chain in the hemoglobin molecule. a-Thalassemia is prevalent in Asian and African populations. It
would not result in the presentation seen with this patient, and it is not diagnosed by osmotic fragility test .
51
Hemoglobin Erythrocyte fragility
52
Dis n ot co rrect . 1 10/o ch ose this.
53 The hemoglobin 13 chain deficiency will result in 13-thalassemia, and one of the most common 13 chain mutations causes sickle cell anemia . 13-Thalassemla
54 patients have severe anemia and will often develop secondary hemochromatosis from needed transfusions. Peripheral smears will show target cells. Sickle
cell anemia is caused by a mutation in the 13 chain, and patients can have complications, including aplastic and pain crises, as well as autosplenectomy
55 with a subsequent increased risk of infection from encapsulated organisms.
Autosplenectomy Iron overload Hemoglobin Sickle- cell disease Anemia Polysaccharide encapsulated bacteria Mutation I nfection Sickle
56
57 •
a
Lock
s
Suspend
8
End Block
Item:57of64 ~ ., . M k <:] t> al ~· ~
QIO: 5115 .l. ar Previous Next lab 'Vfl1ues Notes Calculator

37
Bottom Line:
38
Hereditary spherocytosis is caused by a lack or malfunction of the RBC proteins spectrin or ankyrin.
39 Hereditary spherocytosis Spectrin Spherocytosis Ankyrin Red blood cell Protein

40
41

42 Iiii I;fi IJ I•J for year:[ 2017 ..


FI RST AI D FAC TS
43
44 FA17 p400.1
45 Intrinsic hemolytic anemia
46 DESCRIPTION FINDINGS

47
Hereditary Extravascular hemolysis due to defect in Splenomegaly, aplastic crisis (parvovirus Bl9
spherocytosis proteins interacting with RBC membrane infection).
48
skeleton and plasma membrane (eg, ankyrin, Labs: osmotic fragility test<±>. lormal to
49 band 3, protein 4.2, spectrin). Mostly l .\llCV with abundance of cells.
50 autosomal dominant inheritance. Treatment: splenectomy.
51 Results in small, round RBCs with less surface
52
area and no central pallor (t MCI !C)
- premature removal by spleen.
53
G6PD deficiency Most common enzymatic disorder of RBCs. Back pain, hemoglobinuria a few clays after
54
Causes extravascular and intravascular oxidant stress.
55
hemolysis. X-linked recessive. Labs: blood smear shows RBCs with Heinz
56 Defect in C6PD - l glutathione - t RBC bodies and bite cells.
57 susceptibility to oxidant stress. Hemolytic "Strc~s makes me eat bites of fa,a beans with

6
lock
s
Suspend
0
End Block
Item: 57 of 64 ~ 1 • M k -<:J 1>- Jil ~· !:';-~
QIO: 5115 ..L ar Pre v ious Next Lab~lues Notes Calcula t o r
A A

37 Paroxysmal nocturnal t complement-mediated intravascular RBC Associated with aplastic anemia.


38 hemoglobinuria lysis (impaired synthesis of GPI anchor e
Triad: Coombs hemolytic anemia,
39 for decay-accelerating factor that protects pancytopenia, and venous thrombosis.
40
RBC membrane from complement). Labs: CD55/59 8 RBCs on Aow crtometry.
Acquired mutation in a hematopoietic Treatment: eculizumab (terminal complement
41
stem cell. t incidence of acute leukemias. inhibitor).
42 Patients may report red or pink urine {from
43 hemoglobinuria).
44 Sickle cell anemia HbS point mutation causes a single amino Complications in sickle cell disease:
45 acid replacement in ~ chain {substitution plastic crisis {due to parvovirus B19).
46 of glutamic acid with valine). Causes J\utosplenectomy (llowell-Jolly bodies)
extravascular and intravascular hemolysis. ..... f risk of infection by encapsulated
47
Pathogenesis: low 0 2, high altitude, or acidosis organisms (eg, S pneumoniae).
48
precipitates sickling (deoxygenated ll bS Splenic infarct/sequestration crisis.
49 • polymerizes) ..... anemia, vase-occlusive disease. Salmonella osteomyelitis.
so Newborns are initially asymptomatic because of • Painful crises (vaso-occlusive): dactyl itis I}]
51
- 11!1 t HbF and l HbS. (painful swelling of hands/feet}, priapism,
Heterozygotes (sickle cell trait) also have acute chest syndrome, avascular necrosis,
52
resistance to malaria. stroke.
53
8% of African Americans carry an ll bS allele. Renal papillary necrosis (l Po2 in papilla)
54 Sickle cells are crescent-shaped RBCs rJ. and microhematuria (medullary infarcts).
55 "Crew cut" on skull x-ray due to marrow Diagnosis: hemoglobin electrophoresis.
56 expansion from f erythropoiesis {also seen in Treatment: hydroxyurea {f HbF'), hydration.
thalassemias).
57 •
a
Lock
s
Suspend
8
End Bl ock
Item: 58 of 64 ~ 1 • M k -<:J 1>- Jil ~· !:';-~
QIO: 1393 ..L ar Pre v ious Next Lab~lues Notes Calcula t o r
A A
38 A 28-year-old man presents to his physician complaining of worsening arthritis In his left knee . He has a history of sponta neous bleeding episodes
39 (which have led to his osteoarthritis) and easy bruising. Previous testing has shown a normal platelet count and bleeding t ime but a prolonged
activated partial thromboplastin time.
40
41 This patient Is most likely deficient in which of t he following factors?

42 :
43 A. Factor VII

44 B. Factor VIII
45
c. Factor X
46
47
o. Factor XI

48 E. von Willebrand factor

49
so
51
52
53
54
55
56
57
. 58

a
Lock
s
Suspend
8
End Bl ock
Item:58of64 ~. , . M k <:] t> al ~· ~
QIO: 1393 .l. ar Previous Next lab 'Vfl1 ues Notes Calculator

38 The co rrect a nswer is B. 70% cho se this.


This patient has hemophilia A, an X-linked disorder characterized (in moderat e to severe deficiency) by spontaneous bleeding into deep soft tissues (eg,
39
muscle), ea sy bruising, and hemarthrosis into weight -bearing joints (hip, knee, and ankle), due to a deficiency of factor VIII. Repetitive hemarthroses can
40 lea d to osteoarthritis of the affected joint, which increa ses the risk of subsequent bleeds into that joint. Laboratory t esting in patients with this condition
will show a prolonged activat ed partial thromboplastin time; prothrombin time is usually normal. Of note, deficiency of factor IX (Christmas disea se or
41 hemophilia B) is clinically indistinguishable from factor VIII deficiency, although factor VIII deficiency is about 5 times more common . Both will result in an
increa sed PTT, normal PT, and normal bleeding time; decrea sed levels of factor VIII or IX will be present and can be confirmed when the addition of the
42 missing factor to a sample of blood results in more rapid clot formation .
43 Hemarthrosis Prothrombin time Haemophilia Osteoarthritis Haemophilia A Haemophilia B Partial thromboplastin time Factor VIII Thrombin Factor IX
Bleeding time Sex linkage Muscle Bruise Thrombus
44
45 A is no t co rrect. 8% cho se this.
Fa ctor VII deficiency is a rare autosomal recessive disorder. Spontaneous or posttraumatic bleeding can occur. Hemarthroses are rare.
46 Factor VII Hemarthrosis Recessive Autosome Dominance (genetics)

47 c is no t co rrect. 4 % cho se this.


48 Fa ctor X deficiency is a rare autosomal recessive disorder. Spontaneous or posttraumatic bleeding can occur. Hemarthroses are rare.
Factor X Hemarthrosis Autosomal recessive Recessive Autosome Dominance (genetics)
49
D is no t co rrect. 8% cho se this.
50
Fa ctor XI deficiency (hemophilia C) is an autosomal recessive bleeding disorder. Unlike hemophilia A (factor VIII deficiency) or hemophilia B (factor IX
51 deficiency), hemarthroses are uncommon, there is less spontaneous bleeding, and the correlation between the amount of factor XI activity and the clinical
symptoms is not as well defined. Fa ctor XI deficiency is more common in Ashkenazi Jews, although it is still a relatively rare bleeding disorder.
52 Haemophilia C Haemophilia Factor XI Haemophilia B Haemophilia A Hemarthrosis Factor VIII Ashkenazi Jews Autosomal recessive Factor IX Recessive

53 Dominance (genetics) Von Willebrand disease Autosome Coagulopathy

54 E is no t co rrect. 10 % cho se this.


von Willebrand factor (vWF) aids in plat elet adhesion. A deficiency of vW F causes prolonged bleeding time, which manifests as increa sed bleeding after
55
trauma or surgery, nosebleeds, and hematomas. Laboratory t ests may show a slightly prolonged activat ed partial thromboplastin time (due to vW F's
56 action as a stabilizer of factor VIII) and a normal prothrombin time. A major variable distinguishing vW F deficiency from a deficiency in factor VIII is
bleeding time, which is prolonged with vW F deficiency and normal in factor VIII deficiency.
57 Von Willebrand factor Prothrombin time Partial thromboplastin time Factor VIII Platelet Thrombin Bleeding time Nosebleed Thromboplastin Hematoma

58 Major trauma

6
lock
s
Suspend
0
End Block
Item:58of64 ~. , . M k <:] t> al ~· ~
QIO: 1393 .l. ar Previous Next lab 'Vfl1 ues Notes Calculator

38
Bottom Line:
39
Hemophilia, a group of X-linked recessive hematologic disorders, exists as two forms : hemophilia A, or factor VIII deficiency, and hemophilia B, or factor
40 IX deficiency. It manifests clinically as spontaneous hemarthroses, excessive soft -tissue bruising, and spontaneous bleeding.
Haemophilia Haemophilia B Haemophilia A Factor VIII Hemarthrosis Factor IX X- linked recessive inheritance Recessive Dominance (genetics) Sex linkage
41
Hematology Bruise Soft tissue
42
43
44 lijj ;fi IJ l•l for year:l 2017 ..
FIRST AID FACTS
45
46
FA17 p404.2
47
Coagulation disorders PT- tests function of common and extrinsic pathway (factors I, II, V, VII, and X). Defect -+ t PT
48
I lR (international normalized ratio)-calculated from PT I =normal,> I =prolonged. Most
49 common test used to follow patients on warfarin.
50 PTT- tests function of common and intrinsic pathway (all factors except VII and XII I). Defect
51 - t PIT.
Coagulation disorders can be due to clotting factor deficiencies or acquired inhibitors. Diagnosed
52
with a mixing study, in which normal plasma is added to patient's plasma. Clotting factor
53
deficiencies should correct (the PT or PIT returns to within the appropriate normal range),
54 whereas factor inhibitors will not correct.
55
DISORDER PT PTT MECHANISM AND COMMENTS
56 Hemophilia A, B, or C t Intrinsic pathway coagulation defect.
57 • A: deficiency of factor VIII ..... t PTT; X-Iinked recessive.
58 • B: deficiency of factor IX ..... t PTT; X-Iinked recessive.

6
lock
s
Suspend
0
End Block
Item: 58 of 64 ~ 1 • M k -<:J 1>- Jil ~· !:';-~
QIO: 1393 ..L ar Pre v ious Next Lab~lues Notes Calcula t o r
A A
38
39 FA17 p 406.1

40 Mixed platelet and coagulation disorders


DISORDER PC BT PT PTT MECHANISM AND COMMENTS
41
von Willebrand t -It Intrinsic pathway coagulation defect: l v\VF
42
disease - t PTf (v\VF acts to carry/protect factor
43 VIII).
44 Defect in platelet plug formation: l v\VF
45 - defect in platelet-to-v\VF' adhesion.
46 utosomal dominant. 1\ lild but most common
inherited bleeding disorder. 'o platelet
47
aggregation with ristocetin cofactor assay.
48
Treatment: desmopressin, which releases
49 vWF stored in endothelium.
so Disseminated t t t Widespread activation of clotting - deficiency
51 intravascular in clotting fa ctors - bleeding state.
52 coagulation Causes: Sepsis (gram 8 ), Trauma, Obstetric
53 compi ications, acute Pancreatitis,
vial igna ney, Nephrotic syndrome,
54
Transfusion (STOP M aking New T hrombi).
55 Labs: schistocytes, t fibrin degradation
56 products (o-dimers), l fibrinogen, l factors
57 and Ill.
58 •
a
Lock
s
Suspend
8
End Bl ock
Item: 58 of 64 ~ 1 • M k -<:J 1>- Jil ~· !:';-~
QIO: 1393 ..L ar Pre v ious Next Lab~lues Notes Calcula t o r

38
A
vWF stored in endothelium. A

39 Disseminated t t t Widespread acti,·ation of clotting - defi ciency


40 intravascular in clotting factors - bleeding slate.
coagulation Causes: Sepsis (gram 8), Trauma, O bstetric
41
complications, acute Pancreatitis,
42
\lalignancy, :'\eph rotic syndrome,
43 Transfusion (STOP ~ laking i\ew T hrombi).
44 Labs: schistoc~tes, t fibrin degradation
45 products (o-dimers), l fibrinogen, l factors V
and VIII.
46
47
48 FA17 p 56.4
49 X-linked recessive Ornithine transcarbamylase defi cienC}', Fabry Oblivious Female \\'ill O ften G ive ller Boys
so disorders disease, \Viskott- ldrich syndrome, Ocular ller x-Linked Disorders
51 albinism, G6PD defi ciency, l lunter syndrome,
Bruton agammaglobul inem ia, l lemoph ilia
52
A and B, Lesch-Nyhan syndrome, Duchcnne
53
(and Becker) muscular dystrophy.
54 Lyonization - female carriers \'ariably affected
55 depending on the pattern of in act ivaiion of the
56 X chromosome carrying the mutant vs normal
gene.
57
58 •
a
Lock
s
Suspend
8
End Bl ock
Item: 59 of 64
QIO: 1273
~
..L
1 • Ma r k -<:J
Pre v ious
I>
Next Lab
fJ
lues
£!1}>'

Notes
!!":-~
Calcula t o r
A A

39 Griseofulvin is an antifungal agent that acts primarily during the M phase of the cell division cycle .
40
41 On which of the following protein structures does this drug act?

42 :
A. o/ 1}-Tubulin dimer
43
44 B. y-Tubulin

45 c. Actin
46
o. Kinesin
47
48 E. Kinetochore

49 F. Myosin

so
51
52
53
54
55
56
57
58
• 59 •
a
Lock
s
Suspend
8
End Bl ock
Item: 59 of 64 ~ 1 • M k -<:J 1>- Jil ~· !:';-~
QIO: 1273 ..L ar Prev ious Next Lab~lues Notes Calculat o r

A A

39
Th e co rrect an swer i s A. 510/o chose this.
40 During the M phase, or mitotic phase, of the cell division cycle, duplicated genetic material is equally distributed to two daughter cells. Microtubule spindle
41 fibers, which are composed of o/ 13-tubulin dimers, form the major structures along which kinetochore-attached kinesin motors carry the sister chromatids.
Griseofulvin binds to tubulin dimers and disrupts the spindle apparatus.
42 Griseofulvin Microtubule Kinesin Tubulin Spindle apparatus Mitosis M phase Chromatid Cel division Cell cycle Sister chromatids Genome Protein dimer

43 Dime1 "chem"sty·

44 B i s not correct. 230/o chose t his.


y-Tubulln Is present at very high concentrations in centrioles, and is required for nucl eation of microtubule spindle fibers. Centrioles are the core of the
45 centrosome, which develops in prophase to direct chromosome separation.
46 1"1crot· b1 1e Centrosome Prophase Spindle apparatus Centriole Chromosome N11cleation

47 C i s not correct. 80/o chose this.


Acti n filaments form a contractile ring in the cell during cytokinesis, wh ich facilitates creation of two cells. A drug that inhibits metaphase would be
48
unlikely to affect acti n filaments.
49 Cytokinesis Actin Metaphase Microfilament

so 0 is not co rrect. 70fo chose t his.


The klneslns are a family of motor proteins, similar to myosin, but attach to and are directed along microtubules (rather than actin filaments) . Klnesln
51 proteins are required for the movement of chromosomes during anaphase.
52 Anaphase Myosin Actin Kinesin Motor protein Microtubule Microfilament Chromosome Protein Molecular motor

53 E is not co rrect . 80/o ch ose this.


Klnetochores are complexes of proteins that make up the centromere of the chromosome . They contain the kinesin motor proteins that move sister
54 chromatids during anaphase.
55 Centromere Anaphase Kinesin Motor protein Chromosome Kinetochore Chromatid Sister chromatids Protein

56 F i s n ot co rrect. 30/o ch ose this.


Myosin filaments work with actin filaments to divide the cell during cytokinesis.
57
Myosin Cytokinesis Actin Microfilament
58
59 •
a
Lock
s
Suspend
8
End Block
Item:59of64 ~. , . M k <:] t> al ~· ~
QIO: 1273 .l. ar Previous Next lab 'Vfl1 ues Notes Calculator

39
Bottom Line:
40
Griseofulvin is an antifungal agent that binds to a/ j3 -tubulin dimers and disrupts the spindle apparatus, preventing successful anaphase during mitosis.
41 Griseofulvin Mitosis Anaphase Spindle apparatus Antifungal Protein dimer Fungicide

42
43
44 I ill ;fi 1!1 I•J for year:[ 2017 ..
FIRST AID FAC T S
45

46 FA17 p 44.3
47 Microtubule Cylindrical outer structure composed of a D rugs that act on microtubules (M icrotubules
48 helical array of polymerized heterodimers G et C onstructed Very Poorly):
Positive
49 end (+} of o.- and ~-tubul in . Each dimer has 2 GTP • \1ebendazole (antihelminthic)
50 Heterodimer A' bound. Incorporated into Aagella, cilia, mitotic • G riseofulvin (antifungal)
spindles. Grows slowly, collapses quickly. • C olchicine (antigout)
51
Also involved in slow axoplasmic transport in • YincristineNvinblastine (anticancer)
52 neurons. • Paclitaxcl (anticancer)
53 Molecular motor proteins-transport cellular
54 cargo toward opposite ends of microtubule
55 tracks.
Negative • Dynein- retrog rade to microtubule(+ - -). Negative end Near N ucleus
56
end (- ) • Kinesin-anterograde to microtubule(- - +). Positive end Points to Periphery
57
58
FA11 p 4 2.1
59

6
lock
s
Suspend
0
End Block
Item: 59 of 64 ~ 1 • M k -<:J 1>- Jil ~· !:';-~
QIO: 1273 ..L ar Pre v ious Next Lab~lues Notes Calcula t o r
A A

39 FA17 p42.1

40 Cell cycle phases Checkpoints control transitions between phases of cell C)cle. This process is regulated by C) dins,
41 cyclin-dependent kinases (CDKs), and tumor suppressors. ~,1 phase (shortest phase of cell C)cle)
42
includes mitosis (prophase, prometaphase, metaphase, anaphase, telophase) and cytokinesis
(cytoplasm splits in h\O). C 1 and C 0 are of ,·ariahle duration.
43
REGULATION OF CELL CYCLE
44
Cyclin-dependent Constituti,·e and inactive.
45
kinases
46
Cyclins Regulatory proteins that control cell C)clC
47
e\'ents; phase specific; acti\'ate CDKs.
48
Cyclin-CDKcomplexes Phosphorylate other proteins to coordinate
49 cell cycle progression; must be act iva ted and
so inactivated at appropriate times for cell cycle
51 to progress.
52 Tumor suppressors p53 induces p21, which inhibits CDKs
53 - hypophosphorylation (aelivotion) of Rb
54
- inhibition of Gr S progression. Mutations
in tumor suppressor genes can result in Rb. p53 modulate
55
unrestrained cell division (eg, Li-Fraumeni G1 restrictiOn point
56 syndrome).
57 CEll TYPES
58 Permanent Remain in G 0, regenerate from stem cells. 'eurons, skeletal and cardiac muscle, RBCs.
59 • Stable (auiescent) Enter G. from G,. when stimulated. 1-leoatoc\'tes. lvmohoc\'tes.
a
Lock
s
Suspend
8
End Bl ock
Item: 59 of 64 ~ 1 • M k -<:J 1>- Jil ~· !:';-~
QIO: 1273 ..L ar Pre v ious Next Lab~lues Notes Calcula t o r
A
.. . A

39 inactivated at appropriate times for cell cycle


40 to progress.
41 Tumor suppressors p53 induces p21, which inhibits CDKs
42 - hypophosphorylation (acti\'ation) of Rb
43
- inhibition of CrS progression. \lutations
in tumor suppressor genes can result in Rb. p53 modulate
44
unrestrained cell di\•ision (eg, l.i-Fraumeni G, reS1tiC!JOn pomt D
45 syndrome).
46 <Ell TYPES
47 Permanent Remain in C 0 , regenerate from stem cells. eurons, skeletal and cardiac muscle, RBCs.
48 Stable (quiescent) Enter C 1 from C 0 when stimulated. Hepatocytes, lymphocytes.
49 Labile Ne\·er go to C 0, divide rapidly with a short C 1• Bone marrow, gut epithelium, skin, hair follicles,
so Most affected by chemotherapy. germ cells.
51
52 FA1 7 p 196.2

53 Griseofulvin

54 MECHANISM Interferes with microtubule function; disrupts mitosis. Deposits in keratin-containing tissues (eg,
nails).
55
56 CLINICAL USE Oral treatment of superficial infect ions; inhibits growth of dermatophytes (tinea, ringworm).
57 ADVERSE EFFECTS Teratogenic, carcinogenic, confusion, headaches, disulfiram-like reaction, t cytochrome P-450 and
58
warfarin metabolism.

59 •
a
Lock
s
Suspend
8
End Bl ock
Item: 60 of 64 ~ 1 • M k -<:J 1>- Jil ~· !:';-~
QIO: 3940 ..L ar Pre v ious Next Lab~lues Notes Calcula t o r
A A

40 A 30-year-old man presents to the physician wit h a painless, flesh-colored nodule on his forearm . He reports t hat t he mass has always been
asymptomatic, but has gradually increased in size over the past two years. The patient wears hearing aids in both ears. He explains that his hearing
41
Impairment is due to benign, bilat eral brain t umors that were removed three years ago. He also has a history of cata racts. The patient ultimately
42 undergoes surgery to remove the mass on his arm and t he t issue is sent for biopsy. Histologic examination shows one cell t ype, wit h elongated nuclei In a
whorllng and palisading pattern.
43
44 Which of the following diseases has t he same inherita nce pat t ern as this patient's condition?
45
:
46 A. Angelman syndrome

47 B. Fragile X syndrome
48
c. Hereditary nonpolyposis colorectal cancer
49
so o. Homocystinuria

51 E. Spina bifida occulta


52
53
54
55
56
57
58
59
• 60 •
a
Lock
s
Suspend
8
End Bl ock
Item: 60 of 64 ~. I • M k <:] t> al ~· ~
QIO: 3940 .l. ar Previous Next lab 'Vfl1 ues Notes Calculator

40
41 The co rrect a nswer is c. 66 % cho se this.
The patient in the stem is suffering from neurofibromatosis type II (N F2 ). NF2 is caused by an autosomal dominant mutation on chromosome 22. Ninety
42 percent of people with NF2, including this patient, develop bilat eral vestibular schwannomas by their 30s. NF2 is associat ed several other CNS tumors,
including schwannomas of other cranial nerves, meningiomas, and ependymomas of the spine. NF2 is also associat ed with cutaneous schwannomas, which
43
appear as cells with elongat ed nuclei in a whorling pattern on histological examination . Hereditary nonpolyposis colorectal cancer ( HN PCC), also called
44 Lynch syndrome, is another autosomal dominant condition that predisposes to the development of cancer. The mutation is in DNA mismatch repair genes.
Individuals with this disea se have an approximat ely 70% risk of developing colorectal cancer over the course of their lives. Females with Lynch syndrome
45 are also at a signfiicantly increa sed risk of developing endometrial cancer.
Endometrial cancer Neurofibromatosis type II Hereditary nonpolyposis colorectal cancer Colorectal cancer Neurofibromatosis DNA mismatch repair
46
Dominance (genetics) Mutation Cranial nerves Chromosome 22 (human) Meningioma Central nervous system Chromosome Autosome Histology Endometrium
47
Cancer Schwannoma Cell nucleus DNA Neoplasm Merlin (protein)
48
A is no t co rrect. 6 % cho se this.
49 Angelman syndrome results from a mat ernally derived microdeletion on chromosome 15. It is characterized by intellectual disability, cheerful disposition
with frequent, inappropriat e laughter; gait at axia, microcephaly, seizures, and difficulty sleeping.
50
Angelman syndrome Microcephaly Ataxia Intellectual disability Chromosome 15 (human) Deletion (genetics) Epileptic seizure Chromosome Gait abnormality
51
B is no t co rrect. 1 0 % cho se this.
52 Fragile X syndrome is an X-linked disorder caused by CGG repea ts that result in the loss of function of the fragile X mental ret ardation ( FMRl ) gene.
People with grea t er than 200 CGG repea ts have Fragile X syndrome, while 50 -200 repea ts can cause milder symptoms, though the FMRl gene is still
53
active. Macrocephaly, joint hyperlaxity, unusually soft skin, learning and language delays, autism spectrum disorders, impulsivity, and anxiety are all
54 fea tures of the disea se. Females with the full mutation can be asymptomatic, though they frequently have mild symptoms.
Fragile X syndrome Gene FMRl Autism Macrocephaly Autism spectrum Sex linkage Mutation Intellectual disability Anxiety
55
D is no t co rrect. 1 5 % cho se this.
56
Homocystinuria is an autosomal recessive disorder in which methionine is not properly met abolized, lea ding to a buildup of homocysteine. Clinical
57 fea tures include Marfanoid body habitus (tall, thin build with high arched feet, long limbs, and pectus deformities) and downward lens dislocation . Unlike
Marfan syndrome, homocystinuria is associat ed with mental ret ardation .
58 Marfan syndrome Homocystinuria Ectopia lentis Autosomal recessive Dominance (genetics) Autosome Methionine Marfanoid Homocysteine Intellectual disability

59 Metabolism Morphology (biology) Recessive

60 E is no t co rrect. 3 % cho se this.

6
lock
s
Suspend
0
End Block
Item: 60 of 64 ~. I • M k <:] t> al ~· ~
QIO: 3940 .l. ar Previous Next lab 'Vfl1 ues Notes Calculator

40 E is not correct. 3% chose this.


Neural tube defects, such as spina bifida occulta, have combined hereditary and environmental causes. Genetic predisposition interplays with modifiable
41
risk factors such as maternal folic acid deficiency. In spina bifida occulta, vertebral bodies do not fuse normally, but the meninges and spinal cord do not
42 herniate through the defect, and the overlying skin is normal. As a result, spina bifida occulta is generally asymptomatic and often goes undetected.
Spina bifida Folic acid Meninges Neural tube Spinal cord Spina bifida occulta Neural tube defect Vertebra Genetic predisposition Brain herniation
43
44
Bottom Line:
45
NF2 is an autosomal dominant disease associated with a variety of nervous system tumors, including bilateral vestibular schwannomas, meningiomas,
46 and ependymomas.
Dominance (genetics) Meningioma Autosome Merlin (protein) Nervous system Vestibular system
47
48
49
lijj ;fi IJ l•l for year:l 2017 ..
50 FIRST AID FAC T S

51
52 FA17p56.1

53 Autosomal dominant Achondroplasia, autosomal dominant polycystic kidney disease, fam ilial adenomatous polyposis,
54 diseases familial hypercholesterolemia, hereditary hemorrhagic telangiectasia, hereditary spherocytosis,
Huntington disease, Li-Fraumeni syndrome, Marfan syndrome, multiple endocrine neoplasias,
55
neurofibromatosis type I (,·on Recklinghausen disease}, neurofibromatosis type 2, tuberous
56
sclerosis, von 1-1 ippei-Lindau disease.
57
58
FA11 p495.1
59 Neurocutaneous disorders
60 SturnP-WPhPr

6
lock
s
Suspend
0
End Block
Item: 60 of 64 ~ 1 • M k -<:J 1>- Jil ~· !:';-~
QIO: 3940 ..L ar Pre v ious Next Lab~lues Notes Calcula t o r
A A

40 FA17 p 495.1
41 Neurocutaneous disorders
42 Sturge-Weber Congenital, non inherited (sporadic), de' elopmenlal anomaly of neural crest deri,·atives due to
43 syndrome somatic mosaicism for an activating mutation in one copy of the C AQ gene. Affects small
44
(encephalotrigeminal {capillarr-sized) blood \'essels - port-\\ ine stain of the face rJ (ne\'US Aammeus, a non-neoplastic
angiomatosis) "birthmark" in CN /V 2 distribution); ipsilateral leptomeningeal angioma I] - seizures/
45
epilepsy; intellectual disability; and episcleral hemangioma - t lOP - early-{)nset glaucoma.
46 STU RCE-\Veber: Sporadic, pori-\\ ine Stain; Tram track calcifications {opposing gyri); Unilateral;
47 Retardation (intellectual disability); Glaucoma, C 'AQ gene: Epilepsy.
48 Tuberous sclerosis TSCiffSC2 mutation on chromosome 16. utosomal dominant, ,·ariablc expression.
49 IL\MARTO~ IAS: Hamartomas in C t Sand ~kin; Angiofibromas ~; ~litral regurgitation;

so Ash-leaf spots [!l; cardiac Rhabdomyoma; (Tuberous sclerosis); autosomal dO minant; l\lental
retardation (intellectual disabi lity); renal \ ngiomyol ipoma 0 ; Seizures, Shagreen patches.
51
t incidence of subependymal gi~mt cell astroc) lomas and ungual fibromas.
52
Neurofibromatosis l'v lutation in Fl tumor suppressor gene on chromosome 17 (17 letters in "von Recklinghausen"),
53
type I (von which normally codes for ncurofibromi n, <1 negative regulator of RAS. Autosomal dominant,
54 Reckli nghausen 100% penetrance. Cafe-au-lait spots D. cutaneous neurofibroma s ~. optic gliomas,
55 disease) pheochromocytomas, Lisch nodules (pigmented iris hamartomas CJ).
56 Neurofibromatosis Mutation in NF2 h1mor suppressor gene on chromosome 22. Autosomal dominant. F'indings:
57 t ype II bilateral acoustic schwannomas, juvenile cataracts, meningiomas, and ependymomas. lf'2
58 affects 2 ears, 2 eyes, and 2 parts of the brain.
59 von Hippel-lindau Deletion of VI IL gene on chromosome 3p (VII L = 3 letters). utosomal dominant. Characteri1ed
60 •
disease
1 1 • 1 . .. .... ..
·· - . ...
by development of numerous tumors, both benign and malignant. HARP: Hemangioblastomas
-
a
Lock
s
Suspend
8
End Bl ock
Item: 60 of 64 ~ 1 • M k -<:J 1>- Jil ~· !:';-~
Lab~lues
QIO: 3940

40
..L ar
.. ..
Pre v ious
.. .
Next
, ..
Notes
.
Calcula t o r
. .. ... : . ..
disease) pheochromocytomas, Lisch nodules (pigmented iris hamartomas C)).
41 Neurofibromatosis Mutation in rF2 tumor suppressor gene on chromosome 22. Autosomal dominant. F'indings:
42 type II bilateral acoustic schwannomas, juvenile cataracts, meningiomas, and ependymomas. 1F2
43 affects 2 ears, 2 eyes, and 2 parts of the brain.
44 von Hippel-lindau Deletion of VIIL gene on chromosome 3p ( Il L = 3 letters). utosomal dominant. Characterized
45 disease by development of numerous tumors, both benign and malignant. HARP : H emangioblastomas
(high \<!Scularity with hyperchromatic nuclei ) in retina. brain stem. cerebellum, spine
46
\ ngiomatosis (cg, cm-crnous hemangiomas in skin, mucosa, organs); bilateral Renal cell
47
carcinomas; Pheochromocvtomas.
48 '

49
so
51
52
53
54
55
56
57
58
59
60 •
a
Lock
s
Suspend
8
End Bl ock
Item: 61 of 64 ~ 1 • M k -<:J 1>- Jil ~· !:';-~
QIO: 3944 ..L ar Pre v ious Next Lab~lues Notes Calcula t o r
A A

41 A 6-year-old boy is being followed by a doctor for a genetic disease causing Intellectual disability, short stature, obesity, and hypogonadism.
Fluorescence in-situ hybridization analysis shows a microdeletion on chromosome 15.
42
43
This genetic disorder is also caused by which of the following mechanisms?
44
:
45 A. DNA acetylation
46
B. DNA glycosylation
47
48 c. DNA methylation

49 D. DNA phosphorylation
50
E. DNA ribosylation
51
52
53
54
55
56
57
58
59
60
. 61 •
a
Lock
s
Suspend
8
End Bl ock
Item: 61 of 64 ~ 1 • M k -<:J 1>- Jil ~· !:';-~
QIO: 3944 ..L ar Prev ious Next Lab~lues Notes Calculat o r

41
A
Th e co rrect a nsw er i s c . 82% ch ose this. A

The clinical description and assay results showing a microdeletion on chromosome 15 fit the picture of Prader-Willi syndrome. I n this disease, the paternal
42 copies of the gene located at chromosome 15qll.2-1 3 are deleted or mutated. Lack of expression due to deletion/mutation lea ds to development of
features in Prader-Willi syndrome such as hyperphagia, obesity, intellectual disability, hypogonadism and hypot onia. DNA methylation, a process that plays
43
a part In genomic imprinting, involves addit ion of methyl groups to regions of DNA to silence expression of t he gene. Angelman syndrome Is another
44 example of DNA methylation, but t he maternal copies of chromosome 15qll.2·13 are deleted or mutated in t his disea se.
Angelman synd ome Prader-Willi syndrome Genomic imprinting Polyphagia DNA methylation Hypotoma Gene Hypogonadism Chromosome Intellectual disabioity
45
Deletion ge oetics Chromosome 15 (human) Methylation Obesity DNA Genome Genomocs
46
A i s not correct. 8% chose this.
47 Acetylation Is a form of posttranslation al modification of proteins such as p51, histone, and tubulins at N-t ermin al o -amin e residues like lysine. It Is not a
mechanism of gene silencing in Prad er-Willi syndrome.
48
Post·t ans• ttiona modification Prader-Willi syndrome Gene Lysine Acetylation Histone Gene silencing N-terminus Tubulin Protein
49
B i s not co rrect. 4 % chose this.
50 Glycosylatlon is a posttranslational modification of prote ins in mucosal secretions and extracellular matrix with an N-acetylglucosamine or N-
acetylgalactosamine moiety onto t yrosine or serine residues. It is not a mechanism of gene silencing in Prader-Willi syndrome.
51
Post-translational modification Extracellular m atrix Gene Glycosylation Serine Tyrosine Gene silencing Moiety (chemistry) Extracellular Protein
52 Prader-Willi syndrome Mucous membrane Functional g roup
53 0 is not co rrect . JOfo chose t his.
54 Phosphorylation is the process of adding a phosphate group onto an organic molecule; it plays an important role in energy t ransduction, metabolism,
enzyme regulation, and int racellular signaling. It is not a mechanism of gene silencing in Prader-Willi syndrome.
55 Enzyme Gene Prader-Willi syndr om e Or ganic compound Phosphorylation Phosphate Metabolism Molecule Gene silencing Intr acellular Cofactor {biochemistry)
56 Signal transduction

57 E i s n o t co rrect. JO/o ch ose this.


58 Rlbosylatlon Is a posttranslational modificat ion of an adenosine diphosphate (ADP)-rlbose moiety onto an organic molecule. Although this process occurs
In eukaryotlc cells, it is more commonly associated with the cholera toxin, which ADP-ribosylates G proteins, leading to massive diarrhea. It Is not a
59 mechanism of gene silencing in Prader-Willi syndrome.
Post-translational modification Adenosine diphosphate Gene Cholera toxin Organic compound Cholera Eukaryote Diarrhea ADP- ribosylation Pyrophosphate
60
Molecule Adenosine G protein Functional group Toxin Moiety (chemtstry)
61 •
a
Lock
s
Suspend
8
End Bl ock
Item: 61 of 64 ~. I • M k <:] t> al ~· ~
QIO: 3944 .l. ar Previous Next lab 'Vfl1 ues Notes Calculator

41
Bo tto m Line:
42 Prader-Willi syndrome is a genetic disease arising from several mechanisms, one of which is genetic imprinting through DNA methylation. The disease is
43 characterized by intellectual disability, short stature, obesity, and hypogonadism.
Prader-Willi syndrome Genomic imprinting DNA methylation Hypogonadism Genetic disorder Intellectual disability Methylation Obesity DNA Short stature
44
45
46
I ill ;fi 1!1 I•J f o r yea r:[ 20 1 7 ..
47 FIRST AID FAC T S

48
FA17 p 52.2
49
Genetic term s
50
TERM DEFINITION EXAMPLE
51
Codominance Both alleles contribute to the phenotype of the Blood groups A, B, AB; a 1-antitrypsin
52 heterozygote. deficiency.
53 Variable expressivity Patients with the same genotype have varying 2 patients with neurofibromatosis type 1 ( JFl)
54 phenotypes. may have varying disease severity.
55 Incomplete 'ot all individuals with a mutant genotype BRCAJ gene mutations do not always result in
56 penetrance show the mutant phenotype. breast or ovarian cancer.
57
Pleiotropy One gene contributes to multiple phenotypic Untreated phenylketonuria (PKU) manifests with
58 effects. light skin, intellectual disability, and musty body
59 odor.
60 Anticipation Increased severity or earlier onset of disease in Trinucleotide repeat diseases (cg, Huntington
61 succeeding generations. disease).

6
lock
s
Suspend
0
End Block
Item: 61 of 64 ~ 1 • M k -<:J 1>- Jil ~· !:';-~
QIO: 3944 ..L ar Pre v ious Next Lab~lues Notes Calcula t o r
A A

41 Uniparental disomy Offspring receives 2 copies of a chromosome from Uniparental is euploid (correct number of
42 1 parent and no copies from the other parent. chromosomes), not aneuploid . .\!lost occurrences
43
Heterodlsomy (heterozygous) indicates a meiosis of uniparental disomy (UPD) ..... normal
I error. Isodlsomy (homozrgous) indicates a phenotype. Consider UPD in an individual
44
meiosis II error or postzygotic chromosomal manifesting a recessive disorder when only one
45 duplication of one of a pair of chromosomes, parent is a carrier.
46 and loss of the other of the original pair.
47
48 FA17 p54.1
49 Imprinting At some loci, only one allele is acti\e; the Both Prader-\Villi and Angelman syndromes
50 other is inacti,·e (imprinted/inacti,·ated b) are due to mutation or deletion of genes on
51 methylation). With one allele inactin1ted, chromosome 15.
deletion of the active allele ..... disease.
52
53
Prader-Willi syndrome Maternal imprinting: gene from mom is normally 25% of cases due to maternal uniparental
silent and Paternal gene is deleted/mutated. disorny (two maternally imprinted genes arc
54
Results in hyperphagia, obesity, intellectual received; no paternal gene received).
55 disability, hypogonadism, and hypoton ia.
56 AngeiMan syndrome Paternal imprinting: gene from dad is normally 5% of cases due to paternal uniparenta l disomy
57 silent and Maternal gene is deleted/mutated. (two paternally imprinted genes are received; no
58 Results in inappropriate laughter ("happy materna I gene received).
59 puppet"), seizures, ataxia, and severe
intellectual disability.
60
61 •
a
Lock
s
Suspend
8
End Bl ock
Item: 62 of 64 ~ 1 • M k -<:J 1>- Jil ~· !:';-~
QIO: 1214 ..L ar Previous Next Lab~lues Notes Calculator

42
A

A 74-year-old man comes to the clinic with gastrointestinal bleeding. He has had eight episodes of dark maroon-colored stools in the past month. He
has no associated abdominal or rectal pain or nausea and vomiting. He has a history of diabetes, hypertension, and hypercholesterolemia . Fecal
IA•A] A

43
occult blood tests and colonoscopy were done, and biopsy of the colon showed adenocarcinoma . Genetic analysis of the tumor reveals the presence
44 of a mutant ras oncogene, which is the most common abnormality in human cancer, and it is involved in approximat ely 30% of tumors.

45
To which of the follow1ng families of molecules does ras belong?
46
47 :
A. Cyclin-dependent kinases
48
B. Epidermal growth factor receptor
49
so c. Guanosine triphosphate -binding proteins

51 o. Non-receptor-associated tyrosine kinase


52
E. Transcriptional activators
53
54
55
56
57
58
59
60
61
. 62 •
a
Lock
s
Suspend
8
End Block
Item: 62 of 64 ~. I • M k <:] t> al ~· ~
QIO: 1214 .l. ar Previous Next lab 'Vfl1ues Notes Calculator

42 The co rrect a nswer is c. 38% cho se this.


43 Ra s is a G protein that cycles between two conformations: the activa t ed ras-GTP and the inactivat ed ras-GDP. Mutation of the ras gene results in an
aberrant protein with intact GTP binding but with a loss of GTPase activity, which prevents it from getting inactivat ed, resulting in a constitutively active G
44 protein.
Gene Protein GTPase G protein Guanosine triphosphate Ras subfamily Mutation Conformational isomerism Protein structure
45
A is no t co rrect. 1 4 % cho se this.
46
Cyclin -dependent kinases direct progression of cells through the cell cycle. They are activa t ed by cyclins.
47 Cell cycle Cyclin Cyclin-dependent kinase Kinase

48 B is no t co rrect. 11% cho se this.


49 The oncogenes e rb and HER2 are examples of oncogenes that encode growth factor receptors. Her2 receptors are crucial in det ermining the regimen for
brea st cancer patients.
50 HER2/neu Growth factor Breast cancer Oncogene Cancer

51 D is no t co rrect. 1 7 % cho se this.


52 abl is an example of a proto-oncogene with tyrosine kinase activity. In chronic myeloid leukemia, abl on chromosome 9 is translocat ed to chromosome 22,
fusing it with bcr and crea ting a hybrid tyrosine kinase with potent activity.
53 Chronic myelogenous leukemia leukemia Chromosome 22 (human) Oncogene Tyrosine kinase ABl (gene) Proto-oncogene Philadelphia chromosome Tyrosine

54 Chromosome 9 (human) Chromosome Myeloid Kinase Chromosomal translocation

55 E is no t co rrect. 20 % cho se this.


The myc protein is an example of a transcriptional activator that binds DNA, activating many growth-relat ed genes. All three types of Burkitt lymphoma
56 are characterized by disregulation of the c- MYC gene by one of three chromosomal translocations.
57 Burkitt' s lymphoma Gene Myc Protein Chromosomal translocation DNA lymphoma Activator (genetics) Chromosome Transcription (genetics)

58
59 Botto m Li ne:
60 Ra s is a small GTPase proto-oncogene involved in signal transduction. Increa sed signal through the Ra s pathway lea ds to cell growth, differentiation,
and survival. Ra s is active when it is bound to GTP and inactive when it is bound to GDP. Loss of GTPase activity lea ds to constitutive activa tion of Ra s.
61 GTPase Signal transduction Oncogene Proto-oncogene Cell growth Small GTPase Guanosine triphosphate Ras subfamily Transduction (genetics)
62

6
lock
s
Suspend
0
End Block
Item: 62 of 64 ~ 1 • M k -<:J 1>- Jil ~· !:';-~
QIO: 1214 ..L ar Previous Next Lab~lues Notes Calculator
A A

42 FA17p218.1
43
Oncogenes Cain of function - t cancer risk. eed damage to only one allele of an oncogene.
44 GENE GENE PRODUCT ASSOCIATED NEOPLASM
45 ALK Receptor tyrosine kinase Lung adenocarcinoma
46 BCR-ABL C \IL, ALL
Tyrosine kinase
47
BCL-2 Antiapoptotic molecule (inhibits apoptosis) Follicular and diffuse large B celllp11phomas
48
BRAF Serine/threonine kinase \ lela noma, non-Hodgkin lymphoma, papillar}
49 tiH• roid carcinoma
so c-KIT Cytokine receptor Gastrointestinal stromal tumor (G IST)
51
c-MYC Transcription factor Burkitt lymphoma
52
HER2/ neu (c-erb82) Receptor tr rosine kinase Breast and gastric carcinomas
53
JAK2 Tvrosine kinase C hron ic myeloproli fcrati,·c d isordcrs
54
KRAS CTPase Colon cancer, lung cancer, pancreatic cancer
55
MYCL1 Transcription factor L ung tumor
56
MYCN Transcription factor Neuroblastoma
57
58 RET Receptor tyrosine kinase tf EI 2A and 28, medullary thyroid cancer

59
60 FA17 p220.2

61 Serum tumor markers Tumor markers should not be used as the 1° tool for cancer diagnosis or screen ing. They may be
62 used to monitor tumor recurrence and response to therapy, but definitive diagnosis is made via

a
Lock
s
Suspend
8
End Block
Item: 62 of 64 ~ 1 • M k -<:J 1>- Jil ~· !:';-~
QIO: 1214 ..L ar Previous Next Lab~lues Notes Calculator
A A

42 FA17 p 371 .2
43 Colorectal cancer
44 EPIDEMIOLOGY 1\lost patients are > 50 }ears old.- 25% ha,e a
45 family history.
46 RISK FACTORS Adenomatous and serrated pol} ps, familial
47
cancer syndromes, I BD, tobacco use, diel of
processed meat with low fiber.
48
PRESENTATION Rectosigmoid >ascending> descending. Right side bleeds; left side obstructs.
49
Ascending-exophytic mass, iron deficiency
so anemia, weight loss.
51 Descending-infiltrating mass, partial
52 obstruction, colicky pain , hematochezia.
53 Rarely, presents with S bovis (gcdlolyticus)
bacteremia.
54
55
DIAGNOSIS Iron defi ciency anemia in males (especiall}' >50
years old) and postmenopausa l females raises
56
suspicion.
57 Screen low-risk patients starting at age 50 wilh
58 colonoscopy r.J; allernalives include flexible
59 sigmoidoscopy, fecal occult blood lesting
60
(FOBT), fecal immunochemical testing
(FIT ), and CT colonography. Palienls wilh
61
a first-degree relati,·e who has colon cancer
62 • (:hnulrl hP ~rrPPnPrl vi'!! f"'nlnnncf"nn" !It !lOP-

a
Lock
s
Suspend
8
End Block
Item: 63 of 64 ~ 1 • M k -<:J 1>- Jil ~· !:';-~
QIO: 3933 ..L ar Pre v ious Next Lab~lues Notes Calcula t o r

43
A

A 14-year-old boy presents to the clinic for a follow-up visit. He has a history of asthma that has been inadequately controlled with a rescue Inhaler,
prednisone inhaler, and salmeterol. Despite maximal therapy, he continues to have daily exacerbations. He also reports a history of recurrent sinusitis.
lA• A] A

44 x-ray of the chest demonstrates a heart shadow in the right thorax. ACT scan of the thoracic cavity is shown.
45
46
47
48
49
so
51
52
53
54
55
56
57
58
59
60
61
62
. 63

a
Lock
s
Suspend
8
End Bl ock
43
44
45
46
47
48
49
so
51
52
53
54
A defect In which of the following structures is t he underlying cause of his symptoms?
55
:
56 A . Actin

57
B. Alveolar t ype II cells
58
c. Oynein
59
60 D. Microtubules

61 E. Myosin
62
. 63

a
Lock
s
Suspend
8
End Block
Item: 63 of 64 ~ 1 • M k -<:J 1>- Jil ~· !:';-~
QIO: 3933 ..L ar Previous Next Lab~lues Notes Calculat or
A A
43 Th e co rrect a nsw er i s c. 770/o chose this.
The patient in this vignette most likely has Karta g en er syndro m e. The underlying mechanism is an autosomal recessive Positive end(+)
44 Mocor
defect In the dyn ein motor protein. Ciliary motility occurs when microtubules slide past each other, an act which is Adaptor
45 facilitated by dynein arms. A lack of functional dynein results in impaired ciliary movement throughout the body. The C.ogo
clinica l manifestations of Kartagener syndrome include chronic sinusitis, bronchiectasis, and si tus i n versu s (as shown in
46 the CT In the vignette), where the heart and other major organs are flipped to the opposite side (right vs left) in the
body. Recurrent otitis media is also commonly associated with this syndrome, along woth chronic bronchitis and recurrent
47 pneumonia. Infertility can occur in females due to impaired dliary propulsion of the ovum. Children and young adults may ~
48 present with unrem1tt111g asthma; this patient suffered from daily nighttime symptoms and daytime exacerbations that
were d ifficult to control.
49 Bronchiec •• s Situs i1111ersus Primary ciliary dyskinesia Otitis media Dyneon Sinusiti~ Asthma Bronchitis Pneumonia Protein

so Autosomal recessive Dominance (genetics) Autosome Recessive Microtubule Egg cell Motility Infertility
Chronic obstructive pulmonary disease
51
Negative tnd (-)
52
53 A is n ot co rrect. 6 0/o chose t his.
Actlns are long filaments consisting of multiple globular G-actin subunits. Actlns are another component of the cell's cytoskeleton. They provide tracts
54
upon which myosins operate, either in cell motility, vesicular transport, or muscle contraction . Actins are not affected in Kartagener syndrome.
55 Cytoskeleton Cell migration Actin Vesicle (biology and chemistry) Myosin Motility Muscle contraction Primary ciliary dyskinesia

56 8 is not co r rect. 20/o chose this.


Alveolar type II cells are specialized cells that m ake up about 5% of the lung's gas exchange suoface area. They primarily produce suofactant and serve as
57
a source for new alveolar cell type I and II generation . These cells are not affected In Kaotagener syndrome.
58 Surfactant Alveolar cells Pulmonary alveolus Lung Gas exchange

59 D i s n o t co rrect. 130/o ch ose this.


Mlcrotubules are structural components of cells, consisting of repeating ot3 tubulln dimeric subunits. In
60
addition to providing structural support to the cell's cytoplasm, microtubules also organize into mitotic
61 spindles and flagella in eukaryotic cells, thus playing essential roles in cell division as well as motility. The
body of a cilium contains two central microtubule singlets in addition to the nine outer doublets (called a 9
62 + 2 axoneme-see image). However, these microtubules are not affected In Kartagener syndrome.
Microtubule F~age lum Primary ciliary dyskinesia Cytoplasm Tubu in Eukaryote C. urn Protein dimer
63 •
a
Lock Suspend
s 8
End Bl ock
Item: 63 of 64 ~. I • M k <:] t> al ~· ~
QIO: 3933 .l. ar Previous Next lab 'Vfl1 ues Notes Calculator

• Surfactant Alveolar cells Pulmonary alveolus lung Gas exchange


43
D is no t co rrect. 13% cho se this.
44
Microtubules are structural components of cells, consisting of repea ting aj3 tubulin dimeric subunits. In
45 addition to providing structural support to the cell's cytoplasm, microtubules also organize into mitotic
spindles and flagella in eukaryotic cells, thus playing essential roles in cell division as well as motility. The
46 body of a cilium contains two central microtubule singlets in addition to the nine outer doublets (called a 9
47 + 2 axoneme - see image). However; these microtubules are not affected in Kartagener syndrome.
Microtubule Flagellum Primary ciliary dyskinesia Cytoplasm Tubulin Eukaryote Cilium Protein dimer
48 Spindle apparatus Cell division Mitosis Motility Dimer (chemistry)
49
50
51
52
53
I mage courtesy of Wikimedia
54
55 E is no t co rrect. 2 % cho se this.
Myosins are ATP-driven dimeric molecular motor proteins that move along actin filaments. Functions of myosins include vesicular transport, cell motility,
56 and muscle contraction . Myosins are not affected in Kartagener syndrome.
57 Actin Molecular motor Myosin Protein dimer Motor protein Vesicle (biology and chemistry) Cell migration Motility Microfilament Muscle contraction Protein

58
59 Bo tto m Line:

60 Kartagener syndrome is characterized by chronic sinusitis, bronchiectasis, and situs inversus. It is caused by a defect in the dynein arms that operat e in
all cilia of the body.
61 Primary ciliary dyskinesia Bronchiectasis Situs inversus Sinusitis Dynein Cilium

62
63 •

6
lock
s
Suspend
0
End Block
Item: 63 of 64 ~ 1 • M k -<:J 1>- Jil ~· !:';-~
QIO: 3933 ..L ar Pre v ious Next Lab~lues Notes Calcula t o r
A
43 A 14!;fjiJ!•J fo r yea r : 20 17 ..
FIRST AID FACTS
44
45
FA17p 45.1
46
Cilia structure 9 doublet + 2 singlet arrangement of Kartagener syndrome (1° ciliary dyskinesia)-
47
microtubules (arrows in ). immotile cilia due to a dynein arm defect.
48 Basal body (base of cilium belo" cell Results in l male and female fertility due to
49 membrane) consists of 9 microtubule triplets immotile sperm and dysfunctional fallopian
so (arrow in I}]) '' ith no central microtubules. Lube cilia. respecli\'ely; t risk of ectopic
51 Axonemal dynein-ATPase that links peripheral pregnancy. Can cause bronchiectasis,
9 doublets and causes bending of cilium br recurrent sinusitis, chronic ear infections,
52
differential sliding of doublets. conductive hearing loss, and situs invcrsus (cg,
53
dextrocardia on CXR ~) .
54
55
56
57
58
59
60
61
62
63 •
a
Lock
s
Suspend
8
End Bl ock
43
44
45
46
47
48
49
so
51 FA17 p 44.3
52 Microtubule Cylindrical outer structure composed of a Drugs that act on microtubules (\ 1icrotubules
53
Positive helical arra) of pol) merized hctcrodimers Gel C onstructed Very Poor!) ):
54 end(+} of c:x- and ~-tubulin. Each dimer has 2 CTP \llebendazole (antihelminthic)
55
HeterodimerA bound. Incorporated into Aagclla, cilia, mitotic Griseofulvin (antifungal)
spindles. Crows slowly, collapses quickly. • Colchicine (antigout)
56
Also involved in slow axoplasmic transport in • VincristineNvinblastine (anticancer)
57 neurons. • Paclitaxcl (anticancer)
58 Molecular motor proteins-transport cel lular
59 Protofilament - --r-:-1 cargo toward opposite ends of microtubule
60
tracks.
Nega~ve Dynein- retrograde lo microtubule (+ - -). 1\egal·ive end l'\ear ~ucleus
61
end(-) Kinesin-anterograde to microtubule(- - +). Positive end Points to Periphery
62
63 •
a
Lock
s
Suspend
8
End Block
Item: 64 of 64 ~ 1 • M k -<:J 1>- Jil ~· !:';-~
QIO: 1200 ..L ar Pre v ious Next Lab~lues Notes Calcula t o r
A A

44 In the genetics clinic, a 12-yea r-old boy is diagnosed with a rare mitochondrial myopathy. The clinician explains to the family that his condition Is
caused by a mutation in the gene encoding a mutated transfer RNA (tRNA) molecule. This mutated tRNA can no longer be conjugated to its target
45
amino acid.
46 c-A B
47
,c ...
A
48
49 E
so
51
52
D
53
54
55 Where Is the mutation most likely located?
56
:
57 A

58 B
59
c
60
61 0

62 E
63
• 64 •
a
Lock
s
Suspend
8
End Bl ock
Item: 64 of 64 ~. I • M k <:] t> al ~· ~
QIO: 1200 .l. ar Previous Next lab 'Vfl1 ues Notes Calculator

44
The co rrect a nswer is B. 5 1% cho se this.
45 B is the CCA sequence at the 3' end of the tRNA, which has to recognize the correct aminoacyl-tRNA synthetase in order to pair with the appropriate
46 amino acid. This is the most likely site of mutation.
Amino acid Transfer RNA Aminoacyl tRNA synthetase Mutation Aminoacyl-tRNA
47
A is no t co rrect. 20 % cho se this.
48 The mutation most likely occurred at the 3' end of the tRNA, where the amino acid is conjugated to the tRNA . Site A is the 5 ' end of the tRNA . A
49 mutation at site A would not prevent the tRNA from being conjugated to its target amino acid.
Amino acid Transfer RNA Mutation Directionality (molecular biology)
50
c is no t co rrect. 6 % cho se this.
51 The mutation most likely occurred at the 3 ' end of the tRNA, where the amino acid is conjugated to the tRNA . A mutation at site C would not prevent the
52 tRNA from being conjugated to its target amino acid.
Amino acid Transfer RNA Mutation
53
D is no t co rrect. 18% cho se this.
54 D is the anticodon region of tRNA, which base-pairs with the codon on messenger RNA (mRNA ). A mutation here would impair binding of the tRNA to the
55 m RNA, not the binding of the tRNA to the amino acid.
Messenger RNA Anticodon Transfer RNA Amino acid Mutation Genetic code RNA Base pair Nucleotide
56
E is no t co rrect. 5 % cho se this.
57 The mutation most likely occurred at the 3 ' end of the tRNA, where the amino acid binds.
58 Amino acid Transfer RNA Mutation

59
60 Botto m Li ne:
61 Amino acids are covalently bound to the 3 ' end of tRNA molecules. This reaction is catalyzed by aminoacyl-tRNA synthetase. tRNA molecules have a
CCA triplet on the 3 ' end.
62 Transfer RNA Amino acid Aminoacyl tRNA synthetase Aminoacyl-tRNA Covalent bond Catalysis

63
64

6
lock
s
Suspend
0
End Block
Item: 64 of 64 ~ 1 • M k -<:J 1>- Jil ~· !:';-~
QIO: 1200 ..L ar Pre v ious Next Lab~lues Notes Calcula t o r
A A

44 FA17 p 40.1
45 tRNA
46 Structure 75-90 nucleotides, zo structure, cloverleaf form, anticodon end is opposite 3' aminoacyl end. II
47 tRl As, both eukaryotic and prokar)olic, have CCA at 3' end along with a high percentage of
chemically modified bases. The amino acid is covalently bound to the 3' end of the tR 'A. CC.\
48
C an C arry \ mino acids.
49
T-arm: contains the T\f'C (riboth} midinc, pseudouridine, cytidine) sequence necessary for tR 'A-
so ribosome binding. T-arm Tethers tR A molecule to ribosome.
51 0-arm: contains dihydrouridine residues necessary for tR~ recognition by the correct aminoacyl-
52 tRJ A svnthetase. D -arm D etects theIR A b\ aminoacd-tRN svnthetase.
~ i "' ~

Acceptor stem: the 5'-CC -3' is the amino acid acceptor site.
53
54
Charging Aminoacyl-tR1 A synthetase (I per amino acid; "matchmaker''; uses ATP) scrutinizes amino acid
before and after it binds to tRl A. If incorrect, bond is hydrolyzed. The amino acid-tRNA bond
55
has en erg}' for formation of peptide bond. A mischarged tR reads usual codon but inserts
56 wrong amino acid.
57 Aminoacyl-tR A synthetase and binding of chargecl t R A to the codon arc responsible for
58 acc uracy of am ino acid selection.
59 Structure Charging Pairing
(aminoacytation) (codon-anticodon)
60
Amino acid-.... Amino acid,
0 ...... • 3'
61
62
Acceptor stem{OH- i 3
.
0 ...... . 3'
c
c
c
c
s· s· s·
63 Am1noacyHRNA
64 • T-arm ... __ syntl etase
.tTn. .. . ,,., • 1\1'\
IF2

a
Lock
s
Suspend
8
End Bl ock
Item: 64 of 64 ~ 1 • M k -<:J 1>- Jil ~· !:';-~
QIO: 1200

44
..L ar Previous
.. •
Next
. .
Lab~lues
. .. ..
Notes Calculator
. ...
( ... ' ( ... '
45 I

46 Variable arm_/

47
48
Anticodon
loop • c
• ' -wobble
position
..( Anticodon (5• CAU 3"1 -c • • <
.. . • c
49 mRNA
L,--1
so Codon
(5' AUG 3')
51
52
53 FA17 p 38.3

54 RNA polymerases

55 Eukaryotes RNA polymerase ! makes rRN (most I, II, and Ill are numbered in the same order
numerous Rt A, rampant). that their products are used in protein
56
Rt A polymerase II makes m R A (largest R A, synthesis: rR N , mRt A, then tRNA.
57
massive). mRNA is read 5' to 3'. a -amanitin, found in Amanita phalloides (dealh
58 RNA polymerase Ill makes 5S rR , 1R cap mushrooms}, inhibits R 'A polymerase II.
59 (smallest R 'A, t iny}. Causes severe hepatotoxicity if ingested.
60 l\o proofreading function, but can i11 il iale Actinomycin D inhibits R1 A polymcmse in
chains. RNA polymerase II opens Dl A at both prokaryotes and eukaryotes.
61
promoter site.
62
Prokaryotes I RNA polymerase (mult isubunit complex) Ri fampin inhibits D. A-dependent R:-.JA
63
makes all 3 kinds of R 1 • polymerase in prokaryotes.
64 •
a
Lock
s
Su spend
8
End Block

Potrebbero piacerti anche